Download as pdf or txt
Download as pdf or txt
You are on page 1of 200

1 Chapter 1 | Fundamentals of Mathematics

Chapter
Fundamentals of
1 Mathematics
LECTURE-1 ELEMENTARY NUMBER THEORY

Non-positive integers
The set {......., –3, –2, –1, 0} is the set of all non-
positive integers.

Perfect numbers
If sum of proper divisors of a number is the number
itself then the number is known as perfect number.
e.g., 6, 28 ..... etc.

Prime numbers
Except 1 each natural number which is divisible by 1
and itself only is called as prime number. e.g., 2, 3, 5,
7, 11, 13, 17, 19, 23, 29, 31, ..... etc.
• There are total 25 prime numbers up to 100
• There are total 46 prime numbers up to 200
Natural numbers
• 2 is the only even prime number and the least
Counting numbers 1, 2, 3, 4, 5, ..... are known as prime number.
natural numbers. The set of all natural numbers • 1 is neither prime nor composite number.
can be represented by • There are infinite prime numbers.
N = {1, 2, 3, 4, 5, ....} • A list of all prime numbers up to 100 is given
below.
Whole numbers
If we include 0 among the natural numbers, then Table of prime Numbers (1-100):
2 11 23 31 41 53 61 71 83 97
the numbers 0, 1, 2, 3, 4, 5 ..... are called whole
3 13 29 37 43 59 67 73 89
numbers. The set of whole numbers can be
5 17 47 79
represented by 7 19
W = {0, 1, 2, 3, 4, 5, ....}
Clearly, every natural number is a whole number but Test to find whether a given number is a prime
0 is a whole number which is not a natural number. Step-1 : Select a least positive integer n such that
n2 > given number.
Integers Step-2 : Test the divisibility of given number by
All counting numbers and their negatives every prime number less than n.
Step-3 : The given number is prime only if it is not
including zero are known as integers. The set of
divisible by any of these primes.
integers can be represented by
Z or I = {..... –4, –3, –2, –1, 0, 1, 2, 3, 4, .....} Even numbers
All those numbers which are exactly divisible by 2
Positive integers are called even numbers,
The set I+ = {1, 2, 3, 4, ....} is the set of all positive e.g. 2, 6, 8, 10 etc.
integers, Clearly, positive integers and natural
numbers are synonyms. Odd numbers
All those numbers which are not exactly divisible
Negative integers by 2 are called odd numbers,
The set I– = {–1, –2, –3, .....} is the set of all negative e.g. 1, 3, 5, 7 etc.
integers.
0 is neither positive nor negative. Co-prime Numbers
If the H.C.F. of the given numbers (not necessarily
Non-negative integers prime) is 1 then they are known as co-prime
The set {0, 1, 2, 3, ......} is the set of all non-negative numbers.
integers.
Chapter 1 | Fundamentals of Mathematics 2
e.g. 4, 9 are co-prime as H.C.F. of (4, 9) = 1. OR
e.g. 3, 5 are co-prime as H.C.F. of (3, 5) = 1. A terminating decimal doesn't keep going. A
e.g. 4, 5 are co-prime as H.C.F. of (4, 5) = 1. terminating decimal will have finite number of
digits after the decimal point.
Composite numbers 3 8 5 25
All natural numbers, which are not prime are = 0.75, = 0.8, = 1.25, = 1.5625
4 10 4 16
composite numbers. If C is the set of composite Method to convert non-terminating decimal to the
number then C = {4, 6, 8, 9, 10, 12 .....} form p/q.
In a non-terminating decimal. we have two types
Imaginary numbers of decimal representations
All the numbers whose square is negative are called (i) Pure recurring decimal
imaginary numbers. (ii) Mixed recurring decimal
e.g. 3i, –4i, ....; where i = −1 . (i) Pure recurring decimal
It is a decimal representation in which all the
Complex numbers digits after the decimal point are repeated.
A number of the form a + ib, where a, b  R and i Following are the steps to convert it in the
form p/q.
= −1 is called a complex number. Step-1 : Denote pure recurring decimal as x.
Complex number is usually denoted by z and the
Step-2 : Write the number in decimal form by
set of all complex numbers is represented by
removing bar from top of repeating digits.
C = {(x+iy) : x, y  R, i = −1 } Step-3 : Count the number of digits having bar
NWIQRC on their heads.
Step-4 : Multiply the repeating decimal by 10,
Twin primes 100, 1000, ... depending upon 1 place
Prime numbers differing by two are called twin repetition, 2 place repetition, 3 place
primes, repetition and so on present in decimal
e.g. (3, 5), (5, 7), (11, 13) etc, are called twin primes. number.
Step-5 : Subtract the number obtained in step
Prime triplet 2 from a number obtained in step 4.
A set of three consecutive primes differing by 2, Step-6 : Find the value of x in the form p/q.
such as (3, 5, 7) is called a prime triplet (ii) Mixed recurring decimal
It is a decimal representation in which there
are one or more digits present before the
Rational numbers
repeating digits after decimal point.
The rational numbers are all the numbers that can
Following are the steps to convert it to the
p form p/q.
be expressed in the form of , where p and q are
q Step-1 : Denote mixed recurring decimal as x.
integers and coprime and q  0. Step-2 : Count the number of digits after the
2 decimal point which do not have bar on them.
e.g., , – 3, 0, 4.33 etc. Let it be 'n'.
3
Step-3 : Multiply both sides of x by 10n to get
only repeating decimal numbers on the right
Rational numbers between two numbers side of the decimal point.
One way to find a rational number between two Step-4 : Further use the method of converting
rational numbers is to find their average, called pure recurring decimal to the form p/q and
mean. get the value of x.
To find a rational number between x and y, we will
x+y Irrational numbers
find the mean of x and y. i.e. is a rational
2 A number is called an irrational number, if it
number lying between x and y. This number will cannot be written in the form p/q, where p & q are
be the mid-value of that two numbers. integers and q  0. All Non-terminating & Non-
repeating decimal numbers are Irrational
Decimal expansion of rational numbers numbers.
Every rational number can be expressed as
terminating decimal or non-terminating but E.g. 2, 3,3 2,2 + 3, 2 + 3 ,  , etc ....
repeating decimals.
Decimal expansion of irrational numbers
Terminating decimal (The remainder becomes Every irrational number can be expressed as non-
zero) terminating and non-repeating decimal.
The word "terminate" means "end". A decimal that e.g. 2 = 1.4142135 ....... .
ends is a terminating decimal.
3 Chapter 1 | Fundamentals of Mathematics
Remark : To find an irrational number between two Step 2: If r = 0, then b is the HCF of a and b. If r 
0, apply division Lemma to b and r.
numbers a and b is ab . Step 3: Continue the process till r is 0. The divisor
at this stage is the HCF of a and b.
(t) Real numbers This procedure has to work because the HCF of a
The rational and irrational numbers combined and b is same as HCF of b and r.
together are called real numbers,
13 2 3 Example 1.1
e.g. , , − , 3,4 + 2, etc. are real numbers.
21 5 7
Express the following mixed recurring decimals in the
The set of real numbers is denoted by R. form p/q .
Note, that the sum or difference of a rational and
an irrational number is irrational, and the product (i) 0.32 (ii) 0.123 (iii) 15.712
of a non zero rational and an irrational number is (iv) 23.43
irrational, Solution : (i) Let x = 0.32
2 Clearly, there is just one digit on the right side of
e.g. 3 + 2,4 − 3, − 5,4 3, −7 5 are all
3 the decimal point which is without bar. So, we
irrational numbers. multiply both sides of x by 10 so that only the
repeating decimal is left on the right side of the
Operations on real numbers decimal point.
Following are some useful results on real  10x = 3.2  10x = 3 + 0.2
numbers. 2
(i) Negative of an irrational number is an  10x = 3 +
irrational number. 9
(ii) The sum or difference of a rational number 93+ 2 29 29
 10x = x=  x= .
and an irrational number is an irrational number. 9 9 90
(iii) The product of a non-zero rational number (ii) Let x = 0.123
and an irrational number is an irrational number. Clearly, there are two digits on the right side of the
(iv) The sum, difference, product and quotient of decimal point which is without bar. So, we
two irrational numbers need not be an irrational multiply both sides of equation by 102 = 100 so
number. that only the repeating decimal is left on the right
side of the decimal point.
Statement of Euclid’s Division Lemma
 100x = 12.3  100x = 12 + 0.3
Let ‘a’ and ‘b’ be any two positive integers. Then,
there exists unique integers ‘q’ and ‘r’ such that a 3
 100x = 12 +
= b q + r, where 0  r < b. If b|a, then r = 0. 9
This can easily be remembered as follows 12  9 + 3 108 + 3
 100x =  100x =
q 9 9
b a 111 111 37
 100x =  x= =
r 9 900 300
This can be restated as follows : (iii) Let x = 15.712
Dividend = Divisor × Quotient + Remainder. Clearly, there is just one digit on the right side of
e.g. Consider positive integer 47 and 4. the decimal point which is without bar. So, we
Sol. 47 = 4 × 11 + 3 multiply both sides of x by 10 so that only the
a = 47, b = 4, q = 11, r = 3 repeating decimal is left on the right side of the
e.g. Consider positive integer 91 and 11. decimal point.
Sol. 91 = 11 × 8 + 3  10x = 157.12
a = 91, b = 11, q = 8, r = 3
 10x = 157 + 0.12
12 4
Remark : In Division Lemma, q or r may be 0 but r is  10x = 157 +  10x = 157 +
always less than b.
99 33
157  33 + 4 5181 + 4
 10x =  10x =
Euclid’s Division Algorithm 33 33
If ‘a’ and ‘b’ are positive integers such that a = bq 5185 5185 1037
 10x = x= = .
+ r, then every common divisor of ‘a’ and ‘b’ is a 33 330 66
common divisor of ‘b’ and ‘r’, and vice-versa. The (iv) Let x = 23.43  x = 23.434343 …..(1)
HCF of positive integers a and b where a > b is Multiplying both sides of (1) by 100, we get
obtained as follows. 100x = 2343.4343 …..(2)
Step 1: Apply Euclid’s division Lemma to a and b. Subtracting (1) from (2) we get
That is, find whole numbers q and r such that 100x – x = (2343.4343 .........) – (23.4343 ........)
a = bq + r, 0  r < b
Chapter 1 | Fundamentals of Mathematics 4
2320 Solution : Applying Euclid’s division lemma on 657
 99x = 2320  x = and 963.
99
2320 963 = 657 × 1 + 306
Hence, 23.43 = 657 = 306 × 2 + 45
99 306 = 45 × 6 + 36
Alter method
45 = 36 × 1 + 9
43 36 = 9 × 4 + 0
We have, 23.43 = 23 + 0.43 = 23 +
99 So, the H.C.F of 657 and 963 is 9.
43 Given : 657 x + 963 × (– 15) = H.C.F of 657 and 963.
Using the above rule, we have 0.43 =
99 657 x + 963 × (– 15) = 9
23  99 + 43 2277 + 43 2320 657 x = 9 + 963 × 15
 23.43 = = = 657 x = 14454
99 99 99
14454
x= = 22.
Example 1.2 657
Show that one and only one out of n, n + 2 or n + 4 is
divisible by 3, where n is any positive integer. NUMBERS TO REMEMBER :
Solution : Let n is any positive integer of form 3q + r Number Square Cube Sq. Root
where 0  r < 3 2 4 8 1.41
Case-I When r = 0 3 9 27 1.73
n = 3q, which is divisible by 3. 4 16 64 2
n + 2 = 3q + 2 5 25 125 2.24
 n + 2 leaves remainder 2, when divided by 3 6 36 216 2.45
 n + 2 is not divisible by 3 7 49 343 2.65
n + 4  3q + 4 = 3 (q + 1) + 1 8 64 512 2.83
 n + 4 is not divisible by 3 9 81 729 3
Thus, n is divisible by 3 but n + 2 and n + 4 is not 10 100 1000 3.16
divisible by 3. 11 121 1331
Case-II When r = 1
12 144 1728
n = 3q + 1
13 169 2197
n + 2 = 3q + 3
14 196 2744
and n + 4 = 3q + 5
Thus n + 2 is divisible by 3 but n and n + 4 are not 15 225 3375
divisible by 3. 16 256 4096
Case-III When r = 2 17 289 4913
n = 3q + 2 18 324 5832
n + 2 = 3q + 4 19 361 6859
and n + 4 = 3q + 6 20 400 8000
Thus n + 4 is divisible by 3 but n and n + 2 is not
divisible by 3. Note
1. Square of a real number is always non negative
Example 1.3 (i.e. x2  0)
2. Square root of a positive number is always
Use Euclid's division algorithm to find the HCF of 441,
567, 693. positive e.g. =2
Solution : In order to find the HCF of 441, 567 and 693, 3. = |x|, x  R
we first find the HCF of 441 and 567 by Euclid's
division algorithm.
Using division algorithm, we get DIVISIBLITY
567 = 441 × 1 + 126 Division Algorithm : General representation of
441 = 126 × 3 + 63 Dividend Remainder
126 = 63 × 2 + 0 result is, = Quotient +
Divisor Divisor
So, HCF (567, 441) = 63
Now, we find the HCF of 63 and 693 Dividend = (Divisor × Quotient) + Remainder
693 = 63 × 11 + 0
 HCF (63, 693) = 63 Example 1.5
Hence HCF (441, 567, 693) = 63 On dividing 15968 by a certain number, the quotient
is 89 and the remainder is 37. Find the divisor.
Example 1.4 Solution :
If the H.C.F of 657 and 963 is expressible in the form Dividend − Remainder 15968 − 37
Divisor = = = 179
657 x + 963 × (– 15), find x. Quotient 89
5 Chapter 1 | Fundamentals of Mathematics
HCF and LCM of numbers
HCF and LCM of numbers can be determined by
Note prime factorization. This is nothing but an
1. (xn – an) is divisible by (x – a) for all the values application of the fundamental theorem of
of n. arithmetic.
2. (xn – an) is divisible by (x + a) and (x – a) for all HCF = Product of the smallest power of each
the even values of n. common factor.
3. (xn + an) is divisible by (x + a) for all the odd LCM = Product of the biggest power of each prime
values of n. factor
Let a and b be natural numbers. Then their
HCF × LCM = a × b
Test of Divisibility :
No. Divisibility Test Remark : LCM is always divisible by HCF.
2 Unit digit should be 0 or even
The sum of digits of no. should be divisible (i) A number on being divided by d1 and d2
3 successively leaves the remainders r1 and r2,
by 3
The number formed by last 2 digits of given respectively. If the number is divided by d1 ×
4 d2, then the remainder is (d1 × r3 + r1).
no. should be divisible by 4.
5 Unit digit should be 0 or 5. e.g. A number on being divided by 10 and 11
successively leaves the remainder 5 and 7,
6 Number should be divisible by 2 & 3 both
respectively. Find the remainder when the
Double the last digit of given number and
same number of divided by 110.
7 subtract from remaining number the result
Sol. The required remainder
should be or divisible by 7.
= d1 × r2 + r1 = 10 × 7 + 5 = 75.
The number formed by last 3 digits of given
8
no. should be divisible by 8.
(ii) To find the number of numbers divisible by a
Sum of digits of given no. should be divisible certain integer.
9
by 9 e.g. (i) How many numbers up to 532 are
The difference between sums of the digits at divisible by 15?
11 even & at odd places should be zero or Sol. We divide 532 by 15.
multiple of 11. 532 = 35 × 15 + 7
Last 2 digits of the number should be 00, 25, The quotient obtained is the required number
25
50 or 75. of numbers. Thus there are 35 such numbers.
e.g. (ii) How many numbers up to 300 are
Example 1.6 divisible by 5 and 7 together?
Prove that : Sol. LCM of 5 and 7 = 35
We divide 300 by 35
(a) the sum ab + ba is multiple of 11; 300 = 8 × 35 + 20
(b) a three-digit number written by one and the same Thus there are 8 such numbers.
digit is entirely divisible by 37.
Solution : (a) ab + ba =(10a + b) + (10b + a) (iii) Two numbers when divided by a certain divisor
= 11(a + b); give remainders r1 and r2. When their sum is
(b) aaa = 100a + 10a + a = 111a = 37.3a. divided by the same divisor, the remainder is r3.
The divisor is given by r1 + r2 – r3.
e.g. Two numbers when divided by a certain
Example 1.7
divisor give remainders 437 and 298,
Prove that the difference 1025 – 7 is divisible by 3. respectively. When their sum is divided by the
Solution : Write the given difference in the form 1025 same divisor, the remainder is 236. Find the
– 7 = (1025 – 1) – 6. The number 1025 – 1 = 99.9 is divisor.
25digits Sol. The required divisor
divisible digits by 3(and 9). Since the numbers (1025 – = 437 + 298 – 236 = 499.
1) and 6 are divisible by 3, the number 1025 – 7, being
their difference, is also divisible by 3 without a (iv) Product of two numbers = LCM of the
remainder. numbers × HCF of the numbers.
e.g. The HCF and the LCM of any two numbers
are 63 and 1260, respectively. If one of the
Fundamental Theorem of Arithmetic two numbers is 315, find the other number.
Every composite number can be expressed as a Sol. The required number =
product of primes, and this factorisation is unique, LCM  HCF 1260  63
except for the order in which the prime factors = = 252
First number 315
occurs.
Chapter 1 | Fundamentals of Mathematics 6
(v) To find the greatest number that will exactly = HCF of (772 – 5) and (2778 – 5)
divide x, y and z. = HCF of 767 and 2773 = 59.
Required number = HCF of x, y, and z. e.g. Find the greatest number which on
e.g. Find the greatest number that will exactly dividing 152, 277 and 427 leaves same
divide 200 and 320. remainder.
Sol. The required greatest number Sol. The required greatest number.
= HCF of 200 and 320 = 40. = HCF of |x – y|, |y – z| and |z – x|
= HCF of |152 – 277|, |277 – 427| and |427 – 152|
(vi) To find the greatest number that will divide x, = HCF of 125, 150 and 275 = 25.
y, and z leaving remainders a, b, and c,
respectively. (xi) To find the n-digit greatest number which,
Required number = HCF of (x – a), (y – b) and when divided by x, y and z
(z – c). (a) leaves no remainder (i.e., exactly
e.g. Find the greatest number that will divide divisible)
148, 246 and 623 leaving remainders 4, 6 and Step-1 : LCM of x, y and z = L
11, respectively. L ndigit greatest number
Sol. The required greatest number Step-2 :
= HCF of (148 – 4), (246 – 6) and (623 – 11), Remainder = R
i.e. HCF of 144, 240 and 612 = 12. Step-3 : Required number =n-digit greatest
number – R
(vii) To find the least number which is exactly (b) leaves reminder K in each case
divisible by x, y and z. Required number = (n-digit greatest number
Required number = LCM of x, y and z. – R) + K.
e.g. What is the smallest number which is e.g. Find the greatest number of 4 digits
exactly divisible by 36, 45, 63 and 80? which, when divided by 12, 18, 21 and 28,
Sol. The required smallest number leaves 3 as a remainder in each case.
= LCM of 36, 45, 63 and 80 = 5040. Sol. LCM of 12, 18. 21 and 28 = 252.
252)9999(39
(viii) To find the least number which when 9828
divided by x, y and z leaves the remainders a, 171
b and c respectively, such that (x – a) = (y –
 The required number = (9999 – 171) + 3
b) = (z – c) = k (say).
= 9931.
 Required number = (LCM of x, y and z) – k. (xii) To find the n-digit smallest number which
e.g. Find the least number which when when divided by x, y and z
divided by 36, 48 and 64 leaves the (a) leaves no remainder (i.e. exactly divisible)
remainders 25, 37 and 53, respectively. Step-1 : LCM of x, y and z = L
Sol. Since (36 – 25) – (48 – 37) = (64 – 53) =
11, therefore the required smallest number L ndigit smallest number
Step-2 :
(LCM of 36, 48 and 64) –11 = 576 – 11 = 565. Remainder = R
Step-3 : Required number = n-digit smallest
(ix) To find the least number which when divided by number + (L – R).
x, y and z leaves the same remainder r in each (b) leaves remainder K in each case.
case. Required number = n-digit smallest number
Required number = (LCM of x, y, and z) + r. + (L – R) + K.
e.g. Find the least number which when divided e.g. (a) Find the least number of four digits
by 12, 16 and 18, will leave a remainder 5 in which is divisible by 4, 6. 8 and 10.
each case. Sol. LCM of 4, 6, 8, and 10 = 120.
Sol. The required smallest number 120)1000(8
= (LCM of 12, 16 and 18) + 5 = 144 + 5 960
= 149. 40
(x) To find the greatest number that will divide x, y  The required number = 1000 + (120 – 40)
and z leaving the same remainder in each case. = 1080.
(a) When the value of remainder r is given: e.g. (b) Find the smallest 4-digit number, such
Required number = HCF of (x – r), (y – r) and that when divided by 12, 18, 21 and 28, it
(z – r). leaves remainder 3 in each case.
(b) When the value of reminder is not given: Sol. LCM of 12. 18, 21 and 28 = 252.
Required number = HCF of |x – y|, |y – z| and 252)1000(3
|z – x|. 756
e.g. Find the greatest number which will divide 244
772 and 2778 so as to leave the remainder 5 in
 The required number = 1000 + (252 –
each case.
244) + 3 = 1011.
Sol. The required greatest number
7 Chapter 1 | Fundamentals of Mathematics
Surds
(xiii) HCF and LCM of fractions Any root of an arithmetical number which cannot
a c e HCFof numerators be completely found is called surd.
HCF of , , =
b d f LCM of denominators e.g. etc. 3 2, 4 5, 3 7 are all surd.
HCF(a, c, e)
=
LCM(b, d, f) Pure Surds
A surd which consists of purely an irrational
a c e LCMof numerators
LCM of , , = number expressed as n a , a  Q where a  xn is
b d f HCF of denominators
called a pure surd. e.g. 3 7 , 5 5, 3 8 etc.
LCM(a, c, e)
=
HCF(b, d, f)
Mixed surd
A surd when multiplied with a rational number
(xiv) For any three positive integers p, q, r becomes a mixed surd.
HCF (p, q, r) × LCM (p, q, r)  p × q × r. However,
e.g. 2 3 3, 4 5 5, 2 3 etc.
the following result hold good:
Mixed surds can be written as a pure surd.
p.q.rHCF ( p, q, r )
LCM (p, q, r) = e.g. 2  3 3 = 3 3  8 = 3 24 , 2 5 = 20
HCF ( p, q ) .HCF ( q, r ) .HCF ( p, r )
Similar surd or Like surd
Example 1.8 Two or more surds are said to be similar or like
Given that HCF (306, 657) = 9. Find LCM (306, 657). when they can be reduced as to have the same
Solution : HCF (306, 657) = 9 means HCF of 306 and irrational factor. e.g. 45 and 80 are similar
657 = 9 surds as they can be expressed as 3 5 and 4 5 .
Required LCM (306, 657) means required LCM of 306 The sum of similar surds can be expressed as a
and 657.
mixed surd. e.g. sum is 7 5 .
For any two positive integers:
Product of the numbers
their LCM (a, b) = Order of Surd
HCF(a, b) The order of surd is indicated by the number
306  657 denoting the roots i.e. 4 2 , 3 5 , 6 7 are surd
i.e., LCM (306, 657) = = 22,338
9 of 4th, 3rd and 6th order respectively.

Example 1.9 Surd of same order


Surds are said to be of same order or equiradical
In a morning walk three persons step off together, if they all have the same root symbols. e.g.
their steps measure 80 cm, 85 cm and 90 cm 5
respectively. 5, a3 ,(a + x) 2 are all the surds of 2nd order.
What is the minimum distance each should walk so that A surd of second order is called quadratic surd
they can cover the distance in complete steps? 3
4 , 3 25 all surds of 3rd order or cubic surds.
Solution : Required minimum distance each should
walk so, that they can cover the distance in complete
Simple Surds
step is the L.C.M. of 80 cm, 85 cm and 90 cm
Surds consisting of one term only are called
80 = 24 × 5
simple surds.
85 = 5 × 17
90 = 2 × 32 × 5 e.g. 5 2, 3 3, 3 a2bc etc. are simple surds or
LCM = 24 × 32 × 51 × 171 Monomial surds.
LCM = 16 × 9 × 5 × 17
LCM = 12240 cm = 122 m 40 cm. Compound Surd
An expression consisting of two or more simple
surds connected by (+) or (–) sign is called a
Example 1.10
compound surd.
Find the greatest number of six digits exactly divisible e.g. 5 2 + 4 3, 3 + 2, 3 − 5 .
by 15, 24 and 36.
Solution : We have : Binomial surd
15 = 3 × 5; 24 = 23 × 3; 36 = 22 × 32 A compound surd consisting of two terms is called
LCM (15, 24, 36) = 23 × 32 × 5 = 8 × 9 × 5 = 360 Binomial surd.
Now the greatest six-digit number is 999999. Two binomial quadratic surds which differ only in
If we divide 999999 by 360, then q = 2777 and r = 279 sign which connects their terms are said to be
 Required number = 999999 – 279 = 999720. Conjugate or Complementary to each other.
Chapter 1 | Fundamentals of Mathematics 8
e.g. 3 + 2 and 3 − 2 ; 2 5 + 7 and 1
Thus, x2 + = (3 − 2 2)2 + (3 + 2 2)2
2 5 − 7 are examples of conjugate surds. Every x2
binomial quadratic surd is rationalised when = (3)2 + (2 2)2 − 2 3 2 2 + (3)2 + (2 2)2 + 23 2 2
multiplied to its conjugate surd. = 9 + 8 − 12 2 + 9 + 8 + 12 2 = 34

Example 1.11 Example 1.15


Simply (make the denominator rational)
1
12 Rationalise the denominator of
. 3 − 2 −1
3+ 5 −2 2
1 1 3 + 2 +1
12(3 + 5 + 2 2 ) Solution : = 
Solution : The expression = 3 − 2 −1 3 − ( 2 + 1) 3 + 2 +1
(3 + 5 )2 − (2 2 )2
3 + 2 +1 2 + 3 +2  2 + 6 +2
12(3 + 5 + 2 2 ) 2(3 + 5 + 2 2 )( 5 − 1) = = = − 
= = ( 3)2 − ( 2 + 1)2 −2 2  4 
6+6 2 ( 5 + 1)  ( 5 − 1)
2(2 + 2 5 + 2 10 − 2 2 )
= = 1 + 5 + 10 − 2 .
4 INDICES
Some useful Formulae
Example 1.12 (i) am × an = am + n (ii) am ÷ an = am – n
m m

Find the square root of 7 + 2 10 . a n  bn
(iii) (am)n = (an)m = amn (iv)   = 
b a
Solution : Let 7 + 2 10 = x + y . Squaring, (v) am ÷ b–n = am × bn
x + y + 2 xy = 7 + 2 10 (vi) ( n a ) = a, where n  N, n  2 and 'a' is
n

Hence x + y = 7 and xy = 10. These two relations give positive rational number
x = 5, y = 2. Hence 7 + 2 10 = 5 + 2 . (vii) n a n b = n ab , where n  N, n  2 and 'a', 'b'
are non-negative rational numbers.
Remark : symbol stands for the positive square n
a na
root only. (viii) = , where n  N, n  2 and 'a', 'b' are
n
b b
rational numbers.
Example 1.13
(ix) m n a = mn a = n m a , where m, n  N, m, n  2
1
If x = , find the value of x3 – x2 – 11x + 4 and 'a' is a positive rational number
2+ 3
(x)
n m
( a k )m = n a k = mn a km , where m, n  N, m,
1 2− 3 2− 3
Solution : As x =  = n  2, 'k' are positive integers and 'a' is a positive
2 + 3 2 − 3 (2)2 − ( 3 )2
rational number
2− 3 (xi) a  a = a; (a  0)
x= = 2− 3
4 −3 (xii) a  b = ab , a, b  R and a  0, b  0
x – 2 = – 3 squaring both sides; we get
( a + b ) = a + b + 2 ab , a, b  R and ab  0
2
(xiii)
(x – 2)2 = (− 3)2
(xiv) ( a − b ) = a + b − 2 ab , a, b  R and ab  0
2

 x2 + 4 – 4x = 3  x2– 4x + 1 = 0
Now, x3 – x2 – 11x + 4 (xv) a + b = c + d  a = c and b = d where a, c
= x3 – 4x2 + x + 3x2 – 12x + 4 are rational numbers and b , d are irrational
= x (x2 – 4x + 1) + 3(x2 – 4x + 1) + 1
numbers
= x × 0 + 3 (0) + 1 = 0 + 0 = 0 = 1
1 a+ b a+ b
(xvi) = =
Example 1.14 a− b ( a − b )( a + b ) a−b

1 1 a− b a− b
If x = 3 − 2 2 , find x2 + . (xvii) = =
x2 a + b ( a + b )( a − b ) a−b
Solution : We have, x = 3 – 2 2 (xviii) If x = n(n+1), then
1 1 1 3+2 2 (a) x − x − x − ...... = n, n  0
 = = 
x 3−2 2 3−2 2 3+2 2
(b) x + x + x + ...... = n, n  0
3+2 2 3+2 2
= = = 3+2 2 (xix) a0 = 1, a  0
(3)2 − (2 2)2 9−8
9 Chapter 1 | Fundamentals of Mathematics
−3 3
Example 1.16  11   13  2197
=  =  =
Evaluate the following :  13   11  1331
−3
−1
(  121  2
(i) 3
64 ) 2
(ii)   Example 1.17
 169 
−1 If ax = b, by = c, cz = a, prove that xyz = 1 where a, b, c
−1
 
1 2 1 −1 −1
are distinct numbers
Solution : (i) ( 3 64 ) 2

= (64) 3  = (64) 3 2 = (64) 6 Solution : We have,
−1  −1 
6  1 axyz = (ax)yz
= (26 ) 6 = 2  6 
= 2−1 =
2  axyz = (b)yz [ ax = b]  axyz = (by)z
−3
 11  11 
−3 / 2
 112 
−3 / 2
 11 
2
2  axyz = cz [ by = c]  axyz = a [ cz = a]
(ii)   = 2  = 
 13  13   13   13   axyz = a1  xyz = 1

WORKED-OUT PROBLEMS - 1

1. If n2 + 2n – 8 is a prime number where n  N, then (E) {23, 81}


n is Solution : (A) → (p, r) ; (B) → (p, q, r) ; (C) → (p, r) ;
(A) also a prime number (D) → (s) ; (E) → r
(B) relatively prime to 10
(C) relatively prime to 6 6. The multiplication of a rational number 'x' and an
(D) a composite number irrational number 'y' is :
Solution : n2 + 2n – 8 = p  (n + 1)2 = p + 9 (A) always rational
 n  N so p + 9 is a perfect square (B) rational except when y = 
So p can only be 7  n = 3 Ans. (A, B) (C) always irrational
(D) irrational except when x = 0
2. Consider the equation x3 – 3x2 + 2x = 0, then Solution : For if x = 0
(A) Number of even integers satisfying the then xy = 0
equation is 2 which is rational. Ans. (D)
(B) Number of odd integers satisfying the
equation is 1 7. If x, y are rational numbers such that (x + y) + (x
(C) Number of odd prime natural numbers
satisfying the equation is 1 – 2y) 2 = 2x – y + (x – y – 1) 5 then
(D) Number of composite natural numbers (A) x = 1, y = 1
satisfying the equation is 1 (B) x = 2, y = 1
Solution : x  x2 – 3x + 2 = 0 (C) x = 5, y = 1
x(x – 1 )(x – 2) = 0 (D) x & y can take infinitely many values
x = 0, 1, 2 Ans. (A, B) Solution : Equality can hold only when
x + y = 2x – y  x – 2y = 0 ... (1)
4. Consider the following statements and x – 2y = x – y – 1 = 0 ... (ii)
(i) The sum of a rational number with an (i) – (ii) we get –y + 1 = 0
irrational number is always irrational.  y = 1 & x = 2. Ans. (B)
(ii) The product of two rational numbers is always
rational. 8. Six bells start tolling together and they toll at
(iii) The product of two irrationals is always intervals of 2, 4, 6, 8, 1 0, 12 sec. respectively, find
irrationals. (i) after how much time will all six of them toll
(iv) The sum of two rational is always rational. together ?
(v) The sum of two irrationals is always irrational. (ii) how many times will they toll together in 30
The correct order of True/False of above min ?
statements is: Solution : The time after which all six bells will toll
(A) T F T F F (B) F F T T T together must be multiple of 2, 4, 6, 8, 1 0, 12.
(C) T T F T F (D) T T F F T Therefore, required time = LCM of time intervals
Solution : (C) = LCM (2, 4, 6, 8, 1 0, 12) = 120 sec.
Therefore after 120 s all six bells will toll together.
5. After each 120 s, i.e. 2 min, all bell is tolling together.
Column I Column II  30 
Therefore in 30 min they will toll together  + 1 
(A) {2, 3} (p) is a pair of primes  2 
(B) {11,13} (q) is a pair of twin primes = 16 times
(C) {5, 11} (r) is a pair of co-primes 1 is added as all the bells are tolling together at the
(D) {2, 6} (s) is a pair of even number start also, i.e. 0th second.
Chapter 1 | Fundamentals of Mathematics 10
2
9. Find the largest four digit number which when  Ans. (A)
n
reduced by 54, is perfectly divisible by all even
natural numbers less than 20. 14. Solve for x  R –
Solution : Even natural numbers less than 20 are 2, 4, 1. 4x – 10.2x – 1 = 24
6, 8, 1 0, 12, 14, 16, 18. 2. 4.22x – 6x = 18.32x
Their LCM = 2 × LCM of first 9 natural numbers 3. 32x – 3 – 9x – l + 272x/3 = 675.
= 2 × 2520 = 5040.
1
This happens to be the largest four-digit number 4. 7x+2 – .7x+l – 14.7x – l + 2.7x = 48
divisible by all even natural numbers less than 20. 7
x +1 x2 + 2x −11 9

5.   .   5
54 was subtracted from our required number to get 5 9
= 
this number. 3  25  3
Hence, (required number – 54) = 5040  Required 6. 52x = 32x + 2.5x + 2.3x
number= 5094. Solution :
1. 22x – 5.2x – 24 = 0
10. Which digits should come in place of * and $ if the (2x – 8)(2x + 3) = 0
number 62684*$ is divisible by both 8 and 5 ? 2x = 8 (2x  –3  ax > 0)
Solution : Since the given number is divisible by 5, so
0 or 5 must come in place of $. But, a number ending x=3
with 5 in never divisible by 8. So, 0 will replace $.
Now, the number formed by the last three digits is 4*0, 2. Divide by 22x
2x x
which becomes divisible by 8, if * is replaced by 4 or 8. 3 3
Hence, digits in place of * and $ are (4 or 8 or 0) and 0 18   +   − 4 = 0
2 2
respectively.
2x x x
3 3 3
11. 123 × 34 × 52, find the total number of even factors 18   + 9.  − 8   − 4 = 0
2  2  2
of N.
3  3    3 x 
x x
Solution : The factorised form of N is (22 × 31)3 × 34 ×
9    2.   + 1  − 4  2.   + 1  = 0

5 2  2 6 × 3 7 × 5 2. 2  2   2 
Hence, the total number of factors of N is (6 + 1) (7 +
1) (2 + 1) = 7 x 8 x 3 = 168.   3 x   3 x 
 2  + 1 
 9   − 4  = 0
Some of these are odd multiples and some are even.  2   2  
The odd multiples are formed only with the x
combination of 3s and 5s. 3 −1
So, the total number of odd factors is (7 + 1) (2 + 1)   
2 2
= 24. x −2
Therefore, the number of even factors is 168 – 24 3 4 3
   = = 
= 144. 2 9 2
 x = –2
12. How many numbers from 200 to 600 are divisible
by 4, 5, 6?
32x 32x
Solution : Every such number must be divisible by 3. − + 32x = 675
L.C.M. of (4, 5, 6) = 60. 24 9
 600   200   1 3 27 
9x  − +  = 675
 60  −  60  = 10 – 3 = 7.  27 27 27 
Such numbers are 240, 300, 360, 420, 480, 540 and 600. 675  27
9x = = 93
Clearly, there are 7 such numbers. 25
 x=3
13. When simplified, the product
 1  1  1   1 4. 49.7x – 7x – 2.7x + 2.7x = 48
 1 −   1 −   1 −  ....  1 −  equals
 3  4  5   n 48.7x = 48
1 2 7x = 1
(A) (B)  x=0
n n
2(n − 1) 2
(C) (D) 2x2 +4x −22−x −1 −9
n n(n + 1) 3 3
5.   = 
 3−1  4 −1  5−1   n −1  5 5
Solution :     ......   2x + 3x – 23 = –9
2
 3  4  5   n 
2x2 + 3x – 14 = 0
2 3 4 n −1
   ....... 2x2 + 7x – 4x – 14 = 0
3 4 5 n (2x + 7)(x – 2) = 0
11 Chapter 1 | Fundamentals of Mathematics

 x=
−7
,2 =2 ( )
2 + 3 = 8 + 4 3 = ( 2 + 6 )2
2
= 2+ 6 = a + 6
6. 52x – 32x= 2(5x
+ 3x) On comparing a+ b = 2 + 6 = 8.
(5x + 3x)(5x – 3x)=2(5x + 3x)
(5x + 3x)(5x – 3x – 2) = 0 16. Which is greater ?
 5x + 3x  0 (i) 3 3 or 4 5
 only solution 5x – 3x – 2 = 0 (ii) 8
12 or 4
6
5x = 3x + 2 3
(iii) 2 or 3
 x = 1.
3 4
Solution : (i) 3 or 5

15. If 2 ( 3+ )
5 − 13 + 48 = 4 + b where a and (3 3)
12
or ( 4 5)
12
 34 or 53
b are natural number find (a + b).  53 is greater  4 5 is greater
  (ii) 12 or 62
Solution : 2  3 + 5 − ( 12 + 1)2  62 is greater  4 6 is greater
 

( ) ( )
(iii) 23 or 32
=2 3 + 5 − 12 − 1 = 2 3 + 4 − 12 32 is greater  3 3 is greater

=2 ( )
3 + ( 3 − 1)2 = 2 ( 3+ 3 −1 )

LECTURE-2 POLYNOMIAL THEORY

POLYNOMIAL IN ONE VARIABLE the degree of the polynomial 5x6 – 4x2 – 6 is 6.


An algebraic expression of the form Polynomials classified by degree
p(x) = anxn + an –1 xn –1 + an – 2xn – 2 + .... + a1x1 + General
Degree Name Example
a0x0, where form
(i) an  0 Zero
(undefined) 0 0
(ii) power of x is whole number, is called a polynomial
polynomial in one variable. Non-zero)
Hence, an, an–1, an–2, ....., a0 are coefficients of xn, xn– 0 constant a; (a  0) 1
1, ..........., x0 respectively and anxn, an–1xn–1, an–2xn–2, polynomial
...... are terms of the polynomial. Here the term anxn Linear ax + b; (a 
1 x+1
is called the Leading term and its coefficient an, the polynomial 0)
leading coefficient. Quadratic ax2 + bx +
2 x2 + 1
polynomial c; (a  0)
ax3 + bx2 +
Cubic
3 cx + d; (a  x3 + 1
polynomial
0)
Usually, a polynomial of degree n, for n greater
than 3, is called a polynomial of degree n, although
the phrases quartic polynomial and quantic
ZERO POLYNOMIAL polynomial are sometimes used.
3
Constants 2, –2, 2, and a can be written as 2x0, SOME SPECIAL TYPES OF POLYNOMIALS
2 Monomials : Polynomials having only one term
3 are called monomials.
–2x0, 2 x , x0 and ax0 respectively.
0

2 E.g. 2, 2x, 7y5, 12t7etc.


Therefore, these Constants are expressed as Binomials : Polynomials having exactly two
polynomials which contain single term in variable dissimilar terms are called binomials.
x and the exponent of the variable is 0. Thus, we E.g. p(x) = 2x + 1, r(y) = 2y7 + 5y6 etc.
can define a constant as a constant polynomial. Trinomials : Polynomials having exactly three
In particular, the constant number 0 as the zero distinct terms are called trinomials.
polynomial. E.g. p(x) = 2x2 + x + 6, q(y) = 9y6 + 4y2 + 1 etc.

DEGREE OF POLYNOMIALS ZEROS / ROOTS OF A POLYNOMIAL / EQUATION


Degree of the polynomial in one variable is the Consider a polynomial f(x) = 3x2 – 4x + 2. If we
largest exponent of the variable. For example, the replace x by 3 everywhere in the above
degree of the polynomial 3x7 – 4x6 + x + 9 is 7 and
Chapter 1 | Fundamentals of Mathematics 12
expression, we get f(3) = 3 × (3)2 –4×3+2= x − 6 x3 − 7x 2 + 6x + 4(x 2 − x
27 – 12 + 2 = 17
We can say that the value of the polynomial f(x) at x3 − 6x 2
− +
x = 3 is 17.
Similarly, the value of polynomial f(x) = 3x2 – 4x + 2 − x 2 + 6x + 4
at x = – 2 is f(–2) = 3(–2)2 – 4 × (–2) + 2
− x 2 + 6x
= 12 + 8 + 2 = 22 + −
at x = 0 is f(0) = 3(0)2 – 4(0) + 2 = 0 – 0 + 2 = 2 Re mainder = 4
1 Thus, p(a) is remainder on dividing p(x) by (x – a).
at x = is
2
2
1 1 1 3 3 Note
f   = 3   − 4    + 2 = − 2 + 2 =
2 2 2 4 4 1. p(–a) is remainder on dividing p(x) by (x + a)
In general, we can say f() is the value of the [ x + a = 0  x = – a]
polynomial f(x) at x = , where  is are a real
number. A real number  is zero of a polynomial 2. is remainder on dividing p(x) by (ax – b)
f(x) if the value of the polynomial f(x) is zero at x
=  i.e. f() = 0. [ ax – b = 0  x = b/a]
OR
The value of the variable x, for which the 3. is remainder on dividing p(x) by (ax +
polynomial f(x) becomes zero is called zero of the
polynomial. b) [ ax + b = 0  x = –b/a]
E.g. : consider, a polynomial p(x) = x2– 5x + 6;
replace x by 2 and 3. 4. is remainder on dividing p(x) by (b – ax)
p(2) = (2)2 – 5 × 2 + 6 = 4 – 10 + 6 = 0,
p(3) = (3)2 – 5 × 3 + 6 = 9 – 15 + 6 = 0 [ b – ax = 0  x = b/a]
 2 and 3 are the zeros of the polynomial p(x).

ROOTS OF A POLYNOMIAL EQUATION Example 2.1


An expression f(x) = 0 is called a polynomial Find the remainder when x3 – ax2 + 6x – a is divided
equation if f(x) is a polynomial of degree n  1. A by x – a
real number  is a root of a polynomial f(x) = 0 if Solution : Let p(x) = x3 – ax2 + 6x – a
f() = 0 i.e.  is a zero of the polynomial f(x). p(a) = a3 – a(a)2 + 6(a) – a = a3 – a3 + 6a – a = 5a
E.g. consider the polynomial f(x) = 3x – 2, then 3x So, by the Remainder theorem, remainder = 5a
– 2 = 0 is the corresponding polynomial equation.
2  2 Example 2.2
Here, f   = 3  − 2 = 0
3 3 A polynomial in x of degree greater than three, leaves
2 remainders 2, 1 and –1 when divided, respectively, by
i.e. is a zero of the polynomial f(x) = 3x – 2 or (x – 1), (x + 2) and (x + 1). What will be the remainder
3
when it is divided by (x – 1) (x + 2) (x + 1) ?
2
is a root of the polynomial equation 3x – 2 = 0 Solution : Let required polynomial be f(x) = p(x) (x – 1)
3 (x + 2) (x + 1) + a0x2 + a1x + a2
By remainder theorem, f(1) = 2, f(–2) = 1, f(–1) = –1.
REMAINDER THEOREM  a 0 + a 1 + a2 = 2
Statement : Let p(x) be a polynomial of degree  1 4a0 – 2a1 + a2 = 1
and 'a' is any real number. If p(x) is divided by (x a0 – a1 + a2 = –1
– a), then the remainder is p(a). 7 3 2
E.g. Let p(x) be x3 – 7x2 + 6x + 4 Solving we get, a0 = , a1 = , a2 =
6 2 3
Divide p(x) with (x – 6) and to find the remainder,
put x = 6 in p(x) i.e. p(6) will be the remainder. Remainder when f(x) is divided by (x – 1) (x + 2) (x +
 required remainder be 7 3 2
1) will be x 2 + x + .
p(6) = (6)3 – 7.62 + 6.6 + 4 = 216 – 252 + 36 + 4 6 2 3
= 256 – 252 = 4

FACTOR THEOREM
Statement : Let f(x) be a polynomial of degree  1
and a be any real constant such that f(a) = 0, then
(x – a) is a factor of f(x). Conversely, if (x – a) is a
factor of f(x), then f(a) = 0.
13 Chapter 1 | Fundamentals of Mathematics
Proof : By Remainder theorem, if f(x) is divided by Solution : Use a2 – b2 = (a – b)(a + b)
(x – a), the remainder will be f(a). Let q(x) be the (3x – y)2 – (2x – 3y)2 = (3x – y + 2x – 3y) (3x – y – 2x
quotient. Then, we can write, + 3y) = (5x – 4y) (x + 2y)
f(x) = (x – a) × q(x) + f(a)
( Dividend = Divisor × Quotient + Remainder)
If f(a) = 0, then f(x) = (x – a) × q(x) Type -2 : a3 ± b3  (a ± b) (a2 ab + b2 )
Thus, (x – a) is a factor of q(x).
Converse Let (x – a) is a factor of f(x). Example 2.5
Then we have a polynomial q(x) such that f(x)
= (x – a) × q(x) a6 – b6
Replacing x by a, we get f(a) = 0. Hence, proved. Solution : a6 – b6 = (a2)3 – (b2)3
= (a2 – b2)(a4 + a2b2 + b4)
= (a – b)(a + b)(a2 – ab + b2)(a2 + ab + b2)
Example 2.3
Use the factor theorem to determine whether (x – 1)
is a factor of Type -3 : Factorising the quadratic
f (x) = 2 2x3 + 5 2x2 − 7 2
Solution : By using factor theorem, (x – 1) is a factor of Example 2.6
f(x), only when f(1) = 0 x2 + 6x – 187
f (1) = 2 2(1)3 + 5 2(1)2 − 7 2 = 2 2 + 5 2 − 7 2 = 0 Solution : x2 + 6x – 187 = x2 + 17x – 11x – 187
Hence, (x – 1) is a factor of f(x). = x(x + 17) – 11(x + 17) = (x + 17) (x – 11)

FACTORIZATION : Type -4 : Factorisation by converting the given


Formulae : expression into a perfect square.
(a) (a ± b)2 = a2 ± 2ab + b2 = (a b)2 ± 4ab
(b) a2 – b2 = (a + b) (a – b) Example 2.7
1 9x4 – 10x2 + 1
If a2 – b2 = 1 then a + b = Solution : 9x4 – 10x2 + 1 = (9x2 – 1)(x2 – 1 )
a−b
= (3x – 1)(3x + 1)(x – 1)(x + 1)
For example : sec  – tan  =
1 1
or 3 + 2 =
sec  + tan  3− 2 Type -5 : Using factor Theorem
(c) (a + b)3 = a3 + b3 + 3ab(a + b)
(d) (a – b)3 = a3 – b3 – 3ab(a – b) Example 2.8
(e) a3 + b3 = (a + b) (a2 – ab + b2) = (a + b)3 –
3ab(a + b) x3– 13x – 12
(f) a3 – b3 = (a – b) (a2 + ab + b2) = (a – b)3 + Solution : x3 – 13x – 12  x = –1 satisfies given
3ab(a – b) expression  x + 1 is a factor
(g) (a + b + c)2 = a2 + b2 + c2 + 2ab + 2bc + 2ca
(h) a3 + b3 + c3 – 3abc = (a + b + c) (a2 + b2 + c2 x + 1 x3 − 13x − 12( x 2 − x − 12
1 x3 + x 2
– ab – bc – ca) = (a + b + c){(a – b)2 + (b
2
– c)2 + (c – a)2} − x2 − 13x − 12
(i) (a + b + c)3 = a3 + b3 + c3 + 3(a + b) (b + c) − x2 − x
(c + a)
− 12x − 12
(j) a4 + a2 + 1 = (a2 + 1)2 – a2 = (1 + a + a2)(1 –
a + a 2) − 12x − 12
0
CYCLIC FACTORS :
If an expression remain same after replacing a by  x3 – 13x – 12 = (x + 1) (x2 – x – 12)
b, b by c & c by a, then it is called cyclic expression = (x + 1)(x – 4)(x + 3)
and its factors are called cyclic factors.
e.g. a(b – c) + b(c – a) + c(a– b)
Type -6 : a3 + b3 + c3 – 3abc = (a + b + c) (a2 + b2 + c2
Factorization : – ab – bc – ac)
Type-1 : a2 – b2 = (a – b)(a + b)
Example 2.9
Example 2.4 8x3 + y3 + 27z3 – 18xyz
(3x – y)2– (2x – 3y)2
Chapter 1 | Fundamentals of Mathematics 14
Solution : + 8x3
+ y3 27z3
– 18xyz = + (2x)3 (y)3 + Solution : x(x + 1) (x + 2) (x + 3) – 8 = x(x + 1) (x +
(3z)3 – 3(2x)(y)(3z) 2) (x + 3) – 8
= (2x + y + 3z)(4x2 + y2 + 9z2 – 2xy – 6xz – 3yz) = x(x + 3) (x + 1) (x + 2) – 8 = (x2 + 3x) (x2 + 3x +
2) – 8
= (x2 + 3x)2 + 2(x2 + 3x) – 8 = (x2 + 3x)2 + 4(x2 + 3x)
Type-7 : – 2(x2 + 3x) – 8
= (x2 + 3x)(x2 + 3x + 4) – 2(x2 + 3x + 4) = (x2 + 3x –
Example 2.10 2)(x2 + 3x + 4)
x(x + 1) (x + 2) (x + 3) – 8

WORKED-OUT PROBLEMS - 2

1 = (2x – 5y)(4x2 + 25y2 + 10xy) + (2x – 5y)


1. If x = 3 − 8 , then find the value of − x. = (2x – 5y)(4x2 + 25y2 + 10xy + 1)
x
Solution : x = 3 − 8 = 3 − 2 2 5. Factorize following expressions
x = ( 2 ) − 2 21 + (1)
2
2 (i) x2 + 3x – 40 (ii) x2 – 3x – 40
(iii) x + 5x – 14
2 (iv) x2 – 3x – 4
x = ( 2 − 1)
2
(v) x – 2x – 3
2 (vi) 3x2 – 10x + 8
(vii) 12x + x – 35
2 (viii) 3x2 – 5x + 2
x = 2 −1
(ix) 3x2 – 7x + 4 (x) 7x2 – 8x + 1
1 1 1 2 +1 (xi) 2x2 – 17x + 26 (xii) 3a2 – 7a – 6
= = 
x 2 −1 2 −1 2 +1 (xiii) 14a2 + a – 3
1 Solution : (i) x2 + 3x – 40 = (x + 8)(x – 5)
= 2 +1 (ii) x2 – 3x – 40 = (x – 8)(x + 5)
x
(iii) x2 + 5x – 14 = (x + 7)(x – 2)
− x  ( 2 + 1) − ( 2 − 1)
1
(iv) x2 – 3x – 4 = (x – 4)(x + 1)
x (v) x2 – 2x – 3 = (x – 3)(x + 1)
 2 +1− 2 +1 (vi) 3x2 – 10x + 8 = 3x2 – 6x – 4x + 8 = (3x – 4)(x – 2)
1 (vii) 12x2 + x – 35 = 12x2 + 21x – 20x – 35
− x =2.
x = 3x(4x + 7) – 5(4x + 7)
= (4x + 7)(3x – 5)
a3 + b3 + c3 − 3abc (viii) 3x2 – 5x + 2 = 3x2 – 3x – 2x + 2
2. The value of is (where a = 3x(x – 1) – 2(x – 1)
ab + bc + ca − a2 − b2 − c2
= –5, b = – 6, c = 10) = (x – 1)(3x – 2)
Solution : a3 + b3 + c3 – 3abc = (a + b + c)(a2 + b2 + (ix) 3x2 – 7x + 4 = 3x2 – 3x – 4x + 4
c2 – ab – bc – ca) = 3x(x – 1) – 4(x – 1)
a3 + b3 + c3 − 3abc = (3x – 4)(x – 1)
=
−(a 2 + b2 + c2 − ab− bc− ca) (x) 7x – 8x + 1 = 7x2 – 7x – x + 1
2

= 7x(x – 1) – 1(x – 1) = (x – 1)(7x – 1)


a3 + b3 + c3 − 3abc (xi) 2x2 – 17x + 26 = 2x2 – 4x – 13x + 26
=
−(a3 + b3 + c3 − 3 abc) = 2x(x – 2) – 13(x – 2) = (x – 2)(2x – 13)
(a + b+ c) (xii) 3a – 7a – 6 = 3a2 – 9a + 2a – 6
2

= –(a + b + c) = –(–5 – 6 + 10) = 1 = 3a(a – 3) + 2(a – 3) = (a – 3)(3a + 2)


(xiii) 14a2 + a – 3 = 14a2 + 7a – 6a – 3
3. Factorize following expressions = (7a – 3)(2a + 1)
(i) x4 – y4 (ii) 9a2 – (2x – y)2
(iii) 4x2 – 9y2 – 6x – 9y 6. Factorize following expressions
Solution : (i) x4 – y4 = (x2 – y2)(x2 + y2) (i) a2 – 4a + 3 + 2b – b2 (ii) x4 + 324
= (x + y)(x – y)(x2 – y2) (iii) x4 – y2 + 2x2 + 1 (iv) 4a4 – 5a2 + 1
(ii) 9a2 – (2x – y)2 = (3a + 2x – y)(3a – 2x + y) (v) 4x4 + 81
(iii) 4x2 – 9y2 – 6x – 9y = (2x + 3y)(2x – 3y) – 3(2x + 3y) Solution : (i) a2 – 4a + 3 +2b – b2 = (a – 2)2 – (b – 1)2
= (2x + 3y)(2x – 3y – 3)
= (a – 2 + b – 1)(a – 2 – b + 1)
4. Factorize following expressions = (a + b – 3)(a – b – 1)
(i) 8x3 – 27y3 (ii) x4 + 324
(ii) 8x3 – 125y3 + 2x – 5y = x4 + 182 = (x2)2 + 2 × 18x2 – 2 × 18x2 + 182
Solution : (i) 8x3 – 27y3 = (2x)3 – (3y)3 = (x2 + 18)2 – (6x)2
= (2x – 3y)(4x2 + 9y2 + 6xy) = (x2 + 18 + 6x)(x2 + 18 – 6x)
(ii) 8x3 – 125y3 + 2x – 5y (iii) x4 – y2 + 2x2 + 1
15 Chapter 1 | Fundamentals of Mathematics
= (x2 + 1)2 – y2 = (x2 + y + 1)(x2 + 1 – y) = (4x2 + 4x – 9)(4x2 + 4x + 6)
(iv) 4a4 – 4a2 – a2 + 1 = 2(2x2 + 4x + 3)(4x2 + 4x – 9)
= 4a4 – 4a2 – a2 + 1
= 4a2(a2 – 1) – 1(a2 – 1) (iii) (x – 3)(x + 2)(x + 3)(x + 8) + 56
= (4a2 – 1)(a2 – 1) = (x2 + 5x – 24)(x2 + 5x + 6) + 56
= (2a + 1)(2a – 1)(a + 1)(a – 1) = (t – 30)(t) + 56 (let x2 + 5x + 6 = t)
(v) 4x4 + 81 = t – 28t – 2t + 56
2
= (2x2)2 + 36x2 + 81 – 36x2 = t(t – 28) – 2(t – 28)
= (2x2 + 9)2 – (6x)2 = (t – 28)(t – 2)
= (2x2 + 6x + 9)(2x2 – 6x + 9) = (x2 + 5x – 22)(x2 + 5x + 4)
= (x2 + 5x – 22)(x + 4)(x + 1)
7. Factorize following expressions
(i) x3 – 6x2 + 11x – 6 (ii) 2x3 + 9x2 + 10x + 3 10. The cubic polynomial P(x) satisfies the condition
(iii) 2x – 9x + 13x – 6 (iv) x6 – 7x2 – 6
3 2
that (x – 1)2 is a factor of P(x) + 2, and (x + 1)2 is
Solution : (i) x3 – 6x2 + 11x – 6 = (x – 1)(x2 – 5x + 6) a factor of P(x) – 2. Then P(3) equals.
= (x – 1)(x – 3)(x – 2) (A) 27 (B*) 18 (C) 12 (D) 6
(ii) 2x3 + 9x2 + 10x + 3 = (x + 1)(2x2 + 7x + 3) Solution : P(x) + 2 =(ax+ b) (x – 1)2 ……(i)
= (x + 1)(2x2 + 6x + x + 3) P(x) – 2 =(cx+ d) (x + 1) ……(ii)
= (x + 1)(2x + 1)(x + 3) equating P(x) from (i) and (ii) and comparing
(iii) 2x3 – 9x2 + 13x – 6 = (x – 1)(2x2 – 7x + 6) coefficients of all powers of 'x'
= (x – 1)(2x2 – 4x – 3x + 6) (ax+ b) (x2 – 2x + 1) – 2 =(cx+ d) (x2 + 2x + 1) + 2
= (x – 1)(2x – 3)(x – 2) coeff. of x3 : a = c
(iv) x6 – 7x2 – 6 = (x2 + 1)(x4 – x2 – 6) coeff. of x2 : –2a + b = 2c + d ……(ii)
= (x2 + 1)(x2 – 3)(x2 + 2) coeff. of x : a – 2b = 2d + c ….... (iii)
const : b – 2 = d + 2  b = d + 4 ….... (iv)
= (x2 + 1)(x2 + 2) (x+ 3)(x− 3)
by (iii) and (iv) – 2b = 2d ( a= c)
8. (i) Factorize the expressions 8a6 + 5a3 + 1 and, b = 2, d = –2 and by (i), a = c = 1
(ii) Show that (x – y)3 + (y – z)3 + (z – x)3 = 3(x – y)  P(x) = (x + 2)(x2 – 2x + 1) – 2 ……... (i)
(y – z) (z – x). So, P(3) = 5(9 – 6 + 1) – 2 = 18
Solution : (i) 8a6 + 5a3 + 1
= 8a6 + 6a3 – a3 + 1 11. Show that (2x + 1) is a factor of the expression
= (2a2)3 + (–a)3 + 13 – 3 × 2a2x(–a) × 1 f(x) = 32x5 – 16x4 + 8x3 + 4x + 5.
= (2a2 – a + 1)(4a4 + a2 + 1 + 2a3 + a – 2a2)  1
Solution : Since (2x + 1) is to be a factor of f(x), f  − 
= (2a2 – a + 1)(4a4 + 2a3 – a2 + a + 1)  2
(ii) Let A = x – y, B = y – z, C = z – x should be zero.
5 4 3
A+B+C=0  1  1  1  1  1
f  −  = 32  −  − 16  −  + 8  −  + 4  −  + 5 .
then A3 + B3 + C3 = 3ABC  2  2  2  2  2
(x – y)3 + (y – z)3 + (z – x)3 Hence (2x + 1) is a factor of f(x).
= 3(x – y)(y – z)(z – x)
12. Using factor theorem, show that a – b, b – c and c – a
9. Factorize following expressions are the factors of a(b2 – c2) + b(c2 – a2) + c (a2 – b2).
(i) (x + 1) (x + 2) (x + 3) (x + 4) – 15 Solution : By factor theorem, a – b will be a factor of the
(ii) 4x(2x + 3) (2x – 1) (x + 1) – 54 given expression if it vanishes by substituting a = b in it.
(iii) (x – 3) (x + 2) (x + 3) (x + 8) + 56 substituting a = b in the given expression, we have
a(b2 – c2) + b(c2 – a2 ) + c(a2 – b2)
Solution : (i) (x + 1)(x + 2)(x + 3)(x + 4) − 15 = b(b2 – c2) + b(c2 – b2) + c(b2 – b2)
= (x2 + 5x + 4)(x2 + 5x + 6) – 15 = b3 – bc2 + bc2 – b3 + c(b2 – b2) = 0
(let x2 + 5x = t)  (a – b) is a factor of a(b2 – c2) + b(c2 – a2) + c(a2 – b2).
Similarly, we can show that (b – c) and (c – a) are also
= (t + 4)(t + 6) – 15
factors of the given expression.
= t2 + 10t + 9 = (t + 9)(t + 1)
Hence, (a – b), (b – c) and (c – a) are factors of the
= (x2 + 5x + 9)(x2 + 5x + 1)
given expression.
(ii) 4x (2x + 3) (2x – 1)( x + 1) – 54
13. If f(x) = x4 – 2x3 + 3x2 – ax + b is a polynomial
such that when it is divided by (x – 1) and (x + 1)
= (4x2 + 4x)(4x2 + 4x – 3) – 54 the remainders are 5 and 19 respectively. If f(x) is
= t(t – 3) – 54 (let 4x2 + 4x = t) divided by (x – 2), then remainder is :
= t2 – 3t – 54 (A) 0 (B) 5 (C) 10 (D) 2
= (t – 9)(t + 6) Solution : P(1) = 5  2 – a + b = 5  b – a = 3 ......(i)
Chapter 1 | Fundamentals of Mathematics 16
P(–1) = 6 + a + b = 19  b + a = 13 .......(ii) 4 4
3 6  6 3 
solving (i) and (ii) 14. The expression  a9   a9  is simplified
b = 8, a = 5 to
so remainder = P(2) = 10 (A) a16 (B) a12 (C) a8 (D) a4
Solution : a9/6 . 4/3 .a 9/3 . 4/6 =a 4

LECTURE-3 RATIO AND PROPORTION

Ratio: (a) a : b > c : d if ad >bc


A ratio is a comparison of two numbers (b) a : b < c : d if ad <bc
(quantities) by division. The ratio of a to b is (c) a : b = c : d if ad = bc
a (d) Ratios can also be compared by expressing
written as a : b or . In the ratio a : b, a and b are the ratios as fractions and then converting
b
them to decimal numbers.
called terms of the ratio. 'a' is the antecedent and
(e) They can also be compared by converting
'b' is the consequent.
them to their equivalent fraction of common
denominator.
Properties of ratio
(a) In a ratio, two quantities are compared. So,
Ratio of greater inequality or lesser
the quantities must be of the same kind i.e.
they must be expressed in the same units. inequality
(b) The order of the terms in a ratio a : b is very (a) A ratio a : b is called a ratio of greater
important, since 2 : 3 is not same as 3 : 2. inequality.
(c) The value of a ratio remains unaltered· if the If a > b e.g. 5 : 2 is a ratio of greater inequality.
given ratio is multiplied or divided by the A ratio of greater inequality is decreased if a
same non–zero quantity. if a, b and m are same positive number is added to both the
non–zero real numbers then a : b = ma : mb terms of the ratio and increased if a same
and a : b = a/m : b/m. positive number is subtracted from both of its
(d) If the ratio of any two quantities can be terms.
expressed exactly by the ratio of two integers, (b) A ratio a : b is called ratio of less inequality if
the quantities are said to be commensurable a < b. A ratio of less inequality is decreased if
otherwise incommensurable. a same positive number is subtracted from
both the terms of the ratio and increased if a
same positive quantity is added to both of its
To divide a number into parts in a given
terms.
ratio
Let 'X' be the given number, the given ratio is x1 ∶ x2 .
More terms related to ratios:
X is to be divided into two parts such that the
(a) Compounded Ratio : The compounded ratio of
Value of first part : Value of second part = x1 : x2
a : b and c : d is ac : bd.
Therefore
(b) Duplicate Ratio : The duplicate ratio of a : b is
 x1  a2 : b2
First part =   × X
 x1 + x 2  (c) Triplicate Ratio : The triplicate ratio of a : b is
a3 : b3
 x2  (d) Sub–duplicate Ratio : The sub–duplicate ratio
Second part =   × X
 x1 + x 2  of a : b is a : b
Compare the ratio 3 : 4 and 1 : 2 (e) Sub–triplicate Ratio : The sub–triplicate ratio
3 3 1 3 of a : b is 3 a : 3 b
=  =
4 4 1 4 (f) Inverse Ratio: The inverse ratio or reciprocal
1 1 2 2 1 1
=  = ratio of a : b is : i.e., b : a ratio × It’s
2 2 2 4 a b
3 2 Inverse Ratio = 1.
3 > 2  i.e. 3 : 4 > 1 : 2
4 4
Proportion:
Comparison of ratio Equality of two ratios is called proportion.
Let a : b and c: d are two given ratios, then If a : b = c : d, then a, b, c and d are in proportion, a
: b = c : d is also represented as a : b :: c :d.

(1) Each quantity a, b, c and d of a proportion a :


b :: c : d is called a term or proportional. a, b, c
17 Chapter 1 | Fundamentals of Mathematics
and d are respectively called first, second, (13) a:b = c:d = e:f = (ka + lc + me)/ (kb +
third and fourth term or proportional of the ld + mf). It follows that if all the ratios are
proportion a: b :: c: d. finite and denominator kb + ld + mf = 0
then it implies ka + lc + me = 0
(2) The first and the fourth (last) terms are called
extremes and the second and third terms are
called middle terms or means. Example 3.1
If a : b = 5 : 9 and b : c = 4 : 7, find a : b : c. (Combining
(3) In a proportional a : b :: c : d product of Ratios)
extremes = product of means. Solution : a : b = 5 : 9 and b : c = 4 : 7
 9  9 63
(4) In a proportion, fourth term is called fourth =  4  : 7  = 9 :
 4   4  4
proportional of the other three terms.
63
 a:b:c=5:9: = 20 : 36 : 63.
a c b d 4
(5) If a : b :: c : d then b : a :: d : c or =  =
b d a c Example 3.2
This is known as Invertendo. Find out:
(i) the fourth proportional to 4, 9, 12.
a c a b (ii) the third proportional to 16 and 36.
=  =
(6) If a : b :: c : d then a : c :: b : d or
b d c d (iii) the mean proportional between 0.08 and 0.18.
This result is known as Alternendo. Solution : (i) Let the fourth proportional to 4, 9, 12 be x.
Then, 4 : 9 : : 12 : x  4 × x = 9 × 12
a c a+b c+d 9  12
(7) If a : b :: c : d  +1 = +1  = x= = 27
b d b d 4
or a : b :: c : d  (a + b) : b :: (c + d) : d...... (1)  Fourth proportional to 4, 9, 12 is 27.
This result is known as Componendo. (ii) Let the third proportional to 16 and 36 is x.
Then, 16 : 36 : : 36 : x  16 × x = 36 × 36
a c a −b c−d 36  36
(8) If a : b :: c : d  −1 = −1 = or x= = 81
b d b d 16
a : b :: c : d  (a – b) : b :: (c – d) : d ...... (2)  Third proportional to 16 and 36 is 81.
This result is known as Dividendo. (iii) Mean proportional between 0.08 and 0.18
8 18
(9) Dividing equation (1) and (2) we get = 0.08  0.18 = 
100 100
a+b c+d
b = d  (a + b):(a − b)::(c + d):(c − d) 144 12
= = = 0.12
a −b c −d 100  100 100
b d
or a : b :: c : d  This result is known as Example 3.3
Componendo and Dividendo. The monthly incomes of two persons are in the ratio
4 : 5. The monthly expenditure of them are in the
(10) a, b, c, d are said to be in continued ratio 7 : 9. If each saves Rs. 1000 per month find
a b c their income.
proportion If = = Solution : The incomes are in the ratio 4 : 5.
b c d
 Their incomes are 4K and 5K (say)
(11) if a, b, c are in continued proportion then Their expenditures are 4K – 1000 and 5k – 1000.
a b 4K − 1000 7
=  b2 = ac  b =  ac , b is called Given =
b c 5K − 1000 9
mean proportional of 'a' and 'c' and its value  36K – 9000 = 35K – 7000  K = 2000.
Their incomes are Rs. 8,000 and Rs. 10, 000.
is ac . (b > 0) The first is to the third is the
duplicate ratio of first to the second.
Example 3.4
a b a a b a a a2
= =  =  =
b c c b c b b b2 a c e a3b + 2c2e − 3ae2f ace
If = = , show that =
b d f b4 + 2d2f − 3bf 3 bdf
(12) If a : b = c : d and e : f = g : h, then ae : bf = cg a c e
: dh Solution : Let = = = k;
b d f
If a : b = c : d , b : x = d : y,
then a = bk, c = dk, e = fk;
then a : x = c : y,
Chapter 1 | Fundamentals of Mathematics 18
a3b + 2c2e − 3ae2f b4 k 3 + 2d2fk 3 − 3bf 3k 3 x + y + z x(y + z) + y(z + x) + z(x + y)
 = =
b + 2d f − 3bf
4 2 3
b + 2d f − 3bf
4 2 3 a+b+c 2ax + 2by + 2cz
a c e ace
k3 =   =
b d f bdf Example 3.7
3
m +1 + 3 m −1
Example 3.5 If x =
3
m +1 − 3 m −1
x y z x +a y +b z +c
2 2 2 2 2 2
Solve that x3 – 3x2 + 3x – m = 0.
If = = ; prove that + +
a b c x+a y +b z+c x 3 m +1 + 3 m −1
Solution : We have =
(x + y + z) + (a + b + c)
2 2
1 3 m +1 − 3 m −1
=
x +y +z+a +b+c By componedo and dividend we get
x y z
= = = k, so that x = ak, y = bk,
x + 1 2 3 m + 1 (x + 1)3 m + 1
Solution : Let =  =
a b c x − 1 2 3 m − 1 (x − 1)3 m − 1
z = ck; Again by componendo and dividend, we get
x2 + a2 a2k2 + a2 (k2 + 1)a (x + 1)3 + (x − 1)3 m +1 + m −1 m
then = = = =
x+a ak + a k +1 m + 1 − (m − 1) 1
(x + 1) − (x − 1)
3 3
x2 + a2 y2 + b2 z2 + c2
 + + x3 + 3x2 + 3x + 1 + x3 − 3x2 + 3x − 1 m
x+a y +b z+c  =
x + 3x + 3x + 1 − (x − 3x + 3x − 1)
3 2 3 2 1
(k 2 + 1)a (k 2 + 1)b (k 2 + 1)c
= + +
k +1 k +1 k +1 2x3 + 6x x(x2 + 3)
 =m  =m
(k 2 + 1)(a + b + c) 6x2 + 2 (3x2 + 1)
=
k +1 x3 + 3x = 3mx2 + m  x3 – 3mx2 + 3x – m = 0
(a+ b+ c)
[Multiply and Divided by ]
(a+ b+ c)
Example 3.8
k 2 (a + b + c)2 + (a + b + c)2
=
k(a + b + c) + a + b + c x2 + y 2 xy x p
If = . Prove that = .
(ka + kb + kc)2 + (a + b + c)2 p +q2 2 pq y q
= Solution : From the given we get
(ka + kb + kc) + a + b + c
x2 + y 2 p2 + q2
(x + y + z)2 + (a + b + c)2 =
= xy pq
x +y +z+a +b+c
x2 + y 2 p2 + q2
 =
Example 3.6 2xy 2pq
By Componendo and dividend
x y z
If = = , x2 + y 2 + 2xy p2 + q2 + 2pq
b+c−a c+a −b a +b−c =
x + y + z x(y + z) + y(z + x) + z(x + y) x2 + y 2 − 2xy p2 + q2 − 2pq
prove that =
a +b+c 2(ax + by + cz) (x + y)2 (p + q)2
 =
Solution : Each of the given fractions (x − y)2 (p − q)2
sumof numerators x+y +z x+y p+q
= = ...... (i)  =
sumof denominators a + b + c x −y p−q
Again, if we multiply both numerator and (One of the possibilities out of positive and negative)
denominator of the three given fractions by y + z, z + again by componendo and 18ividend
x, x + y respectively, each fraction x+y +x−y p+q+p−q
x ( y + z) y (z + x ) z(x + y ) =
= = x + y − (x − y) p + q − (p − q)
(y + z)(b + c − a) (z + x ) (c + a − b) ( x + y ) (a + b − c)
2x 2p x p
sumof numerators  =  = .
= 2y 2q y q
sumof denominators
x(y + z) + y(z + x) + z(x + y) Example 3.9
= ......(ii)
2ax + 2by + 2cz
 from (i) and (ii).
19 Chapter 1 | Fundamentals of Mathematics
A person distributes his pens among four friends A, B, = 20 : 15 : 12 : 10
1 1 1 1 Let number of pens given to A, B, C, D are 20x, 15x, 12x,
C and D in the ratio : : : . What is the minimum 10x
3 4 5 6
number of pens that the person should have? So, minimum number of pens (for x = 1)
= 20 + 15 + 12 + 10 = 57
1 1 1 1
Solution : Ratio of pens = A: B: C: D = : : :
3 4 5 6

WORKED-OUT PROBLEMS - 3

1. Find a third proportional to the numbers 3,8 x+y y +z z+x


Solution : Let x be the third proportional then 7. If = = , then f1nd x : y : z.
2 3 4
3:8::8:x
3x = 8 × 8 Sum of the numerators
Solution : Each =
64 Sum of the denominators
x= 2(x + y + z) x + y + z
3 = =
9 9/2
2. Find the mean proportional between 9 and 16. (x + y + z) − (y + z)
Solution : Let x be the third proportional then and therefore each =
9
9 : x : : x : 16 −3
x2 = 16 × 9 2
x = 12 (x + y + z) − (x + z) (x + y + z) − (x + y)
= =
9 9
−4 −2
3. Find a third proportional to the numbers 2.5, 1.5. 2 2
Solution : Let x be the third proportional, then x y z
2.5 1.5 = = =  x : y : z =3 : 1 : 5
2.5 : 1.5 :: 1.5 : x Or = 3/2 1/2 5/2
1.5 x
1.5  1.5 8. If a(y + z) = b(z + x) = c(x + y), then show that
 x= = 0.9
2.5 a−b b−c c −a
= = .
x −y
2 2
y −z
2 2
z − x2
2
4. Find the mean proportional between 48 and 12.
Solution : Given condition can be written as
Solution : Let x be the mean proportional. Then,
y +z z+x z
48 x = = =k …..(1)
48 : x : : x : 12 or = or x2 = 576 or x = 24. 1/a 1/b 1/c
x 12
(z+ x) − (y + z) (x + y) − (x + z)
Each= =
5. The sum of two numbers is c and their quotient is 1 1 1 1
− −
p b a c b
. Find the numbers.
q (y + z) − (x + y) x − y y − z z − x
= = = = .... (2)
Solution : Let the numbers be x, y. 1 1 a −b b−c c−a
Given x + y = c ... (1) −
a c ab bc ca
x p Form (1) and (2), we get by multiplying
and = ... (2)
y q
x 2 − y 2 y 2 − z2 z 2 − x 2
x p x p = =
 =  = [Using (1)] a−b b−c c−a
x+y p+q c p+q abc abc abc
pc qc
 x= ,y = x2 − y2 y2 − z2 z2 − x2
p+q p+q  = =
a−b b−c c−a
6. The sum of two numbers is 10 and their quotient a−b b−c c−a
 = =
2 x −y
2 2
y −z
2 2
z − x2
2
is . Find the numbers.
3
Solution : x + y =10 2a + 3b + 2a − 3b
9. If x = , show that 3bx2 – 4ax +
x 2 2 2a + 3b − 2a − 3b
= x= y
y 3 3 3b = 0.
2 5y
y + y = 10  = 10
3 3
y=6 x=4
Chapter 1 | Fundamentals of Mathematics 20
x a c e
Solution : Taking the left hand side as , using Solution : = = = k  a = bk, c = dk, e = fk
1 b d f
x +1 2a + 3b a3b + 2c2e − 3ae2f k3(b4 + 2d2f − dbf 3 )
componendo and dividendo, =  =
x −1 2a − 3b b4 + 2d2f − 3bf 3 b4 + 2d2f − 3bf 3
(x + 1)2 2a + 3b ace
Squaring, = and again applying = k3 =
2a − 3b bdf
(x − 1)
2

x2 + 1 2a 13. Two positive numbers are in the ratio of 4 : 5. If


componendo and dividendo = which gives
2x 3b the difference between these numbers is 24, then
the answer on cross multiplication. find the numbers.
Solution : Here a= 4, b = 5 and x = 24.
2y + 2z − x 2z + 2x − y 2x + 2y − z ax 4  24
10. If = = , then  The first number = = = 96
a b c b−a 5−4
9x 9y 9z bx 5  24
show that = = and the second number = = = 120 .
2b + 2c − a 2c + 2a − b 2a + 2b − c b−a 5−4
2y + 2z − x 2z + 2x − y 2x + 2y − z
Solution : Since = = , 14. The value of x satisfying the equation
a b c
6x + 2a + 3b + c 2x + 6a + b + 3c
each is equal to = is
6x + 2a − 3b − c 2x + 6a − b − 3c
2(2z+ 2x − y) + 2(2x + 2y − z) − (2y + 2z − x)
(A) ab/c (B) 2ab/c (C) ab/3c (D) ab/2c
2b + 2c − a
6x + 2a + 3b + c 2x + 6a + b + 3c
9x Solution : =
by a theorem quoted earlier = on 6x + 2a − 3b − c 2x + 6a − b − 3c
2b + 2c − a Applying C and D
simplification. 6x + 2a + 3b + c + 6x + 2a – 3b – c
Similarly, each =
9y
2c + 2a − b
and
9z
2a + 2b − c
and (6x + 2a + 3b + c ) – (6x + 2a – 3b – c )
hence the result.
=
(2x + 6a + b –3c ) + (2x + 6a – b –3c )
2+ x + 2− x
(2x + 6a + b + 3c ) – (2x + 6a – b –3c )
11. Solve : =2 12x + 4a 4x + 12a (3x+ a) x + 3a
2+ x − 2− x =  =
6b + 2c 2(b+ 3c) 3b + c b + 3c
2
Solution : Writing the R.H.S. as and using 3xb + ab + 9xc + 3ac = 3xb + 9ab + cx + 3ac
1
8xc = 8ab  x = ab/c Ans. (A)
componendo and dividendo,
( 2+ x + 2− x)+( 2+ x − 2− x) 2+1 x y 2xy
= 15. Simplify : − − 2
( 2+ x + 2− x)−( 2+ x − 2− x) 2−1 x − y x + y x − y2
2+ x 3 x y 2xy
(i.e.) = Solution : − −
x − y x + y x2 − y 2
2− x 1
x (x + y) − y(x − y) 2xy
2+ x 9 = −
Squaring, = and again applying componendo x −y
2 2
x − y2
2
2− x 1
4 10 8 x2 + xy− x y+ y2 2xy
and dividendo = and hence x = = −
2x 8 5 x −y
2 2
x − y2
2

x2 + y2 2xy
= −
a c e a b + 2c e − 3ae f
3 2 2
x −y
2 2
x − y2
2
12. If = = , then show that
b d f b4 + 2d2f − 3bf 3 x2 + y2 − 2xy (x − y )2 (x − y)(x − y ) x − y
ace = = = =
= (wherever defined) x −y
2 2
x −y
2 2
(x − y)(x + y) x + y
bdf

LECTURE-4 COMPLEX NUMBER

DEFINITION : It is denoted by z i.e. z =a + ib. 'a' is called real part


Complex numbers are defined as expressions of of z (Re z) and 'b' is called imaginary part of z (I m
z).
the form a + ib where a, b  R & i = −1 .
21 Chapter 1 | Fundamentals of Mathematics
Every Complex Number Can Be Regarded As Example 4.2
i592 + i590 + i588 + i586 + i584
Find the value of +2.
Purely real Purely imaginary Imaginary i582 + i580 + i578 + i576 + i574
if b = 0 if a= 0 if b  0 i592 + i590 + i588 + i586 + i584 i584
Solution : +2= +2
Note i582 + i580 + i578 + i576 + i574 i574
1. The set R of real numbers is a proper subset of = i + 2 = –1 + 2 = 1
10

the Complex Numbers. Hence the Complex


Number system is N  W  I  Q  R  C. EQUALITY OF COMPLEX NUMBER :
2. Zero is both purely real as well as purely Let there be two complex number z1 = x1 +iy1
imaginary but not imaginary. and z2 = x2 + iy2.
3. is called the imaginary unit. Also i2 = – If z1 = z2 then Re(z1) = Re(z2) and Im(z1) =
1; = –i; i4 = 1 etc.
i3 Im(z2).
In general i4n = 1 i4n + l = –1, i4n + 2 = –I; i4n + 3 = i.e. if x1 + x2 and y1 = y2
–i where n  I.  x1 = x2 and y1 = y2 simultaneously.

4. only if at least one of either a or-b


Example 4.3
is non- negative
Find the real values of x and y for which the equation
(x4 + 2xi) – (3x2 + yi) = (3 – 5i) + (1 + 2yi) is satisfied.
Example 4.1 Solution : Given equation (x4 + 2xi) – (3x2 + yi)
= (3 – 5i) + (1 + 2yi)
The value of i57 + 1/i125 is :-
(A) 0 (B) –2i (C) 2i (D) 2
 (x4 – 3x2) + i(2x – 3y) = 4 – 5i
Equating real and imaginary parts, we get
1 x4 – 3x2 = 4 ….(i)
Solution : i57 + 1/i125 = i56.i +
124
i .i and 2x – 3y = –5 ….(ii)

= ( i4 ) i +
14 1 1
From (i) and (ii), we get x = ±2 and y = 3,
( i4 ) i 3
31

1 i
=i+ =i+ =i−i=0 Ans. (A)
i i2 Note
Inequality in complex numbers are never talked. If x1
+ iy1 > x2 + iy2 has to be meaningful  y1 = y2 = 0.
ARGAND DIAGRAM Equalities hoever in complex numbers are
Master Argand had done a systematic study on meaningful. Two complex numbers z1 and z2 are
complex numbers and represented every complex said to be equal if
number z = x + iy as a set of ordered pair (x, y) on Re z1 = Re z2 and Im(z1) = Im(z2)
a plane called complex plane (Argand Diagram) (i.e. they occupy the same position on complex
containing two perpendicular axes. Horizontal plane)
axis is known as Real axis & vertical axis is known
as Imaginary axis. ADDITION :
All complex numbers lying on the real axis are z1 + z2 = (x1 + iy1) + (x2 + iy2) = (x1 + x2) + i(y1
called as purely real and those lying on imaginary + y2)  C.
axis as purely imaginary. It is easy to observe that the sum of two complex
numbers is a complex number whose real
(imaginary) part is the sum of the real
(imaginary) parts of the given numbers :
Re(z1 + z2) = Re(z1) + Re(z2);
Im(z1 + z2) = Im(z1) + Im(z2)

SUBTRACTION :
z1 – z2 = (x1 + iy1) – (x2 + iy2) = (x1 – x2) + i(y1 – y2)
INTEGRAL POWERS OF I :
 C.
We have i = −1 so i2 = – 1, i3 = –i, i4 = 1 That
or i4n+1 = i, i4n+2 = –1 for any n  I Re(z1 – z2) = Re(z1) – Re(z2);
i4n+3 = –i, i4n = 1 Im(z1 – z2) = Im(z1) – Im(z2).
Thus any integral power of i can be expressed as
1 or  i. MULTIPLICATION :
z1  z2 = (x1 + iy1) (x2 + iy2) = (x1x2 – y1y2) + i(x1y2
+ x2y1)  C.
Chapter 1 | Fundamentals of Mathematics 22
In other words  5+3 5−3 
Re(z1z2) = Re(z1)  Re(z2) – Im(z1)Im(z2) Using formula 3 + 4i =   +i  = ±(2 + i)
and Im(z1z2) = Im(z1)  Re(z2) + Im(z2)Re(z1)  2 2 
For a real number  and a complex number z = x
+ iy. Example 4.5
z = (x + iy) = x + iy  C x y
is the product of a real number with a complex If z = x + iy, z1/3 = a – ib and − =k(a2 – b2), then
a b
number. The following properties are obvious:
find the value of k.
(a) (z1 + z2) = z1 + z2
Solution : (x + iy)1/3 = a – ib
(b) 1(2z) = (12)z;
 x + iy = (a – ib)3 = (a3 – 3ab2) + i(b3 – 3a2b)
(c) (1 + 2)z = 1z + 2z for all z, z1, z2  C and ,
x
1, 2  R.  x = a3 – 3ab2, y = b3 – 3a2b  = a2 – 3b2 and
Actually, relations (a) and (c) are special cases of a
the distributive law and relation (b) comes from y
= b2 – 3a2
the associative law of multiplication for complex b
numbers. x y
 − = a2 – 3b2 – b2 + 3a2 = 4(a2 – b2)
a b
DIVISION OF COMPLEX NUMBER :
Let z1 = x1 + iy1 & z2 = x2 + iy2  k = 4.
z x + iy 1 (x1 + iy1 )(x 2 − iy2 )
Then 1 = 1 
z2 x2 + iy 2 (x 2 + iy2 )(x 2 − iy2 ) Note
(x1 x2 + y1 y2 ) + i(y1 x2 − x1 y2 ) 1. The algebraic operations on complex numbers
 are similar to those on real numbers treating i
(x22 + y22 ) as a polynomial.
 x1 x2 + y1 y 2   y1 x2 − x1 y 2  2. Inequalities in complex numbers (non-real) are
  + i  not defined. There is no validity if we say that
 x2 + y 2   x2 + y 2 
2 2 2 2
complex number (non-real) is positive or
z1 z  z  negative.
 Re  1  + i Im  1 
z2 z
 2  z2  e.g. z > 0, 4 + 2i < 2 + 4i are meaningless.
3. In real numbers, if a2 + b2 = 0, then a= 0 = b
but in complex numbers, z12 + z22 = 0 does not
SQUARE ROOT OF COMPLEX NUMBER : imply z1 = z2 = 0.
Let z = x + iy be the given complex number and
we have to obtain its square root.
Let a + ib = (x + iy)1/2  a2 + b2 + 2iab = x + iy
Example 4.6
 x = a2 – b2 and y = 2ab
 x2 = (a2 + b2)2 – 4a2b2  x2 + y2 = (a2 + b2)2 Let z1 = (2 + 3i) and z2 = 1 + 2i, then
 a2 + b2 = |z| …...(1) (1) z1 + z2 = 2 + 3i + 1 + 2i = 3 + 5i
 a2 – b2 = x ..….(2) (2) z1 – z2 = 2 + 3i – 1 – 2i = 1 + i
(3) z1 × z2 = (2 + 3i) (1 + 2i) = 2 + 4i + 3i – 6
| z | + x | z | +x
 a2 = a=  –4 + 7i
2 2
| z | −x | z | −x Example 4.7
 b2 = b=
2 2 Write (2 + 3i)2 (3 + 2i) in the form a + ib.
 | z | + Re(z) | z | − Re(z)  Solution : Consider
 x + iy = a + b   +i  (2 + 3i)(2 + 3i)(3 + 2i) = (4 – 9 + 12i)(3 + 2i)
 2 2 
= (–5 + 12i)(3 + 2i)
Replacing i by – i, we get
= (–15 – 24) + (–10 + 36)i
 | z | + Re(z) | z | − Re(z)  = –39 + 26i
x − iy =   −i 
 2 2 
Example 4.8
Example 4.4 Find the real and imaginary parts of
Find the square root of 3 + 4i z = (1 + i) (5 + 2i)2
Solution : Let 3 + 4i = a + ib  3 + 4i = a2 – b2 + Solution : Consider
z = (1 + i)(5 + 2i)2 = (1 + i)(25 – 4 + 20i)
2ab  a2 – b2 = 3, 2ab = 4 = (1 + i)(21 + 20i)
 a2 + b2 = (a 2 − b2 )2 + 4a2b2 = 9 + 16 =5 = 21 – 20 + (21 + 20)i
= 1 + 41i
 a + ib = (2 + i)
Therefore, Re(z) = 1 and Im(z) = 41.
Alternative method :
23 Chapter 1 | Fundamentals of Mathematics
Example 4.9 that is, a( 3 – 2 + i) = ( 3 + 2)i – 1. Therefore
If (x2
+ x) + iy and (– x –1) – i (x + 2y) are conjugate ( 3 + 2)i− 1 [( 3 + 2)i− 1][( 3 − 2) − i]
a= =
of each other, then real value of x & y are 3 −2+ i [( 3 − 2) + i][( 3 − 2) − i]
(A) x = –1, y =1 (B) x = 1, y = –1
(C) x = 1, y = 1 (D) x = –1, y = –1 (3 − 4)i − 3 + 2 + i + 3 + 2 4 1
= = =
Solution : x2 + x = –x – 1, y = x + 2y ( 3 − 2) + 1
2
8−4 3 2− 3
x2 + 2x + 1 = 0, x = – y Ans. (D)
x = –1, y = 1 Ans. (A)
Example 4.13
Example 4.10
The smallest positive integer n for which [(1 + i)/(1
The real values of x and y for which the following – i)]n = 1 is
equation is satisfied : (A) 2 (B) 4 (C) 6 (D) 7
(1 + i)x − 2i (2 − 3i)y + i Solution : We have
+ =i
3+ i 3−i 1 + i (1 + i)2
(A) x = 3, y = 1 (B) x = 3, y = –1 = = i and tn = 1 for n = 4, 8, 12, ...
1−i 2
(C) x = –3, y = 1 (D) x = –3, y = –1 Therefore, the smallest positive integer n for which
(1 + i)x − 2i (2 − 3i)y + i n
Solution : + =i 1+i 
3+ i 3−i   = 1 is 4 Ans. (B)
 1−i 
(3 − i)(x + ix − 2i) + (3 + i)(2y − 3iy + i)
=i
(3 + i)(3 − i) Example 4.17
3x + 3ix – 6i – ix + x – 2 + 6y – 9iy + 3i + 2iy + 3y
– 1 = 10i 3+ i
If x + iy = then (x2 + y2)2 equals
4x + 9y – 3 + i(2x – 7y – 13) 1 + 3i
= 4x + 9y – 3 = 0 ... (1) (A) 0 (B) 2 (C) 3 (D) 1
 2x – 7y – 13 = 0 ... (2) 3 + i (3 + i)(1 − 3 i)
23y + 23 = 0 Solution : x2 – y2 + 2ixy = =
1 + 3i 1+9
y = –1  4x – 9 – 3 = 0  4x =12  x = 3
Comparing the real and imaginary parts we get
Ans. (B)
6 −8
x2 – y2 = and 2xy =
10 10
Example 4.11
Now
2
 4i3 − i  (x2 + y2)2 = (x2 – y2)2 + 4x2y2
  can be expressed in a + ib as 2
 6   −8 
2
9 16
 2i + 1  =   +  = + =1 Ans. (D)
(A) 3 + 4i (B) 3 – 4i (C) 4 + 3i (D) 4 – 3i  10   10  25 25
2
 4i3 − i   −5i  −25
Solution :   = = Example 4.15
 2i + 1   2i + 1  −3 + 4i
(1 + i)(2 − 3 i)
−25 −3 − 4i 25(3 + 4i) Find the real and imaginary parts of z =
=  = (1 − i)(2 + 3 i)
−3 + 4i −3 − 4i (−3)2 − (4i)2
Solution : Consider
25(3 + 4i) (1 + i)(2 − 3 i) (2 + 3) + (2 − 3) i
= = 3 + 4i Ans. (A) z= =
25 (1 − i)(2 + 3 i) (2 + 3) + (−2 + 3) i
Example 4.12
5− i (5 − i)2 25 − 1 − 10i
= = =
3+i a+i 5 + i (5 + i)(5 − i) 25 + 1
If = and a is a real number, then a is
2 a −1 24  −10  12  −5 
= + i = + i
(A) 1/2 + 3 (B) 1/2 – 4 3 26  26  13  13 
(C) 2 – 3 (D) 1/2 – 3 Therefore Re(z) = 12/13 and Im(z) = –5/13.
Solution : The equation
3+i a+i
=
2 a −1
implies that ( 3 + i) (a – i) = 2a + 2i

WORKED-OUT PROBLEMS - 4

1. Express each of the following in the form x + yi. (c) 7i − (4 + 5i)


(a) (3 + 5i) + (2 − 3i) (b) (3 + 5i) + 6 Solution :
Chapter 1 | Fundamentals of Mathematics 24
(a) (3 + 5i) + (2 − 3i) = 3 + 2 + (5 − 3)i = 5 + 2i 3 1
(b) (3 + 5i) + 6 = 9 + 5i x2 = y=
2 2
(c) 7i − (4 + 5i) = 7i – 4 − 5i = −4 + 2i
3 1
x=− y=−
2. Simplify in the form x + yi : 2 2
(a) 3(2 + 4i) 1
4 + 3i =  (3 + i)
(b) (5 + 3i)i 2
(c) (2 − 7i)(3 + 4i)
Solution : 5. Find the square root of :
(a) 3(2 + 4i) = 3(2) + 3(4i) = 6 + 12i (a) 9 + 40i (b) –11 – 60i (c) 50i
(b) (5 + 3i)i = (5)i + (3i)i Solution :
= 5i + (−1)3 = −3 + 5i
(c) (2 − 7i)(3 + 4i) = (2)(3) − (7i) (a) 9 + 40i = 9 + 2  5  4i
(3) + (2)(4i) − (7i)(4i) = (5 + 4 i)2 = |5 + 4i| = ±(5+4i)
= 6 − 21i + 8i − (−28) = 6 − 21i + 8i + 28
= 34 − 13i (b) −11 − 60i = −11 − 2  3  5  2i
= (5 − 6 i)2 = ±(5 – 6i)
3. Simplify the expressions :
1 3 4 + 7i (c) 50i = 5 2i
(a) (b) (c)
i 1+i 2 + 5i 5 (1 + i)2 = ± 5(1 + i)
Solution : To simplify these expressions you multiply
the numerator and denominator of the quotient by
6. Simplify and express the result in the form of a + bi
the complex conjugate of the denominator. 2
(a) The complex conjugate of i is –i, therefore  1 + 2i 
(a)   (b) –i(9 + 6i)(2 – i)–1
1 1 −i (1)(− i) −i  2+ i 
=  = = = −i 2
i i −i (i)(− i) −(−1)  4i3 − i  (2 + i)2 (2 − i)2
(c)   (d) −
(b) The complex conjugate of 1+ i is 1–i, therefore  2i + i  2−i 2+ i
3 3 1−i 3(1 − i) 3 − 3i 3 3 Solution :
=  = = = − i
1 + i 1 + i 1 − i (1 + i)(1 − i) 2 2 2  1 + 2i 
2

(c) The complex conjugate of 2 + 5i is 2 – 5i, (a)  


 2+ i 
therefore
1 − 4 + 4i −3 + 4i
4 + 7i 4 + 7i 2 − 5i 43 − 6i 43 6 z= =
=  = = − i 4 − 1 + 4i 3 + 4i
2 + 5i 2 + 5i 2 − 5i 29 29 29
 −3 + 4i  3 − 4i 
=  
4. Find the square root of  3 + 4i  3 − 4i 
(i) 7 + 24 i (ii) 4 + 3i  −9 + 12i + 12i + 16 
= 
Solution : (i) Let 7 + 24i = x + iy  9 + 16 
7 + 24i = x2 – y2 + 2ixy
 7 + 24i  7 24i
x 2 + y2 = 7 ….(1)  = +
xy = 12 ….(2)  25  25 25
(x2 + y2)2 = (x2 – y2) + 4x2y2 (b) –i(9 + 6i)(2 – i)–1
= 42 + 576 −i(9 + 6 i) (6 − 9i ) (2 + i)
z= =
= 625 (2 − i) (2 − i) (2 + i)
x2 + y2 = ±25
12 − 18i + 6i + 9
x2 + y2 = 25 ….(3) =
x2 – y2 = –25 (reject) 4+1
From (1) and (3) 21 − 12i 21 12i
= = −
2x2 = 32  x2 = 16  x = ±4 5 5 5
x = –4  y = –3  4i3 − i 
2

7 + 24i = ± (4 + 3i). (c)  


 2i + 1 
(ii) Let 4 + 3i = x + iy
(−4 i − i)2  −25   −25   4i + 3 
4 + 3i = x2 – y2 + 2ixy z= =  =  
x2 – y2 = 4 (2i + 1) 2
 −4 + 4i + 1   4i − 3   4i + 3 
2xy = 3 −25(4 i+ 3)
(x2 + y2)2 = (x2 – y2) + (2xy)2 = 16 + 2 = = 4i + 3
−16 − 9
x 2 + y2 = 5
(2 + i)2 (2 − i)2
x 2 + y2 = 5 (d) −
3 3 2−i 2+ i
x2 =  x = 
2 2
25 Chapter 1 | Fundamentals of Mathematics
(4 − 1 + 4 i) (4 − 1 + 4 i) 3 + 4i 3 − 4i = 115 + 236i
z= − = − (–5 + 4i)4 = (9 – 40i)2 = 81 – 1600 – 720i
2− i 2+ i 2− i 2+ i
6 + 8i + 3i − 4 − (6 − 3 i− 8 i− 4) = –1519 – 720i
= f(–5 + 4i) = (– 5 + 4i)4 + 9(–5 + 4i)3 + 35(– 5 + 4i)2
4+1 – (– 5 + 4i) + 4
2 + 11i − 2 + 11 i 22i = –1519 –720i + 9(115 + 236i) + 35(9 – 40i) + 5 –
 =
5 5 4i + 4
= –1519 + 1035 + 315 + 9 – 720i + 2124i – 1400i – 4i
7. Given that x, y  R solve = –160
(a) (x + 2y) + i(2x – 3y) = 5 – 4i
(b) (x + iy) + (7 – 5i) = 9 + 4i 10. If g(x) = x4 – x3 + x2 + 3x – 5, find g(2 + 3i)
(c) x2 – y2 – i(2x + y) = 2i Solution : (2 + 3 i)2 = 4 – 9 + 12i = –5 + 12i
(d) (2 + 3i)x2 – (3 – 2i)y = 2x – 3y + 5i (2 + 3i)3 = (2 + 3i) (–5 + 12i)
(e) 4x2 + 3xy + (2xy – 3x2)i = 4y2 – (x2/2) + = –10 – 15i + 24i – 36
(3xy – 2y2)i = –46 + 9i
Solution : (a) (x + 2y) + i (2x – 3y) = 5 – 4i (2 + 3i)4 = (–5 + 12i)2 = 25 – 144 – 120i
x + 2y = 5 = –119 – 120i
2x – 3y = – 4 g(2 + 3i) = (2 + 3i)4 – (2 + 3i)3 + (2 + 3i)2 + 3(2 +
By solving x = 1; y = 2 3i) – 5
(b) (x + iy) + (7 – 5i) = 9 + 4i = –119 – 120i + 46 – 9i – 5 + 12i + 6 + 9i – 5
x + 7 + i (y – 5) = 9 + 4i = –77 – 108i
x+7=9x=2
y–5=4y=9 11. Show that a real value of x will satisfy the equation
(c) x2 – y2 – i (2x + y) = 2i 1 − ix
= a – ib, if a2 + b2 = 1.
x2 + y2 = 0  y = ± x 1 + ix
2 2 1 − ix 1 − (a − ib)
y=xx= − y= − Solution : We have = a – ib or ix =
3 3 1 + ix 1 + (a − ib)
y = –x  x = –2  y = –2 [by componendo and dividendo],
 2 2
point  − , −  (–2, –2) 1 − a + ib (1 − a ) + ibb − i(1 + a)
 3 3 or x = =
b + i(1 + a) b2 + (1 + a)2
(d) (2 + 3i) x2 – (3 – 2i) y = 2x – 3y + 5i
(2x2 – 3y) + i (3x2 + 2y) = 2x – 3y + 5i 2b + i(a 2 + b2 − 1)
=
2x2 – 3y = 2x – 3y & 3x2 + 2y = 5 b2 + (1 + a)2
2x2 = 2x  x = 0, 1 Therefore, x will be real, if a2 + b2 = 1.
5  5
x=0y= points  0,  , (1, 1)
2  2 12. Find the square root of a + ib
x=1 y=1 Solution : Let a + ib = x + iy, where x and y are real.
 x2  Squaring, a + ib = x2 – y2 + i2xy.
(e) 4x2 = 3xy + (2xy – 3x2)i = 4y2 –   + (3xy – 2y2)i Equating real and imaginary parts, a = x2 – y2 .....(i),
 2
b = 2xy .....(ii)
x2 Now (x2 + y2)2 = (x2 – y2)2 + 4x2y2 = a2 + b2 or x2 +
4x2 + 3xy = 4y2 – & 2xy – 3x2 = 3xy – 2y2
2
y2 = a2 + b2 .....(iii)
3k
By solving x = k, y = ,kR [ x and y are real, the sum of their squares must be
2
positive]
8. Express (1 + 2i)2/(2 + i)2 in the form x + iy. a2 + b2 + a
From (i) and (iii), x2 = or
(1 + 2 i)2 1 + 4i − 4 −3 + 4i 2
Solution : = =
(2 + i)2 4 + 4i − 1 3 + 4i
a2 + b2 + a
(−3 + 4 i)(3 − 4 i) x=
= 2
(3 + 4 i)(3 − 4 i)
a2 + b2 − a a2 + b2 − a
−9 + 16 + 24i 7 24 and y2 = or y = 
 the expression = +i 2 2
9 + 16 25 25
If b is positive, both x and y have the same signs and
in opposite case, contrary signs. [by (ii)].
9. If f(x) = x4 + 9x3 + 35x2 – x + 4, find f(– 5 + 4i)
Solution : (–5 + 4i)2 = 25 – 16 – 40i = 9 – 40i
(–5 + 4i)3 = (–5 + 4i) (9 – 40i) = – 45 + 36i + 200i
+ 160
Chapter 1 | Fundamentals of Mathematics 26

LECTURE-5 SET THEORY

DEFINITION e.g. 2, 3, 5 ,...., e, log2 etc. are all irrational


A set is a collection of well-defined objects which
numbers.
are distinct from each other. Sets are generally
denoted by capital letters A, B, C, ....... etc. and the
elements of the set by small letters a, b, c ....... etc.
METHODS TO WRITE A SET
(i) Roster Method or Tabular Method : In this method a
If a is an element of a set A, then we write a  A
set is described by listing elements, separated by
and say a belongs to A.
commas and enclose then by curly brackets. Note
If a does not belong to A then we write a A,
that while writing the set-in roster form, an element
e.g. the collection of first five prime natural numbers
is not generally repeated e.g. the set of letters of
is a set containing the elements 2, 3, 5, 7, 11.
word SCHOOL may be written as {S, C, H, 0, L}.
NOTATION (ii) Set builder form (Property Method) : In this we
Suppose S is a set (i.e., a collection of objects
write down a property or rule which gives us all
enjoying a certain property).
the element of the set.
1. The symbol  stands for “belongs to” or “is a A = {x : P(x)} where P(x) is the property by which
member of”: a  S means a belongs to S or is a x  A and colon ( : ) stands for ‘such that’.
member of S.
2. The symbol  stands for “does not belong to”. a 
S means a is not a member of S. Example 5.1
3. The symbols  and  stand for “there exists” and Express set A = {x : x  N and x = 2n for n  N} in
“such that”, respectively. Generally, these two roster form
symbols go together, for example,  real number x Solution : A= {2, 4, 8, 16, .........}
 x2 = 2.
4. The symbol ““ stands for “for all” or “for every” Example 5.2
For example, “x2 is a positive integer  non-zero
integer x” Express set B = {x3 : x < 5, x  W} in roster form
5. If S is the set of all objects satisfying a property P, Solution : B = {0, 1, 8, 27, 64}
then S is represented as
S = {x | x has property P} Example 5.3
6. The set having no objects is called the empty set
Express set A = {0, 7, 26, 63, 124} in set builder form
or null set and is denoted by ““.
7. If a set has only a finite number of members x1, x2, Solution : A = {x : x = n3 – 1, n  N, 1 ≤ n ≤ 5}
…., xn, then we write
S = {x1, x2, …., xn}
(iii) Arrow-diagram form : In this form, we draw an
8. The symbol  is read as “implies”. Thus, a  b is
arrow corresponding to each ordered pair (a, b)
read as “a implies b”. The symbol  is read as
in R from the first component a to the second
“implies and is implied by” or as “if and only if”.
component b. For example, consider the relation
Thus, a  b is read as “a implies and implied by b”
R given as below. Then R can be represented as
or “an if and only if b”.
shown in Figure. There are nine arrows
corresponding to nine ordered pairs belonging to
SOME IMPORTANT NUMBER SETS the relation R.
N = Set of all-natural numbers R = {(2, 2), (2, 4), (2, 6), (2, 8), (2, 10), (3, 6), (4,
= {1,2,3,4, ....} 4), (4, 8), (5, 10)}
W = Set of all whole numbers
= {0, 1, 2, 3, ....}
Z or I set of all integers
= {.... – 3, – 2, – 1,0, 1,2,3, ....}
Z+ = Set of all +ve integers
= {1,2,3, ....} = N.
Z– = Set of all –ve integers
= {–1, –2, –3, ....}
Z0 = The set of all non-zero integers FIGURE : Representation of arrow-diagram form.
= {±1, ±2, ±3, ....}
Q = The set of all rational numbers TYPES OF SETS
p  Null set or Empty set : A set having no element in
=  : p,q  I,q  0  it is called an Empty set or a null set or a void set
q  it is denoted by  or { }
R = The set of all real numbers Ex. A = (x  N : 5 < x < 6} = 
R – Q = The set of all irrational numbers
27 Chapter 1 | Fundamentals of Mathematics
A set consisting of at least one element is called a Power set : Let A be any set. The set of all subset of A
non-empty set or a non-void set. is called power set of A and is denoted by P(A).
Singleton : A set consisting of a single element is Ex. 1 Let A = {1, 2} then P(A) = {, {1}, {2}, {1, 2}}
called a singleton set. Ex. 2 Let P() = {}
Ex. Then set {0}, is a singleton set  P(P()) = {, {}}
Finite Set : A set which has, only finite number of  P(P(P( )) = {, {}, {{}}, { { }}
elements is called a finite set.
Ex. A= {a, b, c} Note
Order of a finite set : The number of element in a 1. If A= then P(A) has one element
finite set A is called the order of the set A and is 2. Power set of as given set is always non empty
denoted O(A) or n(A). It is also called cardinal
number of the set. Some Operation on Sets :
Ex. A = {a, b, c, d}  n(A) = 4 (i) Union of two sets : A  B = {x : x  A or x  B}
Infinite set : A set which has an infinite number of e.g. A = {1, 2, 3} B = {2, 3, 4} then A  B = {1, 2, 3 4}
elements is called an infinite set. (ii) Intersection of two sets : A  B = {x : x  and x  B}
Ex. A = {l, 2, 3, 4, .....} is an infinite set e.g. A = {1, 2, 3} B = {2, 3, 4} then A  B = {2, 3}
Equal sets : Two sets A and B are said to b equal if (iii) Difference of two sets : A – B = {x : x  A and x  B}
every element of A is a member of B, and every e.g. A = {1, 2, 3} B = {2, 3, 4}; A – B = {1}
element of B is a member of A. (iv) Complement of a set : A’ = {x : x  A but x  U}
If sets A and B are equal. We write A = B and A and =U–A
B are not equal then A  B e. g. U = {1, 2, ….., 10} A = {1, 2, 3, 4, 5} then
Ex. A = {l, 2, 6, 7} and B = {6, 1, 2, 7}  A = B A’ = {6, 7, 8, 9, 10}
Equivalent sets : Two finite sets A and B are (v) De-Morgan Laws : (A  B)’ = A’  B’;
equivalent if their number of elements are same (A  B)’ = A’  B’
i.e. n(A) = n(B) (vi) A – (B  C) = (A – B)  (A – C);
Ex. A = {1, 2, 3, 7} B = {a, b, c, d} A – (B  C) = (A – B)  (A – C)
n(A) = 4 and n(B) = 4  n(A) = n(B) (vii) Distributive Laws : A  (B  C) = (A  B)  (A  C);
A  (B  C) = (A  B)  (A  C)
Note (viii) Commutative Laws : A  B = B  A; A  B = B  A
Equal set always equivalent but equivalent sets (ix) Associative Laws : (A  B)  C = A  (B  C);
may not be equal (A  B)  C = A  (B  C)
Subsets : Let A and B be two sets if every element (x) A   = ; A  U = A
of A is an element of B, then A is called a subset of A   = ; A  U = U
B if A is a subset of B. we write A  B (xi) A  B  A; A  B  B
Ex. A = {l, 2, 4} and B = {1, 2, 3, 4, 5, 6, 7}  A  B (xii) A  A  B; B  A  B
The symbol “” stands for “implies” (xiii) A  B  A  B = A
Proper subset : if A is a subset of B and A  B then (xiv) A  B  A  B = B
A is a proper subset of B. and we write A  B.
Disjoint Sets : If A  B = . then A, B are disjoint.
e.g. if A = {1, 2, 3} B = {7, 8, 9} then A  B = 
Note
1. Every set is a subset of itself i.e. A  A for all A
2. Empty set , is a subset of every set Note
3. Clearly N  W  Z  Q  R  C A, A’ are disjoint if A  A’ = 
4. The total number of subsets of a finite set
containing n elements is 2n Symmetric Difference of sets :
A  B = (A – B)  (B – A)
• (A’)’ = A
Universal set : A set consisting of all possible
• A  B  B’  A’
elements which occur in the discussion is called a
If A and B are any two sets, then
universal set and is denoted by U
(i) A – B = A  B’
(ii) B – A = B  A’
Note (iii) A – B = A  A  B = 
All sets are contained in the universal set.
(iv) (A – B)  B = A  B
(v) (A – B)  B = 
Ex. If A= {1, 2, 3} B = {2, 4, 5, 6}, C = {1, 3, 5, 7}, (vi) (A – B)  (B – A) = (A  B) – (A  B)
then = {1, 2, 3, 4, 5, 6, 7} can be taken as the
universal set.

WORKED-OUT PROBLEMS - 5

1. Verify whether the following are sets:


Chapter 1 | Fundamentals of Mathematics 28
(1) The collection of all intelligent persons in Solution : (i) {x : x = 3n,n  N and 1 ≤ n ≤ 4}
Visakha-patnam. (ii) {x : x = 2n, n  N and 1 ≤ n ≤ 5}
(2) The collection of all prime ministers of India. (iii) {x : x = 5n, n  N and 1 ≤ n ≤ 4}
(3) The collection of all negative integers. (iv) {x : x is an even natural number}
(4) The collection of all tall persons in India. (v) {x : x = n2, n  N and 1 ≤ n ≤ 10}
Solution : Note that the collections given in (1) and (4)
are not sets because, if we select a person in 6. Fill in the blank infinite/finite
Visakhapatnam, we cannot say with certainty whether (1) The set Z+ of positive integers is an ______ set.
he/she belongs to the collection or not, as there is no (2) {a, b, c, d} is a _____ set, since it has exactly four
stand and scale for the evaluation of intelligence or for elements.
being tall. However, the collections given in (2) and (3) The set R of real numbers is an _________ set.
(3) are sets. (4) {x|x  Z and 0 < x ≤ 100} is _________ set.
(5) {x|x  Q and 0 < x < 1} is an _________ set.
2. Which of the following are sets? Justify your Solution : (1) Infinite (2) Finite
answer. (3) Infinite (4) Finite
(i) The collection of all the months of a year (5) Infinite
beginning with the letter J.
(ii) The collection of ten most talented writers of 7. Let A = {1, 2, 3, 4, 5, 6}. Insert the appropriate
India. symbol  or  in the blank spaces:
(iii) A team of eleven best-cricket batsmen of the (i) 5. . .A (ii) 8 . . . A (iii) 0. . .A
world. (iv) 4. . . A (v) 2. . .A (vi) 10. . .A
(iv) The collection of all boys in your class. Solution : (i)  (ii)  (iii)  (vi) 
(v) The collection of all-natural numbers less
(v)  (vi) 
than 100.
(vi) A collection of novels written by the writer 8. In the following, state whether A = B or not:
Munshi Prem Chand. (i) A = {a, b, c, d} B = {d, c, b, a}
(vii) The collection of all even integers. (ii) A = {4, 8, 12, 16} B = {8, 4, 16, 18}
(viii) The collection of questions in this Chapter. (iii) A = {2, 4, 6, 8, 10} B = {x : x is positive even
(ix) A collection of most dangerous animals of the integer and x ≤ 10}
world. (iv) A = {x : x is a multiple of 10}, B = {10, 15, 20,
Solution : (i), (iv), (v), (vi), (vii) and (viii) are sets. 25, 30, . . . }
Solution : (i) Yes (ii) No (iii) Yes (iv) No
3. (1) Let P be the collection of all prime numbers.
Then it can be represented in the set builder
9. If  denotes null set then find
form as
(a) P() (b) P(P())
(2) Let X be the set of all even positive integers
(c) n(P(P(P()))) (d) n(P(P(P(P()))))
which are less than 15. Then express this set
Solution :
in set builder and roster form.
Solution : (1) P = {x|x is a prime number} (a) P() = {}
(2) X = {x|x is even integer and 0 < x < 15} (b) P(P()) = {,{}}
= {2,4,6,8,10,12,14} (c) n(P(P(P()))) = 22 = 4
(d) n(P(P(P(P())))) = 24 = 16
4. Write the following sets in roster form:
(i) A = {x : x is an integer and –3 ≤ x < 7} 10. State true/false :
(ii) B = {x : x is a natural number less than 6} (1) A = {p, q, r, s}, B = {p, q, r, p, t} then A  B.
(iii) C = {x : x is a two-digit natural number such (2) A= {p, q, r, s}, B = {s, r, q, p} then A  B.
that the sum of its digits is 8} (3) [4, 15)  [–15, 15]
(iv) D = {x : x is a prime number which is divisor Solution :
of 60} (1) False
(v) E = The set of all letters in the word (2) True
TRIGONOMETRY (3) True
(vi) F = The set of all letters in the word BETTER
Solution : (i) A = {–3, –2, –1, 0, 1, 2, 3, 4, 5, 6} 11. Are the following pair of sets equal ? Give reasons.
(ii) B = {1, 2, 3, 4, 5} (i) A = {2, 3}, B = {x : x is solution of x2 + 5x + 6 = 0}
(iii) C = {17, 26, 35, 44, 53, 62, 71, 80} (ii) A = {x : x is a letter in the word FOLLOW}
(iv) D = {2, 3, 5} B = {y : y is a letter in the word WOLF}
(v) E = {T, R, I, G, O, N, M, E, Y} Solution : (i) No (ii) Yes
(vi) F = {B, E, T, R}
12. From the sets given below, select equal sets :
5. Write the following sets in the set-builder form : A = {2, 4, 8, 12}, B = {1, 2, 3, 4}, C = {4, 8, 12, 14},
(i) {3, 6, 9, 12} (ii) {2,4,8,16,32} D = {3, 1, 4, 2}, E = {–1, 1}, F = { 0, a}, G = {1, –1},
(iii) {5, 25, 125, 625} (iv) {2, 4, 6, . . .} H = {0, 1}
(v) {1,4,9, . . .,100} Solution : B = D, E = G
29 Chapter 1 | Fundamentals of Mathematics
A – B = {a, d, e} and B – A = {f, g}
13. Let A= {x|x is an odd prime and x < 20} and B = Therefore
{x|x is an integer and x > 6}. Find A  B. A  B = {a, d, e}  {f, g} = {a, d, e, f, g)
Solution : A = {3, 5, 7, 11, 13, 17, 19} and
B = {7, 8, 9, 10, 11, 12, ...} 18. Let A = The set of all non-negative integers and B =
Therefore, The set of all non-positive integers. Find A  B.
A  B = {7, 11, 13, 17, 19} Solution : A  B = {x|x is an integer, x ≥ 0 and x ≤ 0} = {0}.

14. Let A be the interval [0, 1] and B the interval [1/2, 2]. 19. The set A = {x : x  R, x2 = 16 and 2x = 6} is equal to-
Then find A  B and A  B. (l)  (2) {14, 3, 4}
Solution : We have (3) {3} (4) {4}
A  B = {x|x  A or x  B} Solution : x2 = 16  x = ±4
1 2x = 6  x = 3
= {x|x  R and ‘0≤ x ≤ 1 or ≤ x ≤ 2’}
There is no value of x which satisfies both the
2
= {x|x  R and 0 ≤ x ≤ 2} above equations.
= [0, 2] Thus, A =  Ans. (1)
Also,
1  20. Let A = {x : x  R, |x| < 1}; B = {x : x  R, |x – 1| ≥
A  B =  ,1 1} and A  B = R – D, then the set D is-
2  (l) {x : 1< x ≤ 2} (2) {x : 1 ≤ x < 2}
(3) {x : 1 ≤ x ≤ 2} (4) none of these
15. If A = {3, 5, 7, 9, 11}, B = {7, 9, 11, 13}, C = {11, 13, Solution :
15} and D = {15, 17}; find A = {x : x  R, –1 < x < 1}
(i) A  B (ii) B  C (iii) A  C  D B = {x : x  R : x – 1 ≤ –1 or x – 1 ≥ 1}
(iv) A  C (v) B  D (vi) A  (B  C) = {x : x  R : x ≤ 0 or x ≤ 2}
(vii) A  D (viii) A  (B  D) AB=R–D
(ix) (A  B)  (B  C) (x) (A  D)  (B  C) where D = {x : x  R, 1 ≤ x < 2} Ans. (2)
Solution : (i) {7, 9, 11} (ii) {11, 13}
(iii)  (iv) {11} (v)  21. If aN= {ax : x  N}, then the set 6N  8N is equal
(vi) {7, 9, 11} (vii)  (viii) {7, 9, 11} to-
(ix) {7, 9, 11} (x) {7, 9, 11} (1) 8N (2) 48N (3) 12N (4) 24N
Solution : 6N = {6, 12, 18, 24, 30, .....}
16. Let A be the set of all even primes and B the 8N = {8, 16, 24, 32 .....}
interval (2, 3). Find A  B.  6N  8N = {24, 48, ......} = 24N
Solution : A  B = {x|x is an even prime or x  (2, 3)} Short cut Method
= {x|x = 2 or x  R. such that 2 < x < 3} 6N  8N=24N [24 is the L.C.M. of 6 and 8]. Ans. (4)
= {x|x  R. and 2 ≤ x < 3}
= [2, 3) 22. If R is relation on the set A = {1, 2, 3, 4, 5, 6, 7, 8,
9} given by x Ry  y = 3x then R =
17. Find the symmetric difference of the following: (A) {(3, 1), (6, 2), (8, 2), (9, 3)} (B) {(3, 1),
(1) A= {1, 2, 3, 4} and B = {4, 5, 6} (6, 2), (9, 3)}
(2) A= {a, b, c, d, e) and B = (b, c, f, g) (C) {(3, 1), (2, 6), (3, 9)} (D) {(1, 3),
Solution : (2, 6), (3, 9)}
(1) We have A = {1, 2, 3, 4} and B = {4, 5, 6}. Then Solution : x = 1  y = 3
A – B = {1, 2, 3} and B – A = {5, 6} x=2y=6
Therefore x = 3  y = 9  R = {(1, 3), (2, 6), (3, 9)}
A  B = {1, 2, 3}  {5, 6} = {1, 2, 3, 5, 6}
(2) From the given sets we have
30 Chapter 1 | Fundamentals of Mathematics

LECTURE-6 THEORY OF CARDINAL NUMBER


In mathematics, cardinal numbers, or cardinals
for short, are a generalization of the natural
numbers used to measure the cardinality (size) of
sets. The cardinality of a finite set is a natural
number: the number of elements in the set. The
transfinite cardinal numbers describe the sizes of
infinite sets.
Cardinality is defined in terms of bijective
functions. Two sets have the same cardinality if,
and only if, there is a one to- one correspondence FIGURE : A Venn diagram.
(bijection) between the elements of the two sets.
In the case of finite sets, this agrees with the In Figure, two intersecting sets A and B are
intuitive notion of size. In the case of infinite sets, represented by the intersecting circles, indicating
the behaviour is more complex. A fundamental that the common area of the circles represents the
theorem due to Georg Cantor shows that it is intersection A  B.
possible for infinite sets to have different
cardinalities, and in particular the cardinality of
the set of real numbers is greater than the
cardinality of the set of natural numbers.

FIGURE : Two intersecting sets A and B.


A bijective function, f : X → Y, from set X to set Y
demonstrates that the sets have the same Figure represents the statement "A is a subset of B".
cardinality, in this case equal to the cardinal
number 4.r
e.g. A = {a, e, i, o, u}
n(A) = 5
e.g. n() = 0
e.g. n(Z) =  (tends to infinity)
e.g. Set of all odd number which gives the
remainder as zero when divided by 2
S=={} FIGURE : Representation of “A is a subset of B".
n(S) = 0
The shaded parts in Figures represent the union
VENN DIAGRAM of two sets A and B, namely A  B in the cases A 
A set is represented by a closed curve, usually a B = , A  B   and A  B, respectively.
circle, and its elements by points within it. This
facilitates better understanding and a good
insight. A statement involving sets can be easily
understood with pictorial representation of the
sets. The diagram showing these sets is called the
Venn diagram of that statement, named after the
British logician John Venn (1834-1883).
Usually the universal set is represented by a
rectangle and the given sets are represented by
circles or closed geometrical figures inside the FIGURE : Representation of A  B when A  B = .
rectangle representing the universal set. An
element of set A is represented by a point within
the circle representing A.

In Figure, the rectangle represents the universal


set S, A and B represent two disjoint sets
contained in S and a and b represent arbitrary
elements in A and B, respectively.
FIGURE : Representation of A  B when A  B  .
Chapter 1 | Fundamentals of Mathematics 31
FIGURE : Representation of A – B when A  B. In this
case A – B = ·

FIGURE : Representation of A  B when A  B.

Figures represent the intersection A  B in these FIGURE : Representation of A – B when A  B = .


cases.

FIGURE : Representation of A – B when A  B and B


FIGURE : Representation of A  B when A  B = .  A.

The symmetric differences A  B [= (A – B)  (B –


A)) are represented by the shaded parts in the
Figures in these cases.

FIGURE : Representation of A  B when A  B  .

FIGURE : Representation of A  B when B  A.

FIGURE : Representation of A  B when A  B.

The shaded parts in Figures represent the


difference A – B in various cases.
FIGURE : Representation of A  B when A  B.

FIGURE : Representation of A – B when B  A.


FIGURE : Representation of A  B when A  B = .
32 Chapter 1 | Fundamentals of Mathematics

FIGURE : Representation of A  B when A  B and B FIGURE : Representation of (A  B) C.


 A. Figures and represent the property
Figure represents the complement of a set A in a A – (B  C) = (A – B)  (A – C)
universal set S.

FIGURE : Representation of A – (B  C).


FIGURE : Complement of a set A.
Figures illustrate the cases A  B, (A  B) – C and
C – (A  B), respectively.

FIGURE : Representation of (A – B)  (A – C).

LAWS OF ALGEBRA OF SETS (PROPERTIES OF


FIGURE : Representation of A  B. SETS):
(i) Commutative law : (A  B) = B  A;
AB=BA
(ii) Associative law : (A  B)  C = A  (B  C);
(A  B)  C = A  (B  C)
(iii) Distributive law : A  (B  C)= (A  B)  (A
 C); A (B  C) = (A  B)  (A  C)
(iv) De–morgan law : (A  B)' = A'  B'; (A  B)'
= A'  B'
(v) Identity law : A  U= A ; A   = A
FIGURE : Representation of (A  B) – C. (vi) Complement law : A  A' = , A  A' = ,
(A')' = A
(vii) Idempotent law : A  A = A, A  A = A

Note
(i) A – (B  C)= (A – B)  (A – C); A – (B  C)=
(A– B)  (A – C)
(ii) A   = , A  U = U

FIGURE : Representation of C – (A  B). SOME IMPORT ANT RESULTS ON NUMBER OF


ELEMENTS IN SETS :
(A  B) C is represented by Figure. From this Summary of the formulas
one can easily see that (A  B) C = (A  B) C. Let A, B and C be given finite sets and Sa universal
finite set containing A, B and C. Then the following
hold:
1. n(A  B) + n(A  B) = n(A) + n(B)
2. n(A  B) = n(A – B)+ n(B – A)+ n(A  B)
3. n(A  B) = n(A) + n(B)  A  B = 
Chapter 1 | Fundamentals of Mathematics 33
4. n(A) = n(A – B)+ n(A  B)  n(E) + n(H) – n(E  H) = 48
5. The number of the elements belonging to  n(E  H) = 58 – 48 = 10
exactly one of A and B is
n(A  B) = n(A – B)+ n(B – A) Example 6.6
= n(A) + n(B) – 2n(A  B)
In a group of 50 persons, 14 drink tea but not coffee
= n(A  B) – n(A  B)
and 30 drink tea. Find
6. The number of elements belonging to exactly
(i) How many drink tea and coffee both ?
one of A,B and C is
(ii) How many drink coffee but not tea ?
n(A) + n(B) + n(C) – 2n(A  B) – 2n(B  C) Solution : T : people drinking tea
– 2n(C  A) + 3n(A  B  C) C : people drinking coffee
7. The number of elements belonging to exactly (i) n(T) = n(T – C) + n(T  C)  30 = 14 + n(T  C)
two of A, B and C is
 n(T  C) = 16
n(A  B) + n(B  C) + n(C  A) – 3n(A  B  C)
8. n(A'  B') = n(S) – n(A  B)
9. n(A'  B') = n(S) – n(A  B)

Example 6.1
Let A= {1, 2, 3, 4, 5, 6} and B = {4, 5, 6, 7, 8, 9} then find
AB
Solution : A  B = {1, 2, 3, 4, 5, 6, 7, 8, 9}
(ii) n(C – T) = n(T  C) – n(T) = 50– 30 = 20
Example 6.2
Let A = {1, 2, 3, 4, 5, 6}, 8 = {4, 5, 6, 7, 8, 9}. Find A – B Example 6.7
and B – A. If A and B are sets such that n(A) = 9, n(B) = 16 and
Solution : A – B = {x: x  A and x B} = {1, 2, 3} n(A  B) = 25, find A  B.
similarly B – A – {7, 8, 9} Solution : We have
n(A  B) = n(A) + n(B) – n(A  B)
Example 6.3 Therefore, substituting the values we get
State true or false : 25 = 9 + 16 – n(A  B)
(i) A  A'= A = 25 – n(A  B)
(ii) U  A=A 0 = n(A  B)
Solution : (i) false because A  A' = U Hence A  B = .
(ii) true as U  A = A
Example 6.8
Example 6.4 If A and B are sets such that n(A)=14, n(A  B)=26
Use Venn diagram to prove that A – B = A  B'. and n(A  B) = 8, then find n(B).
Solution : Solution : We have
n(B) = n(A  B) + n(A  B) – n(A)
= 26 + 8 – 14 = 20

Example 6.9
If A, B and C are sets defined as A= {x|x  Z+ and x ≤ 16},
B = {x|x  Z and –3 < x < 8} and C = {x|x is a prime
number}, then find the number of elements belonging to
exactly two of A, B and C, even though C is an infinite set.
Solution : We have
From Venn diagram we can conclude that A – B = n(A) = 16, n(B) = 10 and n(C) = 
A  B'. Now
A  B = {1, 2, 3, 4, 5, 6, 7}
Example 6.5 B  C = {2, 3, 5, 7}
C  A = {2, 3, 5, 7, 11, 13}
In a group of 60 students, 36 read English newspaper, and
22 read Hindi newspaper and 12 read neither of the A  B  C= {2, 3,5, 7}
two. How many read both English & Hindi news Therefore, the required number is
papers ? n(A  B)+ n(B  C)+ n(C  A)– 3n(A  B  C)
Solution : n(U) = 60, n(E) = 36, n(H) = 22 = 7 + 4 + 6 – 3 × 4=5
n(E'  H') = 12  n(E  H)'= 12
 n(U) – n(E  H) = 12 Example 6.10
 n(E  H) = 48
34 Chapter 1 | Fundamentals of Mathematics
In a class there are 400 students. Following is a table n(M  P) = 100, n(M  C) =60, n(P  C) = 40,
showing the number of students studying one or more n(M  P  C)= 30
of the subjects mentioned: We have,
Mathematics 250 n[M– (P  C)] n(M) – n[M  (P  C))
Physics 150 = n(M) – n[(M  P)(M  C)]
Chemistry 100 = n(M) –[n(M  P)+n(M  C) –
Mathematics and Physics 100 n(M  P  M  C))
Mathematics and Chemistry 60 = n(M)– n(M  P) –n(M  C) + n(M  P  C)
Physics and Chemistry 40 =250–100–60+30 = 120
Mathematics, Physics and Chemistry 30 Therefore 120 students study only Mathematics. Also
Only Mathematics _____ n[P – (M  C))= n(P) – n[P  (M  C))
Only Physics _____ = 150 – n[(P  M)  (P  C))
Only Chemistry _____
= 150 – n(P  M) – n(P  C) +
None of Mathematics, Physics and Chemistry _____
n(P  M  C)
Fill in the empty places in the above table.
= 150 – 100 – 40 + 30 = 40
Solution : Let M, P and C stand for the set of students
Therefore 40 students study only Physics. Similarly,
studying Mathematics, Physics and Chemistry. Let S be
n[C – (M  P)]= n(C) – n[C  (M  P)]
the set of all students in the class. The Venn diagram is
as follows: = 100 – n(C  M) – n(C  P) +
n(C  M  P)
= 100 – 60 – 40 + 30 = 30
Therefore 30 students study only Chemistry. Again
n(M  P  C) = n(M) + n(P) + n(C) – n(M  P) –
n(P  C) – n(C  M) + n(M  P  C)
= 250 + 150 + 100 – 1 00 – 40 – 60 + 30
= 330
n[S – (M  P  C))= n(S) – n(M  P  C)
= 400 – 330 = 70
We are given that Therefore 70 students study none of Mathematics,
n(S) = 400, n(M) = 250, n{P) = 150, n(C) = 100 Physics and Chemistry.
Also, from the table,

WORKED-OUT PROBLEMS - 6
6. (i) If A, B, Care sets such that n(A) = 12, n(B) =
1. Find A  B if A = {x : x = 2n + 1, n ≤ 5, n  N} and 16, n( C)= 18, n(A  B)= 6, n(B  C)= 8,
B = {x : x = 3n – 2, n ≤ 4, n  N}. n(C  A)= 10 and n(A  B  C)= 4, then find the
Solution : {1, 3, 4, 5, 7, 9, 10, 11} number of elements belonging to exactly one of A,
B and C.
2. Find A– (A– B) if A= {5, 9, 13, 17, 21} and B = {3, (ii) Find the number of elements belonging to
6, 9, 12, 15, 18, 21, 24} exactly two of A, B and C.
Solution : {9, 21} Solution : (i) The number of elements belonging to
exactly one of A, B and C is
3. If S and T are two sets such that S has 21 elements, n(A) + n(B) + n(C)– 2n(A  B)– 2n(B  C) –
T has 32 elements, and S  T has 11 elements, how 2n(C  A) + 3n(AB  C)
many elements does S  T have? = 12 + 16 + 18 – 2 × 6 – 2 × 8 – 2 × 10 + 3 × 4 = 10
Solution : n(S  T) = n(S) + n(T) – n(S  T)
n(S  T) = 21 + 32 – 11 = 42 (ii) The number is
n(A  B) + n(B  C)+ n(C  A)– 3n(A  B  C)
4. If X and Y are two sets such that X has 40 elements, = 6 + 8 + 10 – 3 × 4 = 12
X  Y has 60 elements and X  Y has 10 elements,
how many elements does Y have? 7. In a group of 80 students, 50 play football, 45 play
Solution : n(X  Y) = n(X) + n(Y) – n(X  Y) cricket and each student plays either football or
60 = 40 + n(Y) – 10 cricket. Find the number of students who play
= n(Y) = 30 both the games.
Solution : Let F be the set of the students who play
5. In a group of 70 people, 37 like coffee, 52 like tea football and C be the set of students who play cricket.
and each person likes at least one of the two Then n(F) =50 and n(C)= 45.
drinks. How many people like both coffee and tea? Since each of the 80 students play at least one of the
Solution : n(X  Y) = n(X) + n(Y) – n(X  Y) two games, we have n(F  C) = 80. Therefore,
70 = 37 + 52 – n(X  Y) n(F  C) = n(F) + n(C)– n (F  C)
n(X  Y) = 19 = 50 + 45 – 80 = 15
Chapter 1 | Fundamentals of Mathematics 9
m
3m =  n(Yj ) = 9  s = 9  15 = 135
j=1

Therefore, m = 45.

11. There are 200 students in a school. Out of these,


100 students play cricket, 50 students play hockey
and 60 students play basketball. 30 students play
both cricket and hockey, 35 students play both
hockey and basketball, and 45 students play both
8. If 65o/o of people in a town like apples and 78°/o basketball and cricket.
like mangoes, then find out the percentage of (a) What is the maximum number of students
people who like both apples and mangoes and the who play at least one game?
percentage of people who like only mangoes. (b) What is the maximum number of students
Solution : Let the total number of people in the village who play all the three games?
be 100. Let A be the set of people who like apples and (c) What is the minimum number of students
M the set of people who like mangoes. Then n(A) = 65, playing at least one game?
n( M) = 78 and n(A  M) = 100. Therefore (d) What is the minimum number of students who
n(A  M) = n(A) + n(M)– n(A  M) play all the three games?
= 65 + 78 – 100 = 43 Solution : Consider the Venn diagram given above:
Hence 43% of people like both apples and mangoes. At first we will convert all the values in terms of x,
Also, which can be seen above.
n(M)– n(A  M) = 78– 43 = 35 Since the number of students cannot be negative.
Therefore, 35% of people like only mangoes. x – 15 ≥ 0
 x –20 ≥ 0
9. The total number of students in a school is 600. If So, iv. For the minimum number of students playing all
150 students drink apple juice, 250 students drink three games, i.e., x = 20.
pineapple juice and 100 students drink both apple For maximum value of x, again none of the categories
juice and pineapple juice, then find the number of should have – ve number of students.
students who drink neither apple juice nor  30 – x ≥ 0
pineapple juice. x ≤ 30
Solution : Let If x is more than 30, 30 – x would be – ve which is not
A =The set of students who drink apple juice and possible.
P =The set of students who drink pineapple juice Total number of students playing at least one game,
We are given that n(A) = 150, n(P) = 250 and = 100 + x – I5 + 35 – x + x – 20
n(A  P) = 100. Then = 100 + x
n(A  P) = n(A) + n(P)– n(A  P) So, the minimum number of students playing at least
= 150 + 250–100 = 300 one game = I 00 + 20 = 120
Let S be the set of all students in the school, then S is Hence, the maximum number of students playing at
the universal set containing A and P. We are given that least one game = I 00 + 30 = 130.
n(S) = 600. Now,
n[S– (A  P)] = n(S)– n(A  P) 12. In a group of 1000 people, there are 750 who can
= 600 – 300 = 300 speak Hindi and 400 who can speak Bengali. How
Therefore 300 students drink neither apple juice nor many can speak Hindi only? How many can speak
pineapple juice. Bengali only? How many can speak both Hindi and
Bengali?
10. Let X1, X2, …., X30 be 30 sets each with five elements Solution : Let A and B be the sets of persons who can
and Y1, Y2, ...., Ym be m sets each with 3 elements. speak Hindi and Bengali respectively.
Let then n(A  B) = 1000, n(A) = 750, n(B) = 400.
30 m Number of persons who can speak both Hindi and
X1 = Yj = S Bengali
i=1 j=1
= n(A  B) = n(A) + n(B)– n(A  B)
Suppose that each element of S belongs to exactly = 750 + 400– 1000 = 150
10 of Xi's and exactly 9 of Yj's. Then find m. Number of persons who can speak Hindi only
Solution : Let n(S) = s. Since each element of S belongs = n(A – B)= n(A)– n(A  B)= 750– 150 = 600
to exactly 10 of Xi's, so Number of persons who can speak Bengali only
30
= n(B– A)= n(B) –n(A  B)= 400– 150 = 250
 n(X ) = 10s
i=1
i

Since each Xi contains 5 elements, therefore 13. Let A and B be two finite sets such that n(A – B) = 15,
30 n(A  B) = 90, n(A  B) = 30. Find n(B)
 n(X ) = 30  5 = 150
i=1
i Solution : n(A  B) = n(A – B) + n(B – A) + n(A  B)
90 = 15 + n(B – A) + 30
Therefore, 10 s = 150 and hence s = 15. Similarly
n(B – A) = 45
10 Chapter 1 | Fundamentals of Mathematics
n(B) = n(B – A) + n(A  B) n(B) = 45 + 30 = 75

LECTURE-7 DEFINATION OF RELATION OF FUNCTION

This lecturer deals with establishing binary Example 7.2


relation between elements of one set and
elements of another set according to some If A = {2, 4} and B = {3, 4, 5}, then (A  B) × (A  B)
particular rule of relationship. is
(1) {(2, 2), (3, 4), (4, 2), (5, 4)}
(2) {(2, 3), (4, 3), (4, 5)}
ORDERED PAIRS (3) {(2, 4), (3, 4), (4, 4), (4, 5)}
A pair of elements written in a particular order is (4) {(4, 2), (4, 3), (4, 4), (4, 5)}
called an ordered pair. It is written by listing its two Solution : A  B = {4} and A  B = {2, 3, 4, 5}
elements in a particular order, separated by a comma  (A  B) × (A  B) = {(4, 2), (4, 3), (4, 4), (4, 5)}
and enclosing the pair in brackets. In the ordered pair
(L, x), L is called the first component or the first Example 7.3
coordinate and x is called the second component or
Let A= {a, b, c} and B = {1, 2}. Then
the second coordinate.
Solution : A × B = {(a, 1), (a, 2), (b, 1), (b, 2), (c, 1), (c, 2)}
and B × A= {(1, a), (2, a), (1, b), (2, b), (1, c), (2, c)}
The ordered pairs (3, 4) and (4, 3) are different even
though they consist of same pair of elements; for
example, these represent different points in the Example 7.4
Euclidean plane. If A= {x, y, z} and B = {a}, then
Solution : A × B = {(x, a), (y, a), (z, a)}
CARTESIAN PRODUCT : and B × A= {(a, x), (a, y), (a, z)}
The Cartesian product of two sets A, B is a non-void
set of all ordered pair (a, b), where a  A and b  B. Example 7.5
This is denoted by A × B
 A × B = {(a, b)  a  A and b  B} If A= {a, b, c}, then
e.g. A = {1, 2}, B = {a, b} Solution : A2 = {(a, a), (a, b), (a, c), (b, a), (b, b), (b, c),
A × B = {(1, a), (1, b) (2, a) (2, b)} (c, a), (c, b), (c, c)}

Note Example 7.6


1. A × B  B × A (Non-commutative) If A = {1, 2}, then
2. n(A × B) = n(A) n(B) and n(P(A × B)) = Solution : A3 = {(1, 1, 1), (1, 1, 2), (1, 2, 1), (1, 2, 2),
2n(A)n(B) (2,1,1), (2,1,2), (2,2,1), (2,2,2)}
3. A =  and B =   A × B = 
4. If A and B are two non-empty sets having n
Example 7.7
elements in common, then (A × B) and (B × A)
have n2 elements in common If A= {a, b, c, d) and B = {1, 2, 3), then
5. A × (B  C) = (A × B)  (A × C) Solution : n(A × B)= n(A) × n(B) = 4 × 3 = 12
6. A × (B  C) = (A × B)  (A × C)
7. A × (B – C) = (A × B) – (A × C) Example 7.8
If A= {a, b, c, d}, then
Solution : n(A2) = n(A)2 = 42 = 16
Example 7.1
and n(A3) = n(A)3 = 43 = 64
If n(A) = 7, n(B) = 8 and n(A  B) = 4, then match the
following columns. CARTESIAN PRODUCT OF THREE SETS
(i) n(A  B) (a) 56 If A, B, C are three non -empty sets then the
(ii) n(A × B) (b) 16 cartesian product of three sets is the set of all
(iii) n(B × A) × A) (c) 392 possible ordered triplets given by
(iv) n((A × B)  (B × A)) (d) 96 A × B × C = {(a, b, c) for all a  A, b  B, c  C}
(v) n((A × B)  (B × A)) (e) 11 Illustration for Geometrical understanding of
Solution : (i) n(A  B) = n(A) + n(B) – n(A  B) = 7 cartesian product of two and three sets
+ 8 – 4 = 11 Let A= {0,1}, B = {0,1}, C = {0, 1}
(ii) n(A × B) = n(A) n(B) = 7 × 8 = 56 = n(B × A) A × B = {0,1} × {0,1} = {(0,0),(0,1) ,(1,0)
(iii) n((B × A) × A) = n(B × A).n(A) = 56 ×7 = 392 (1,1)}
(iv) n((A × B)(B × A)) = (n(A  B))2 = 42 = 16 Representing A × B in the xy-plane we get a
(v) n((A × B)  (B × A)) = n(A × B) + n(B × A) – picture shown in Figure
n((A × B)  (B × A)) = 56 + 56 – 16 = 96
Chapter 1 | Fundamentals of Mathematics 9

RELATION :
Every subset of A × B defined a relation from set
A to set B.
If R is relation from A → B
R : {(a, b) | (a, b)  A ×B and aRb}

(A × B) × C = {(0,0), (0,1), (1,0), (1,1)} × {0, l} Highlights :


= {(0,0, 0), (0,0,1), (0,1,0), (0,1,1), Let A and B be two non-empty sets and R : A → B
(1,0, 0), (1,0,1)(1,1,0), (1,1 1)} be a relation such that R : {(a, b) | (a, b)  R, a  A
Representing A × B × C in the xyz-plane we get a and b  B}.
picture as shown in Figure (i) 'b' is called image of 'a' under R.
(ii) 'a' is called pre–image of 'b' under R.
Thus, A × B represent vertices of a square in two
dimensions and A × B × C represent vertices of a DOMAIN, CO-DOMAIN & RANGE OF A FUNCTION
cube in three dimensions. Let R be a relation from A to B. Then the domain
of R is defined as the set of all first components of
the ordered pairs belonging to R and is denoted by
Dom (R). Mathematically,
Dom(R) = {a | (a, b)  R for some b  B}
Note that Dom(R) is a subset of A and that Dom(R)
is non-empty if and only if R is non-empty.

Let R be a relation from A to B. Then the range of


R is defined as the set of all second components of
Note the ordered pairs belonging to R and is denoted by
In general, cartesian product of two non-empty Range(R). Mathematically,
sets provides a shape in two dimensions and Range(R) = {b | (a, b)  R for some a  A}
cartesian product of three non-empty sets provide Note that Range(R) is a subset of Band that it is
an object in three dimensions. non-empty if and only if R is non-empty.

Example 7.9 Note


1. It is not necessary that each and every element of
Let A = {x  N | 1< x < 4}, B = {x  W | 0 ≤ x < 2} and set A has a image in Set B and each and every
C = {x  N | x < 3}. element of set B has preimage in Set A.
Then verify that 2. Elements of set A having image in B is not
(i) A × (B  C) = (A × B)  (A × C) necessarily unique.
(ii) A × (B  C) = (A × B)  (A × C) 3. Basically relation is the number of subsets of A
Solution : A = {x  N | 1 < x < 4} = {2, 3}, B = {x  W | ×B
0 ≤ x < 2} = {0,1}, C = {x  N | x < 3} = {1,2} Number of non empty relations = no. of ways of
(i) A × (B  C) = (A × B)  (A × C) selecting a non zero subset of A × B
B  C = {0, 1}  {1,2} = {0, 1,2} = mnC1 + mnC2 + …... + mnCmn = 2mn – 1
A × (B  C) = {2,3} × {0,1,2} = {(2,0), (2,1), (2,2), Total number of relation = 2mn (including void
(3,0), (3,1), (3,2)} ... (1) relation)
A × B = {2,3} × {0, l} = {(2,0),(2,1),(3,0),(3,1)}
A × C = {2,3} × {1,2} = {(2,1),(2,2),(3,1),(3,2)}
(A × B)  (A × C) = {(2,0),(2, 1),(3,0),(3, I)}  Example 7.10
{(2,1),(2,2),(3, 1),(3,2)} A = {1, 2, 3, 4, 5} and B = {2, 4, 5}
= {(2,0),(2,1),(2,2),(3,0),(3,1),(3,2)} ... (2) aRb  a and b are relatively prime or co-prime (i.e.
From (1) and (2), A × (B  C) = (A × B)  (A × HCF is 1)
C) is verified. Solution : R = {(1, 2), (1, 4), (1, 5), (2, 5), (3, 2), (3, 4),
(ii) A ×(B  C) = (A × B)(A ×C) (3, 5), (4, 5), (5, 2), (5, 4)})]
B  C = {0,1}  {1,2} = {1} Domain of R{1, 2, 3, 4, 5,}
A ×(B  C) = {2,3} × {1} = {(2,1),(3,1)} ... (3) Range of R{2, 4, 5}
A × B = {2,3} × {0,1} = {(2,0),(2,1),(3,0),(3,1)}
A × C = {2,3} × {1,2} = {(2,1),(2,2),(3,1),(3,2)} Example 7.11
(A × B)  (A × C) = {(2, 0),(2,1) ,(3,0),(3,1)} 
{(2,1),(2,2),(3,1),(3,2)} A = {Jaipur, Patna, Kanpur, Lucknow} and B =
= {(2, 1), (3, 1)} ... (4) {Rajasthan, Uttar Pradesh, Bihar}
aRb  a is capital of b, a  A and b  B
From (3) and (4), A × (B  C)= (A × B)  (A × C) Solution : R = {(Jaipur, Rajasthan), (Patna, Bihar),
is verified. (Lucknow, Uttar Pradesh)}
10 Chapter 1 | Fundamentals of Mathematics
number is nonnegative and every nonnegative
Example 7.12 number y is the square of its own square root, y =
If A = {1, 3, 5, 7}, B = {2, 4, 6, 8} ( y )2 for y ≥ 0.
Relation is aRb  a > b, a  A, b  B The formula y = 1/x gives a real y-value for every x
Solution : R = {(3, 2), (5, 2), (5, 4), (7, 2), (7, 4), (7, 6)} except x = 0. For consistency in the rules of arithmetic,
Domain = {3, 5, 7} we cannot divide any number by zero. The range of y
Range = {2, 4, 6} = 1/x, the set of reciprocals of all nonzero real
numbers, is the set of all nonzero real numbers, since
Note y = 1/(1/y). That is, for y  0 the number x = 1/y is
1. The complete set of all positive real numbers the input assigned to the output value y.
is denoted by R+. The formula y = x gives a real y-value only if x ≥ 0.
2. The complete set of all negative real numbers
is denoted by R–. The range of y = x is [0, ) because every
3. The complete set of all real numbers other nonnegative number is some number's square root
than zero is denoted by R0. (namely, it is the square root of its own square).
4. The complete set of all integers is denoted by Z. In y = 4 − x , the quantity 4 – x cannot be negative.
That is, 4 – x ≥ 0, or x ≤ 4. The formula gives real y-
values for all x ≤ 4. The range of 4 − x is [0, ), the
Example 7.13 set of all nonnegative numbers.
Let A = {1, 2, 3, 4}, B = {a, b, c, d, e}, and R = {(1, a), The formula y = 1 − x2 gives a real y-value for every
(2, c), (3, a), (2, a)}. Then x in the closed interval from –1 to 1. Outside this
Solution : Dom(R) = {1, 2, 3} and Range(R) = {a, c} domain, 1 – x2 is negative and its square root is not a
real number. The values of 1 – x2 vary from 0 to 1 on
Example 7.14 the given domain, and the square roots of these values
do the same. The range of 1 − x is [0, 1].
2
Let A = {2, 3, 4}, B = {2, 3, 4, 5, 6, 7, 8} and R = {(a, b)
 A × B | a divides b}. Then
Solution : R = {(2, 2), (2, 4), (2, 6), (2, 8), (3, 3), (3, 6),
(4, 4), (4, 8)} INVERSE RELATION
Dom(R) = {2, 3, 4} and Range(R) = {2, 4, 6, 8, 3} If relation R is defined from A to B, then the
inverse relation would be defined from B to A, i.e.
Example 7.15 R : A → B  aRb where a  A, b  B
R–1 : B → A  bRa where a  A, b  B
Let R = {(a, b)  Z+ × Z+ | 2a = b}. Then R is a relation Domain of R = Range of R–1
from Z+ to Z+ and is given by and Range of R = Domain of R–1
Solution : R = {(a, 2a) | a is a positive integer}  R–1 = {(b, a) | (a, b)  R}
Then A relation R is defined on the set of 1st ten natural
Dom(R) = Z+ numbers.
and Range(R) =The set of all positive even integers

Example 7.16
e.g. N is a set of first 10 natural nos.
Let's verify the natural domains and associated ranges  N = {1, 2, 3, ….., 10} & a, b  N
of some simple functions. The domains in each case aRb  a + 2b = 10
are the values of x for which the formula makes sense. R = {(2, 4), (4, 3), (6, 2), (8, 1)}
Function Domain (x) Range (y) R–1 = {(4, 2), (3, 4), (2, 6), (1, 8)}
y = x2
(–, ) [0, )
y = 1/x IDENTITY RELATION :
(–, 0)  (0, ) (–, 0)  (0, )
y= x A relation defined on a set A is said to be an
[0, ) [0, )
y = 4−x identity relation if each & every element of A is
(–, 4] [0, ) related to itself & only to itself.
y=
[–1, 1] [0, 1] e.g. A relation defined on the set of natural
1− x2

Solution : The formula y = x2 gives a real y-value for numbers is


any real number x, so the domain is (–, ). The range aRb  a = b where a & b  N
of y = x2 is [0, ) because the square of any real R = {(1, 1), (2, 2), (3, 3), ……}
R is an identity relation

WORKED-OUT PROBLEMS - 7
(iii) Is A × B = B × A ?
1. Let A = {1, 2, 3, 4} and B = {5, 7, 9}. Determine (iv) Is n (A × B) = n (B × A) ?
(i) A × B (ii) B × A
Chapter 1 | Fundamentals of Mathematics 9
Solution : Since A = {1, 2, 3, 4} and B = {5, 7, 9}.
Therefore,
(i) A × B = {(1, 5), (1, 7), (1, 9), (2, 5), (2, 7), (2, 9), (3,
5), (3, 7), (3, 9), (4, 5), (4, 7), (4, 9)}
(ii) B × A = {(5, 1), (5, 2), (5, 3), (5, 4), (7, 1), (7, 2),
(7, 3), (7, 4), (9, 1), (9, 2), (9, 3), (9, 4)} Solution :
(iii) No (i) Set builder form of R = {(x, y) | y = x – 2, x  P,
(iv) No y  Q}
(ii) Roster form R = {(5, 3), (6, 4), (7, 5)}
2. Find x and y if: (iii) Domain of R = {5,6,7} and range of R = {3,4,5}
(i) (4x + 3, y) = (3x + 5, – 2)
(ii) (x – y, x + y) = (6, 10) 7. A Relation R is given by the set {(x, y)/y = x + 3, x
Solution : (i) x = 2, y = – 2 (ii) x = 8, y = 2  {0, 1, 2, 3, 4, 5}}. Determine its domain and
range.
3. If A and B are sets such that n(A × B) = 6 and A × Solution : {0, 1,2 ,3 ,4 ,5}, {3, 4, 5, 6, 7, 8}
B contains (1, 2), (2, 1) and (3, 2), then find the
sets A, B and A × B. 8. Find the domain of following functions :
Solution : Since n(A) · n(B) = n(A × B) = 6, n(A) and
n(B) are divisors of 6. Hence n(A) = 1 or 2 or 3 or 6. y = 5 − 2x
Since (1, 2), (2, 1) and (3, 2)  A × B, 1, 2, 3  A and Solution : 5 – 2x ≥ 0
hence n(A) ≥ 3. Also, 2, 1 EB and hence n(B) ≥ 2.  Domain is (–, 5/2]
Thus n(A) = 3 and n(B) = 2. Therefore, A= {1, 2, 3}
and B = {1, 2}, so that 9. Find the domain of functions :
A × B = {(1, 1), (1, 2), (2, 1), (2, 2), (3, 1), (3, 2)} −1
f(x) =
x +3
4. If A = {1,3,5} and B = {2,3} then Solution : (–, –3)  (–3, +)
(i) find A × B and B × A.
(ii) Is A × B = B × A? If not why? 10. Find the domain of functions :
(iii) Show that n(A × B) = n(B × A) = n(A) × n(B)
Solution : Given that A = {1,3 ,5} and B = {2,3} f(x) = ( − x + 9)
(i) A × E= {1,3,5} × {2,3} = {(1,2), (1,3), (3,2), (3,3), Solution : –x + 9  0 = (–, 9]
(5,2), (5,3)} ... (1)
B × A= {2,3} × {1,3,5} = {(2,1), (2,3), (2,5), (3,1), 11. Find the Range of function :
(3,3), (3,5)} ... (2) f(x) = x2 + 3
(ii) From(1) and(2) we conclude that A × B  B × A as Solution : y = x2 + 3
(1, 2)  (2,1) and (1,3)  (3,1), etc.  0 − 12 
D
(iii) n(A) = 3; n (B) = 2. ymin = − = − = 3
From (1) and (2) we observe that, n (A × B) = n 4a  4 
(B × A) = 6; [3, +)
we see that, n (A) × n (B) = 3 × 2 = 6 and n (B) ×
n (A)= 2 × 3 = 6 12. If (2x + y, 7) = (5, y – 3) then find x and y.
Hence, n (A × E) =n (B × A) = n(A) × n (B)= 6. 5
Thus, n (A × E) =n (B × A) = n(A) × n (B). Solution : x = − , y = 10
2
5. If A × B = {(3,2), (3,4), (5,2), (5,4)} then find A 13. If A × B = {(1, 2), (1, 3), (1, 6), (7, 2), (7, 3), (7, 6)}
and B. then find sets A and B.
Solution : A × B = {(3,2), (3,4), (5,2), (5,4)} Solution : A= {1, 7}, B = {2, 3, 6}
We have A = {set of all first coordinates of elements of
A × B}. Therefore, A= {3,5} 14. If A= {x, y, z} and B = {1, 2} then find number of
B = {set of all second coordinates of elements of A × B}. relations from A to B.
Therefore, B = {2,4} Solution : 64
Thus A= {3,5} and B = {2,4}.
15. Write R= {(4x + 3, 1 – x) : x ≤ 2, x  N}
6. The arrow diagram shows (Figure) a relationship Solution : {(7, 0), (11, –1)}
between the sets P and Q. Write the relation in (i)
Set builder form (ii) Roster form (iii) What is the
domain and range of R.
8 Chapter 1 | Fundamentals of Mathematics

LECTURE-8 CLASSIFICATION OF RELATIONS

DEFINITION Only Reflexive R = {(x, y) | x = y, or x – y = 1, x 


Types of Relations N, y  N}

EQUIVALENCE RELATION :
Reflexive Symmetric Transitive
If a relation is Reflexive, Symmetric and
(i) Reflexive : A relation R on a set A is said to be
Transitive, then it is said to and equivalence
reflexive if every element of A is related to itself.
relation.
i.e. if (a, b)  R, then (a, a)  R. However, if there Examples :
is a single ordered pair of (a, b)  R such (a, a)  (1) A relation defined on N
R, then R is not reflexive. xRy  x = y
e.g. A relation defined on (set of natural number) R is an equivalence relation
aRb  'a' divides 'b' a, b  N (2) A relation defined on a set of | | lines in a plane
R would always contain (a, a) because every aRb  a | | b
natural number divides itself and hence it is a It is an equivalence relation
reflexive relation. (3) Relation defined on the set of integer (I)
Prove that : xRy  (x – y) is even is an equivalence
Note relation.
Every identity relation is a reflexive relation but (4) R = {(1, 2,), (2, 3)} add minimum number of
every reflexive relation need not be an Identity. ordered pairs to make it an equivalence relation.
{(1, 1), (2, 2), (3, 3), (2, 1), (3, 2), (1, 3), (3, 1)} = 7
(ii) Symmetric : A relation defined on a set is said (5) A = {1, 2, 3, ………. 13, 14}
to be symmetric if aRb  bRa. R = {(x, y) | 3x – y = 10} R S T
If (a, b)  R, then (b, a) must be necessarily there R = {(x, y) | x is coefficient of y} R  S  T
in the same relation. R = {(x, y) | x is father of y} R  S  T

Examples : PARTIAL ORDER RELATION :


A relation defined on the set of lines. Definition :
(1) aRb  a | | b A relation R on a set P is called partial order
It is a symmetric relation because if line 'a' is | | to relation if it is reflexive, antisymmetric and
'b' then the line 'b' is | | 'a'. transitive. That means that for all x, y and z in P we
where (a, b)  L {L is set of | | lines} have :
(2) L1 R L2  L1 ⊥ L2 It is a symmetric • x R x;
relation • if x R y and y R x, then x = y;
L1, L2  L {L is a set of lines} • if x R y and y R z, then x R z.
(3) aRb  'a' is brother of 'b' is not a symmetric
relation as 'b' may be sister of 'a' Example :
(4) aRb  'a' is a cousin of 'b'. This is symmetric • The identity relation I on a set P partial order
relation. relation.
If R is symmetric • On the set of real numbers R the relation  is
(1) R = R – 1 partial order relation.
(2) Range of R = Domain of R – 1 • The relation "is a divisor of " defines partial
order on the set of natural numbers N.
(iii) Transitive : A relation on set A is said to be a
transitive if aRb and bRc implies aRc Example 8.1
i.e. (a, b)  R and (b, c)  R, then (a, c)  R and a, Which of the following are identity relations on set A
b, c need not be distinct. = {1, 2, 3}.
R1 = {{1, 1), (2, 2)}, R2 = {(1, 1), (2, 2), (3, 3), (1, 3)},
Examples : R3 = {(1, 1}, (2, 2), (3, 3)).
(1) A relation R defined on a set of natural Solution : The relation R3 is identity relation on set A.
numbers N with rule aRb  a  b R1 is not identity relation on set A as (3, 3)  R1.
Let R : {(1, 2), (1, 1)}. on set {1, 2} R2 is not identity relation on set A as (1, 3)  R2
In this relation a, b, c are not distinct but it is
transitive. It is transitive but not symmetric as (2,
1) is missing. Minimum number of ordered pair Example 8.2
that must be added to make it reflexive, Which of the following are reflexive relations on set A
symmetric and transitive is 2 i.e. (2, 1) and (2, 2). = {1, 2, 3}.
(2) Only Transitive R = {(x, y) | x < y, x  N, y  N} R1 = {(1, 1), (2, 2), (3, 3), (1, 3), (2, 1)}, R2 = {(1, 1), (3,
Only Symmetric R = {(x, y) | x + y = 10, x  N, y  N} 3), (2, 1), (3, 2))..
Solution : R1 is a reflexive relation on set A.
Chapter 1 | Fundamentals of Mathematics 9
R2 is not a reflexive relation on A because 2  A but (2, Hence R is symmetric
2)  R2. (iii) Let (T1, T2)  R and (T2, T3)  R  T1 is congruent
to T2 and T2 is congruent to T3
Example 8.3  T1 is congruent to T2  (T1, T3)  R
 R is transitive
Prove that on the set N of natural numbers, the Hence R is an equivalence relation
relation R defined by x R y  x is less than y is
transitive.
Example 8.5
Solution : Because for any x, y, z  N x < y and y < Z 
x < z  x R y and y R z  x R z. so R is transitive. Show that the relation R in R defined as R = {(a, b) : a
!> b) is transitive.
Example 8.4 Solution : Let (a, b)  R and (b, c)  R
 (a  b) and b  c  a  c  (a, c)  R Hence R is
Let T be the set of all triangles in a plane with R a transitive.
relation in T given by R = {(T1, T2) : T1 is congruent to
T2}. Show that R is an equivalence relation.
Solution : Since a relation R in T is said to be an
Example 8.6
equivalence relation if R is reflexive. symmetric and Show that the relation R in the set {1 , 2, 3} given by R
transitive. = {(1, 2), (2, 1)} is symmetric.
(i) Since every triangle is congruent to itself Solution : Let (a, b)  R  (1, 2)  R]
 R is reflexive  (b, a)  R [ (2, 1)  R]
(ii) (T1, T2)  R  T1 is congruent to T2  T2 is Hence R is symmetric.
congruent to T1  (T2, T1)  R

WORKED-OUT PROBLEMS - 8
1. Determine whether each of the following (iv) symmetric but neither reflexive nor
relations are reflexive, symmetric and transitive: transitive.
(i) Relation R in the set A = {1, 2, 3, ..., 13, 14} (v) transitive but neither reflexive nor symmetric.
defined as (vi) reflexive but neither symmetric nor
R = {(x, y) : 3x – y = 0} transitive.
(ii) Relation R in the set N of natural numbers (vii) neither reflexive nor symmetric nor
defined as transitive.
R = {(x, y) : y = x + 5 and x < 4} (viii) an equivalent relation.
(iii) Relation R in the set A = {1, 2, 3, 4, 5, 6} as Solution : (i) Consider the relation R = {(a, a), (a, b),
R = {(x, y) : y is divisible by x} (b, a), (b, c), (c, b), (b, b), (c, c)} on A.
(iv) Relation R in the set Z of all integers defined Here, R is relation which is reflexive and symmetric
as but not transitive.
R = {(x, y) : x – y is an integer} ( (a, b)  R and (b, c)  R but (a, c)  R)
(v) Relation R in the set A of human beings in a
(ii) Consider the relation R = {(a, a), (a, b), (b, a), (b,
town at a particular time given by
b) on A.
(a) R = {(x, y) : x and y work at the same place}
Here, R is symmetric and transitive but not reflexive
(b) R = {(x, y) : x and y live in the same locality}
(c) R = {(x, y) : x is exactly 7 cm taller than y} ( (c, c)  R)
(d) R = {(x, y) : x is wife of y} (iii) Consider the relation and transitive R = {(a, a), (b,
(e) R = {(x, y) : x is father of y} b), (c, c), (a, b)} on A.
Solution : (i) Neither reflexive nor symmetric nor Here, R is reflexive and transitive but not symmetric.
transitive. ( (a, b)  R but (b, a)  R)
(ii) Neither reflexive nor symmetric but transitive. (iv) Consider the relation R = {(a, b), (b, a)} on A.
(iii) Reflexive and transitive but not symmetric. Here, R is symmetric.
(iv) Reflexive, symmetric and transitive.
R is not reflexive as (a, a)  R.
(v) (a) Reflexive, symmetric and transitive.
Also, R is not transitive as (a, b)  R (b, a)  R but (a,
(b) Reflexive, symmetric and transitive.
a)  R.
(c) Neither reflexive nor symmetric nor transitive.
(v) Consider the relation R = {(a, b), (b, c), (a, c)} on A.
(d) Neither reflexive nor symmetric but transitive.
Here, R is transitive.
(e) Neither reflexive nor symmetric nor transitive.
R is not reflexive as (a, a)  R.
2. Consider the set A = {a, b, c}. Given an example of Also, R is not symmetric as (a, b)  R but (b, a)  R.
a relation R on A which is (vi) Consider the relation R = {(a, a), (b, b), (c, c), (a,
(i) reflexive and symmetric but not transitive. b), (b, c)} on A.
(ii) symmetric and transitive but not reflexive. Here, R is reflexive.
(iii) reflexive and transitive but not symmetric. R is not symmetric as (a, b)  R but (b, a)  R
Also, R is not transitive as (a, b)  R and (b, c)  R but
(a, c)  R.
8 Chapter 1 | Fundamentals of Mathematics
(vii) Consider the relation R = {(a, b), (b, c)} on A. So a R b does not imply b R a.
Here, R is not reflexive as (a, a)  R  R is not symmetric.
R is not symmetric as (a, b)  R but (b, a)  R Transitivity : For a, b, c  A, a R b and b R c  b = a +
Also, R is not transitive as (a, b)  R and (b, c)  R but 1 and c = b + 1
(a, c)  R. c=a+1+1c=a+2ca+1a R c
(viii) Consider the relation R = {(a, a), (b, b), (c, c)} so a R b and b R c  a R c
then R is reflexive as (x, x)  R for all x {a, b, c}.  R is not transitive
Also, R is symmetric. ( there do not exist x, y {a, b, (ii) Reflexivity : As a  a3 is not true for all a  R,
c} s.t. (x, y)  R but (y, x)  R)  1 
Further, R is transitive also. therefore, a R a for all a  R  for a= ,a  a3 
 2 
(Q there do not exist x, y, z  {a, b, c} s.t. (x, y)  R and
 R is not reflexive.
(y, z)  R but (x, y)  R)
Symmetric : For a, b  R, a  b3 need not imply b  a3,
Infect, R is the smallest equivalence relation on A ; it is
i.e., a r b need not imply b R a.
the identity relation on A.
As an example, we have 1  23 but 2  13, i.e., 1 R 2 but
3. (i) For the set A = {1, 2, 3}, define a relation R on 2 R 1.
the set A as follows : Transitivity : For a, b, c  R, a  a3 and b  c3 need not
R = {(1, 1), (2, 2), (3, 3), (1, 3}. imply a  c3, therefore, a R b and b R c   a R c.
Write the ordered pairs to be added to R to make As an example, we find that 100 R 5 and 5 R 2 but 100
it the smallest equivalence relation. R 2 ( 100  53 and 5  22, but 100 100  23)
(ii) Let A = {a, b, c} and R be the relation defined
on A as follows :  R is not transitive
R = {(a, a), (b, c), (a, b)}. (iii) Reflexivity : As no person can be 7 cm taller than
Write minimum number of ordered pairs to be himself (herself), therefore, x R x for x  A.
added to R to make R reflexive and transitive.  R is not reflexive.
Solution : (i) Here, A = {1, 2, 3} and the relation R = Symmetric : For x, y  A, x R y  x is exactly 7 cm taller
{(1, 1), (2, 2,), (3, 3), (1, 3)}. than y   y is exactly 7 cm taller than x
Clearly, R is reflexive but no symmetric as (1, 3)  R So, x R y  yRx
but (3, 1)  R.  R is not symmetric
We shall include (3, 1) to the above relation to make it Transitivity : For x, y, z  A, x R y and y R z
smallest equivalence relation  x is exactly 7 cm taller than y and y is exactly 7 cm
R' = {(1, 1), (2, 2), (3, 3), (1, 3), (3, 1)}. taller than z
R' is certainly transitive as transitivity is not
 x is exactly 14 cm taller than z  x R z
contradicted.
 R is not transitive.
( there do not exist x, y, z  A s.t. (x, y)  R and (y, z)
 R but (x, y)  R) 5. (i) Let L be the set of all lines in a plane and R be
(ii) Here, A = {a, b, c} and R = {(a, a), (b, c), (a, b)}, the relation on L defined as
which is nether reflexive not transitive. R = {(, m) :  is perpendicular to m}.
For reflexivity, (b, b), and (c, c) has to be included and
for transitivity (a, c) should be included. Check whether R is reflexive, symmetric or
transitive.
( (a, b)  R and (b, c)  R)
(ii) Show that the relation R defined in the set A or
Hence, the required ordered pairs are (b, b), (c, c) and all triangles as R = {(S, T) : S is similar to T}, is an
(a, c). equivalence relation. Consider three right angle
triangles T1 with sides 3, 4, 5; T2 with sides 5, 12,
4. (i) Check whether the relation R defined on the set 13 and T3 with sides 6, 8, 10. Which triangles
A = R of reals as R = {(a, b) : b = a + 1} is reflexive, among T1, T2 and T3 are related.
symmetric or transitive. Solution : (i) Reflexivity : As a line cannot be
(ii) Check whether the relation R on the set R of perpendicular to itself, therefore,  R  for any   L
reals is reflexive, symmetric or transitive, where R
= {(a, b) : a  b3}.  R is not reflexive.
(iii) Let A be the set of human beings living in a Symmetric : For , m  L  ⊥ m , therefore,  R m  m R 
town at a particular time and R be the relation on So R is symmetric.
A defined by R = {(x, y) : x is exactly 7 cm taller Transitive : For 1, 2, 3  L, 1 R 2 and 2 R 3  1 ⊥
then y.}
Check whether the relation R is reflexive, 2 and 2 ⊥ 3
symmetric or transitive on A.  1 || 3  1 R 3
Solution : (i) Reflexivity : As a  a + 1, therefore, (a, a)
 R for any a  A. So 1 R 2 and 2 R 3 
 1 R 3
 R is not reflexive.  R is not transitive.
Symmetric : For a, b  A, a R b  b = a + 1  a = b + 1 (ii) Reflexivity : As every triangle is similar to itself,
therefore, T R T for all T  A  R is reflexive.
Chapter 1 | Fundamentals of Mathematics 9
Symmetric : For T1, T2  A, T1 R T2  T1 is similar to T2
 T2 is similar to T1  T2 R T1 7. Let A = {1, 2, 3, ....., 45} and R be the relation ‘is
Hence T1 R T2  T2 R T1 square of element’ on A. Which of the following is
 R is symmetric. false?
Transitive : For T1, T2, T3  A (A) R = {(1, 1), (4, 2), (9, 3), (16, 4), (25, 5), (36,
T1 R T2 and T2 R T3  T1 is similar to T2 and T2 is 6)}
similar to T3  T1 is similar to T3  T1 R T3 (B) Domain of R = (1, 4, 9, 16, 25, 36)
Thus, T1 R T2 and T2 R T3  T1 R T3  R is transitive. (C) Range of R = {1, 2, 3, 4, 5, 6}
So, R is an equivalence relation on the set A. (D) At least one is false.
We know that two triangles are similar if the ratio of Solution : We have (1)2 = 1, (2)2 = 4, (3)2 = 9, (4)2 =
corresponding sides is same. 16, (5)2 = 25, (6)2 = 36
 R = {(1, 1), (4, 2), (9, 3), (16, 4), (25, 5), (36, 6)}
 3 4 5 
As 3 : 4 : 5 : : 6 : 8 : 10  = =  Domain of R = {x ; (x, y)  R} = {1, 4, 9, 16, 25, 36,}
 6 8 10  Range of R = {y ; (x, y)  R} = {1, 2, 3, 4, 5, 6}
therefore, T1 and T3 are similar triangles.  (A), (B), (C) are true
 The correct answer is (D)
6. Let S be the set of all real numbers. Then the
relation R = {(a, b) : 1 + ab > 0} on S is 8. Let a relation R on the set N of natural numbers be
(A) An equivalence relations defined as (x, y)  R if and only if x2 – 4xy + 3y2 = 0
(B) Reflexive but not symmetric for all x, y  N. The relation R is
(C) Reflexive and transitive (A) Reflexive
(D) Reflexive and symmetric but not transitive (B) Symmetric
Solution : R = {(a, b ; 1 + ab > 0)} (C) Transitive
Reflexive : for (a, a) (D) Equivalence Relation
1 + a2 > 0  Reflexive Solution : We have R = {x, y) : x2 – 4xy + 3y2 = 0, x, y  N}
Symmetric : if (a, b) then (b, a) is also present Let x  N. x2 – 4xy + 3x2 = 4x2 – 4x2 = 0  (x, x)  R
for (a, b) 1 + ab > 0  R is reflexive
then (b, a) 1 + ba > 0 is also true We have (C)2 – 4(C)(1) + 3(1)2 = 9 – 12 + 3 = 0
so it is symmetric
 (3, 1)  R
for transitive if (a, b) and (b, c) true then (a, b) is also
Also (1)2 – 4(1) (C) + 3(C)2 = 81 – 108 + 27 = 0
present
 (1, 3)  R  R is not symmetric
1 + ab > 0 & 1 + bc > 0
(9, 3)  R because
then it is not necessary that 1 + ac > 0
(9)2 – 4(9)(C) + 3(1)2 = 9 – 12 + 3 = 0
1 Now (9, 1)  R if (9)2 – 4(9)(1) + 3(1)2 = 0 if 81 – 36
for example a = 1, b = − , c = –2
3 + 3 = 48  0, which is not so
1 + ab > 0 1 + bc > 0  (9, 3), (3, 1)  R and (9, 1)  R  R is not transitive
but 1 + ac < 0
hence not transitive
LECTURE-9 WAVY CURVE

Intervals :
Intervals are basically subsets of R and are commonly used in solving inequalities or in finding domains. If there
are two numbers a, b  R such that a < b, we can define four types of intervals as follows:
Name Representation Description
Open Interval (a, b) {x : a < x < b} i.e. end points are not included.
Close Interval [a, b] {x : a  x  b} i.e. end points are also included. This is possible only
when both a and bare finite.
Open- Closed Interval (a b] {x: a < x  b} i.e. a is excluded and b is included.
Close- Open Interval [a, b) {x : a  x < b} i.e. a is included and b is excluded.
Example 9.1

Solution :

Example 9.2

Solution :

Example 9.3

Solution :

Example 9.4

Solution :
Example 9.5

Example 9.6

Example 9.7
Example 9.8

Example 9.9

Example 9.10

Example 9.11

Example 9.12
Example 9.13

Example 9.14

Example 9.15

Example 9.16
Example 9.17

Example 9.18

WORKED-OUT PROBLEMS - 9

1. Solve the following inequations


(x – 1)2  0
Solution :
– +
– 1 +
x1
2. Solve the following inequations

3. Solve the following inequations

4. Solve following Inequalities over the set of real numbers - (i) x

Solution :

5. Solve following Inequalities over the set of real numbers - (i) x

Solution :

6. Solve following Inequalities over the set of real numbers - (i) x

Solution :
Solution :

7. Solve following Inequalities over the set of real numbers - (i) x

Solution :

8. Solve following Inequalities over the set of real numbers - (i) x

Solution :

9. Solve following Inequalities over the set of real numbers - (i) x

Solution :
10. Solve following Inequalities over the set of real numbers - (i) x

Solution :

11. Solve following Inequalities over the set of real numbers - (i) x

Solution :

12. Solve following Inequalities over the set of real numbers - (i) x

Solution :

13. Solve following Inequalities over the set of real numbers - (i) x
Solution :

14. Solve following Inequalities over the set of real numbers - (i) x

Solution :

15. Solve following Inequalities over the set of real numbers - (i) x

Solution :

16. Solve following Inequalities over the set of real numbers - (i) x
Solution :

17. Solve the following inequations

18. Solve the following inequations

19. Solve the following inequations

21. Solve :

LECTURE-10 DOUBLE INEQUALITY


AND BIQUADRATIC INEQUALITY
Example 10.1

WORKED-OUT PROBLEMS - 10

1.

2.

3.
4.

5. Solve the following inequations

6. Solve following Inequalities over the set of real numbers - (i) x

Solution :

7. Solve following Inequalities over the set of real numbers - (i) x

Solution :
8. Solve following Inequalities over the set of real numbers - (i) x

Solution :

9. Solve following Inequalities over the set of real numbers - (i) x

Solution :
10. Solve following Inequalities over the set of real numbers - (i) x

Solution :

11. Solve following Inequalities over the set of real numbers - (i) x

Solution :
12. Solve following Inequalities over the set of real numbers - (i) x

Solution :

13. Solve following Inequalities over the set of real numbers - (i) x

Solution :
LECTURE-11 GRAPH OF POYNOMIAL

In this section, we will be discussing about the identification of some of the functions through their graphs. In
particular, we discuss graphs of constant Linear, Quadratic, Cubic and Reciprocal functions.

Constant Function

Example 11.1

Draw the graph of f(x) = 3

Example 11.2

Plat the graph of f(x) =  1


2
x[ −1,2]
x(2,3]

Linear equation
Example 11.3
Draw the graph of

Example 11.4
Draw the graph of
Quadratic Function

Example 11.5
Draw the graph of

Example 11.6
Draw the graph of
Example 11.7
Draw the graph of f(x) = x2 – x – 6.

Example 11.8
Draw the graph of y = x2 – x – 2
Cubic
Quadratic function of time. These functions are not one-one. (why?)

Cubic Function
Reciprocal Function

Example 11.9
Draw the graph of y = x2 – x – 2
Example 11.10
Draw the graph of y = x3 + 3 for –3  x  3. Use your graph to find
a) the value of y when x = 2.5
b) the value of x when y = –15
x –3 –2 –10 1 2 3
y –24 –52 3 4 11 30

WORKED-OUT PROBLEMS - 11

1. What is the equation of the straight line shown in the diagram


5 3
(A) y − x + 2
3 5
5 3
(B) y = x + 2
3 5
(C) y = –0.6x + 2.6
(D) y = 0.6x + 2.6
Solution : (C) y = –0.6x + 2.6

2. What is the equation of the straight line shown in the diagram


(A) y = –3
(B) x = –3
(C) y = –3x + 3
(D) y = –3x – 3

Solution : (B) x = –3

3. Draw the graph of y = –x2 – 4x + 2


Solution :

4. Draw the graph of y = –x2 + 2x – 4


Solution :

5. Draw the graph of y = –(x – 5)2 + 3


Solution :

6.

Solution :

7.
Solution :

8.
Solution :

9.

Solution :

10.

Solution :
11.

Solution :

12.

Solution :
LECTURE-12 MODULUS EQUATIONS

1. To find the absolute value of any real number, first locate the number on the real line.

The absolute value of the number is defined as its distance from the origin.

2. For example, to find the absolute value of 7, locate 7 on the real line and then find its distance from the origin.

Example 12.1
Create a definition for the given absolute value function.
f(x) = | x + 1|
Solution :

Example 12.2
Sketch the graph of the given function on a sheet of graph paper
f(x) = |x – 1|
Solution :
Example 12.3
Use interval notation to describe the domain and range of the given function.
f(x) = –|x – 1| + 5
Solution :

Example 12.4

Solution :

Example 12.5

Solution :

Example 12.6

Solution :
Example 12.7

Solution :

Example 12.8

Solution :
Example 12.9

Solution :

Example 12.10

Solution :
Example 12.11
Solve : x |x + 3| + 2 |x + 2| = 0
Solution : Case–I x < –3
–x(x + 3) – 2(x + 2) = 0
x2 + 5x + 4 = 0  x = –1, – 4
 x = –4.  x = –1(reject)
Case–II –3 < x < –2
(x) (x + 3) –2x – 4 = 0
x2 + x – 4 = 0
−1 + 17 −1 − 17
 x= ,
2 2
−1 − 17 −1 + 17
 x= x= (reject)
2 2
Case–III x > –2
x(x + 3) + 2x + 4 = 0
x2 + 5x + 4 = 0
 x = –1, –4.
 x = –1  x = –4(reject)
−1 − 17
Hence x = –4, , –1.
2

Example 12.12
Solve the equation
(i) x2 +7|x| + 10 = 0
(ii) |3x – 2| + x = 11
(iii) |x + 1| + |x – 2| + |x – 5| = 2
Solution : (i) Method 1
x2 + 7|x| + 10 > 0  x  R
Hence x2 + 7|x| + 10 = 0
x
Method 2
x2 = |x|2
|x|2 + 7|x| + 10 = 0
|x|2 + 5|x| + 2|x| + 10 = 0
 |x| = –2, –5. (absurd)
 x
(ii) |3x – 2| + x = 11
Method–1 Here critical point is x = 2/3 so consider two cases:
2
Case–1 –  < x <
3
– (3x – 2) + x = 11
9
x= − permissible
2
2
Case–2  x < 
3
+ 3x – 2 + x = 11
13
 x=
4


final Solution x  − ,
9 13
2 4 
Method–2
|3x – 2| + x = 11  |3x – 2| = 11 – x (3x – 2) = (11 – x)
Taking +ve sign
13
3x – 2 = 11– x  3x + x = 13  x =
4
Taking –ve sign
9
(3x – 2) = –11 + x  3x – x = –11 + 2  x=−
2


Final Solution x  − ,
9 13
2 4 
(iii) |x + 1| + |x – 2| + |x – 5| = 2 Critical point → –1,2,5

Here –1, 2, 5 are three critical points hence four cases


Case–1 –  < x < –1
– (x + 1) – (x – 2) – (x – 5)
4
– 3x + 6 = 2  –3x = 4  x = − (not in the taken interval hence not permissible)
3
Case–2 1  x < 2
(x + 1) – (x – 2) – (x – 5) = 2  x = 6 (not in the taken interval hence not permissible)
Case–3 2  x < 5
(x + 1) + (x – 2) – (x – 5) = 2
 x+1+x–2–x+5=2
 2x – x + 4 = 2
 x = –2 (not in the taken interval hence not permissible)
Case–4 5  x < 
(x + 1) + (x – 2) + (x – 5) = 2
8
 x= (not in the taken interval hence not permissible) Final solution x  .
3

Example 12.13
Find the value of x, |x – 3| + 2 |x + 1| = 4
Solution : Here critical point are 3, –1
Case–I if x  3
|x – 3| + 2|x + 1| = 4
 (x – 3) + 2 (x + 1) = 4
 3x – 1 = 4
4 +1 5
x= = = 1.666 (approximate)
3 3
but here x  3 Hence, there is no value of x In this Interval
Case-II if –1  x < 3
|x – 3| + 2 |x + 1| = 4
 –(x – 3) + 2 (x + 1) = 4
 –x + 3 + 2x + 2 = 4
x+5=4
 x = –5 + 4 = –1
 x = –1
Case–III if x < –1
|x – 3| + 2 |x + 1| =4
 – x + 3 – 2x – 2 = 4
 –3x + 1 = 4
 –3x = 3
 x = –1
but x < –1, Hence there Is no value of x in this Interval
Taking union of all the three cases final solution is x  {–1}

Example 12.14
|x + 1| – |x| + 3|x – 1| – 2|x – 2| = x + 2
Solution : Here, –1, 0, 1, 2 are four critical points hence five cases

Case–I when x  2
|x + 1| – |x| + 3|x – 1|–2 |x – 2| = x + 2
 x + 1 – x + 3x – 3 – 2x + 4 = x + 2
x+2=x+2
Hence this is as Identity so all the values of this Interval will satisfy the equation
 x  2 x  [2, )
Case–II when 1  x < 2
|x + 1| – |x| + 3 |x –1| –2 |x – 2| = x + 2·
 (x + 1) – x + 3 (x – 1) + 2(x – 2) = x + 2
 x + 1 – x + 3x – 3 + 2x – 4 = x + 2
 5x – 6 = x + 2 .· .
 4x = 8 . . .
x=2
But 1  x < 2
Hence there ls no value of x in this Interval
Case–III when 0  x < 1
|x + 1| – |x| + 3|x –.1| – 2 |x – 2| = x + 2
 (x + 1) – x + (– 3) (x – 1) – (–2) (x – 2) = x + 2
 x + 1 – x – 3x + 3 + 2x – 4 = x + 2
 –x = x + 2
 –2x = 2
. x = –1 but 0  x < 1
hence, there is no value of x in this interval
Case–IV when –1  x < 0
|x + 1| – |x| + 3|x – 1| – 2|x – 2| = x + 2
 –(x + 1) + x – 3(x – 1) + 2(x – 2) = x + 2
 x + 1 + x – 3x + 3 + 2x – 4 = x + 2
x=x+2
0=2
Hence there is no solution for x
Case–V when x < – 1
|x + 1| – |x| + 2|x – 1| – 2 |x – 1| = x + 2
 –(x + 1) + x – 3(x – 1) + 2 (x – 2) = x + 2
 –x – 1 + x – 3x + 2x – 4 = x + 2
 –x – 2 = x + 2
 –2x = 4
 x = –2
Hence x  {–2}  [2, )

WORKED-OUT PROBLEMS - 12
1. Create a definition for the given absolute value function.
g(x) = |4 – 5x|
Solution :

2. Create a definition for the given absolute value function.


H(x) = |–x–5|
Solution :

3. Create a definition for the given absolute value function.


f(x) = x + |x|
Solution :

4. Sketch the graph of the given function on a sheet of graph paper


g(x) = |2x – 1|
Solution :

5. Sketch the graph of the given function on a sheet of graph paper


h(x) = |1 – 3x|
Solution :
6. Sketch the graph of the given function on a sheet of graph paper
f(x) = x – |x|
Solution :

7. Use interval notation to describe the domain and range of the given function.
f(x) = |– x|
Solution :

8. Use interval notation to describe the domain and range of the given function.
f(x) = (1/2)|x|
Solution :
9. Use interval notation to describe the domain and range of the given function.
f(x) = |x + 4|
Solution :

10. Use interval notation to describe the domain and range of the given function.
f(x) = |x| + 2
Solution :

11. Use interval notation to describe the domain and range of the given function.
f(x) = |x + 3| + 2
Solution :

12. Use interval notation to describe the domain and range of the given function.
f(x) = –|x – 2|
Solution :

13. Use interval notation to describe the domain and range of the given function.
f(x) = –|x| + 4
Solution :

14. Solve the equation


|x + 2| + 3 = 8
Solution :
15. Solve the equation
3|x – 5| = 6
Solution :

16.

Solution :
17. Solve the equation
|x| = –2
Solution :

18. Solve the equation


|x| = 3
Solution :
19. For each of the equations, provide a purely algebraic solution without the use of a calculator
|3 – 2x| = –1
Solution : Case 1 : 3 – 2x > 0
3 > 2x
3
x
2
3 – 2x = –1
x=2
Case 2 : 3 – 2x < 0
3< 2x
3
x
2
–(3 – 2x) = –1
3 – 2x = 1
2 = 2x
x=1
No Solution

20. Solve the following equation


(i) |x – 3| = 4
(ii) ||x – 1| + 1|| = 4
(iii) |x| – |x – 2| = 2
Solution : (i) |x – 3| = 4
 (x – 3) = ± 4
x=3±4
 x = 7, –1(these values satisfy the original equation);
Final solution x  {7, –1}
(ii) ||x – 1| + 1| = 4
 |x – 1| + 1 = ±4
 |x – 1| = ±4 – 1
 |x – 1| = 3 – 5
 |x – 1| = 3
x=1±3
 x = 4, –2 (these values satisfy the original equation)
(iii) |x| – |x – 2| = 2
 x, x  0
We know that |x| = 
− x, x  0
(x − 2) x  2
| x − 2 |= 
−(x − 2) x  2
Here x = 0, 2 are two critical points hence there are three intervals.
Case–1 when–  < x < 0
|x|–|x – 2| = 2
 –x + x – 2 = 2
 –2 = 2 it is not possible
Hence x  [2, )
Case–2 when 0  x < 2
|x| – |x – 2| = 2
 x– {–(x – 2)} = 2
x+x–2=2
 2x = 4
 x = 2 not in the taken interval  no solution x  
Case–3 when 2  x < 
|x| – |x – 2| = 2
 x – (x – 2) = 2
x–x+2=2
 2 = 2 it is an identity
Hence all the value in this interval.  x  [2, )
Final solution x  (2, )

21. Solve the equation |x – 3| + |x + 2| – |x – 4| = 3


Solution : |x – 3| + |x + 2| – |x – 4| = 3
x = 3, – 2, 4 are three critical points hence four cases

When x  4
|x – 3| + |x + 2| – |x – 4| = 3
x–3+x+2–x+4=3
x+6–3=3
x+3=3
 x = 0 (no solution)
when 3  x < 4
|x – 3| +· |x + 2| – |x – 4| =3
 (x– 3) + (x + 2) – {–(x – 4)} = 3
x–3+x+2+x–4=3
 3x – 5 = 3
8
x= (no solution)
3
when –2  x < 3
|x – 3| + |x + 2| – |x – 4| = 3
 –(x – 3) + (x + 2) – {–(x – 4)} = 3
 –x – 3 + x + 2 + x – 4 = 3
x+1=3
x=2
When x < –2
|x – 3| + |x + 2| – |x – 4| = 3
 –(x – 3) + {–(x + 2)} – {–(x – 4)} = 3
 –x + 3 – x – 2 + x – 4 = 3
 –x + 3 – 6 = 3
 –x – 3 = 3
 x = –6
Hence, x  {–6, 2}

22. Solve for x : 2|x + 1| – 2x = |2x – 1| + 1


Solution : Find critical points
x + 1 nnd 2x – 1 = 0
 x = – 1 and x = 0
so critical points are x = 0 and x = – 1
Consider following cases :
x  –1 ….(I)
2–(x + l) – 2x = –(2x – 1) + 1
2–x – l – 2x = – 2x + 2
 2–x – 1 = 2
 –x – 1= 1
 x = –2
As x = –2 satisfies (I), one solution is x = –2
–1 < x  0 ….(II)
2x + 1 – 2x = –(2x – 1) + 1
 2x + 1 = 2
x+1=1
x=0
As x = 0 satisfies (II), second solution is x = 0
x>0 ….(III)
2x + 1 – 2x = (2x –1) + 1
 2x + 1 = 2 x + 1
 identity in x, i.e. true for all x  R
On combining x  R with (III), we get :
x>0
Now combining all cases, we have the final solution as :
x  0 and x = –2

LECTURE-13 MODULUS INEQUALITY

Example 13.1

Solution :

Example 13.2

Solution :
Example 13.3

Solution :

Example 13.4

Solution :
Example 13.5

WORKED-OUT PROBLEMS - 13

1. Shade the solution of the inequality on the x-axis


|x| > –2
Solution :
2. Shade the solution of the inequality on the x-axis
|x| < 3
Solution :

3. Shade the solution of the inequality on the x-axis


|x| > 1
Solution :

4. Shade the solution of the inequality on the x-axis


|x|  0
Solution :
5. Shade the solution of the inequality on the x-axis
|x|  2
Solution :

6. Shade the solution of the inequality on the x-axis


|3 – 2x| > 5
Solution :

7. Shade the solution of the inequality on the x-axis


|4x + 5| < 7
Solution :
8. Shade the solution of the inequality on the x-axis
|4x – 5| > –2
Solution :

9. Shade the solution of the inequality on the x-axis


|2x – 9|  6
Solution :

10. Shade the solution of the inequality on the x-axis


|13 – 2x|  7
Solution :

11. Shade the solution of the inequality on the x-axis


|3x – 11| > 0
Solution :

12. Shade the solution of the inequality on the x-axis


1
|x + 2| < x+5
3
Solution :
13. Shade the solution of the inequality on the x-axis
|x – 12| > 6
Solution :

14. Shade the solution of the inequality on the x-axis


|2x + 11|  6
Solution :

15. Shade the solution of the inequality on the x-axis


|x – 8| < 7
Solution :
16.
Solution :

17.
Solution :

18.
Solution :
19.
Solution :

20.
Solution :
x2 + x + 1
21. Solve 0.
| x+ 1 |
x2 + x + 1
Solution : 0
| x+ 1 |
 |x + 1| > 0
 x  R – {–1}
 x2 + x + 1 > 0
 D = 1 – 4 = –3 < 0
 x2 + x + 1 > 0  x  R
 x  (–, –1)  (–1, )

x2 − 3x − 1
22. Solve : 3
x2 + x + 1
x2 − 3x − 1
Solution : 3
x2 + x + 1
 In x2 + x + 1
D – 1 – 4 = –3 < 0
 x2 + x + 1 > 0  x  R
 |x2 – 3x – 1| < 3(x2 + x + 1)
 (x2 – 3x – 1)2 – (13(x2 + x + 1))2 < 0
 (4x2 + 2)(–2x2 – 6x – 4) < 0
 (2x2 + 1)(x+ 2)(x + 1) > 0
 x  (–, –2)  (–1, )

23.
Solution :

24.
Solution :
25.
Solution :

Case : I

Case : II

Case : III

26.
Solution :
Case : I

Case : II

Case : III
Case : IV

27.
Solution :

Case : I
Case : II

Case : III

Case : IV
LECTURE-14 LOGARITHAM

Example 14.1
Example 14.2

Solution :

Example 14.3

Solution :

Example 14.4

Solution :
(product)

(Quotient)

(Power)
Proof (a)

Proof (b)

Proof (c)

Example 14.5

Solution : log 3 1.3


4

x = 1.3
3
x =1+
9
1 4
x =1+ =
3 3
−1
4 3
log 3 = log 3   = −1
4
3 4
4

Example 14.6

Solution : log 2 − 3
2+ 3
2− 3
2+ 3
2− 3
1
2− 3
(2 − 3 ) = −1
1 −1
log2− = log2−
3
2− 3 3

Example 14.7

Example 14.8

Example 14.9

Example 14.10
7

Example 14.11
7
Example 14.12
7

Example 14.13
7

Solution :

Example 14.14
7
Example 14.15
7

Solution :
Solution :

Example 14.16
7

Solution :

Example 14.17
7
Solution :

Example 14.18
7

Example 14.19
7

Example 14.20
7

Example 14.21
7
Example 14.22
7

Example 14.23
7
Solution :

WORKED-OUT PROBLEMS - 14

1.
Solution :

2.

Solution :

3.

Solution : 6

4.

43
Solution : (a) log1.43
30
x = 1.43
10x = 14.3
3
10x = 14 +
9
1
10x = 14 +
3
43
10x =
3
43
x=
30
43
log 43 =1
30
30
log 2 5
1
(b)  
2
1 −1
log 2 ( 2 )
( 5)log2  2  = ( 5 )

= ( 5) 2 = 1
1log 2
5

5.
Solution : 4log 2 2x = 36
(2x)log2 4 = 36
2
(2x)log2 2 = 36
(2x)2 = 36
4x 2 = 36
x2 = 9
x = 3
x = –3(reject), x = +3

6.

Solution :

7.
Solution :

8.

Solution :

9.
Solution :

10.

Solution :

11.

Solution :
12.

Solution :

13.

Solution :
14.

Solution :

15.

Solution :
16.

Solution :

17. Which of the following when simplified reduces to unity ?

18.

Ans. (A, C)
Ans. (C)

LECTURE-15 CHARACTERISTIC MANTISSA


Example 15.1

Example 15.2
Example 15.3

Example 15.4

Example 15.5

Example 15.6
Solution :

Example 15.7

Solution :

Example 15.8

Solution :

WORKED-OUT PROBLEMS - 15

1.

Solution :

2.
Solution :

3.
Solution :

4.

Solution :

Ans. (B)

5.

Solution :

6.

Solution :
7.

Ans. (C)

8.

9. Number of integers whose characteristic of logarithms to the base 10 is 3, is


(A) 8999 (B) 9000 (C) 90000 (D) 99000
Solution : Number of integer = 9 × 10n = 9 × 103 = 9000 Ans. (B)

10. If mantissa of logarithm of 719.3 to the base 10 is 0.8569, then mantissa of logarithm of 71.93 is
(A) 0.8569 (B) 1.8569 (C) 1.8569 (D) 0.1431
Solution : Mantissa = {logaN} = {log10719.3} = {log1071.93 + log1010} = {log10 71.93} = 0.8569 Ans. (A)
LECTURE-16 LOGRITHMIC EQUATIONS

Example 16.1

Solution :

x = 3 Ans.

Example 16.2

Solution :

Example 16.3

Solution :

Example 16.4

Example 16.5
Example 16.6

Example 16.7
Example 16.8

Example 16.9

Example 16.10
WORKED-OUT PROBLEMS - 16

1.
Solution : 3
3log3 x
= 27
3 =3
3log 3 x 3

3log 3 x = 3
log3 x = 1
x =3

2.
Solution : (log10x)2 – (log10x) – 6 = 0
Let log10x = P
P2 – P – 6 = 0
P2 – 3P + 2P – 6 = 0
(P + 2) (P – 3) = 0
P = –2 and P = 3
log10x = –2 log10x = 3
x = 10–2 x = (10)3
1
x= x = 1000
100

3.
Solution : 3(log7x + logx7) = 10
1 10
log7 x + =
log7 x 3
let log7x = P
 1
3 P +  = 10
 P
3P2 + 3 = 10P
3P2 – 10P + 3 = 0
3P2 – 9P – P + 3 = 0
(3P – 1)(P – 3) = 0
1
P= and P = 3
3
1
log7x = log7x = 3
3
x = 71/3 x = 73
x = 343

4.
Solution : (x + 2)log2 (x +2) = 8(x + 2)2
Take log2 both side
log 2 ( x + 2 )
log 2 (x + 2)
= log 2 8(x + 2)2
Log2(x + 2) = log2(x + 2) = log28 + 2log2(x + 2)
Let log2(x + 2) = P
P2 = 3 + 2P
P2 – 2P – 3 = 0
P2 – 3P + P – 3 = 0
(P + 1)(P – 3)
P = –1 P=3
log2(x + 2) = –1 and log2(x + 2) = 3
1
x+2= x+2=8
2
1
x= −2 x=6
2
−3
x= .
2

5.

Solution :
6.

Solution :
7.

Ans. (A, B, D)
8. If log10(x – 1)3 – 3 log10 (x – 3) = log108, then logx 625 has the value equal to :
(A) 5 (B) 4 (C) 3 (D) 2

Ans. (B)

9.

10.
11.

Ans. (C)

12.

Ans. (D)

13.

14.
15.

16.
17.

Ans. (A)

18.

Ans. (C)

19.

Ans. (B)
20.

Ans. (B)

21.

22.

Ans. (D)

23.

24.

25.

Ans. (C)
26.

Ans. (C,D)

27.

Ans. (A, B)

28.

Ans. (A, B, C, D)

29.
Ans. (A, B, C, D)

30.

Ans. (B)

31.

Ans. (B)

32.

Ans. (B, C, D)
33.

Ans. (A, B, D)

LECTURE-17 LOGRITHMIC INEQUALITIES


Example 17.1

Solution :

Example 17.2

Solution :

Example 17.3

Solution : (a) log0.5 (x2 – 5x + 6)  0 < x2 – 5x + 6  (0.5)–1


 0 < x – 5x + 6  2

x − 5x + 6  0
2
 2  x  [1, 2)  (3, 4]
x − 5x + 6  2

Hence, solution set of original inequation : x  [1, 2)  (3, 4]
(b) log1/3 (log4(x2 – 5)) > 0  < log4 (x2 – 5) < 1

0  log4 (x − 5)  x − 5  1
2 2
  1  (x2 − 5)  4
log4 (x − 5)  1  0  x − 5  4
2 2

 6 < x2 < 9
 x  ( −3, − 6 )  ( 6 , 3)
Hence, solution set of original inequation : x  ( −3, − 6 )  ( 6 , 3)

Example 17.4

Solution :
Example 17.5

Solution :

Example 17.6

Solution :
WORKED-OUT PROBLEMS - 17

1.
Solution : log 8 (–6x) < 1
0 < –6x < 8
0 > 6x > –8
−8
0>x>
6
−4
0>x>
3

2.
x0 x0
Solution : 
4−x 0 4x
x  (0, 4)
log6 x  log6 4 – x  
x>4–x
2x > 4
x>2
But x  (2, 4)
x  (2, 4)

3. Log10 ( x – 3)  2
Solution : x – 3 > 0
x>3
0 < (x – 3) < 102
0 < x – 3 < 100
3 < x < 103

4.
Solution :
5.
Solution :

6.

Solution :

7.
Solution :
8.
Solution :

9.
Solution :
10.

Solution :

11.

Solution :

12.

Solution :
13.

Solution :
14.

Solution :
15.

Solution :
16.

Solution :
17.
Solution :

18.

Solution :
19.

Solution :
20.

Solution :
21.

Ans. (A)

22. Solve the following inequations

23. Solve the following inequations


24.
LECTURE-18 IRRATIONAL INEQUALITIES

Example 18.1

Solution :

Example 18.2

Solution :

Example 18.3

Solution :
Example 18.4

Solution :
Example 18.5

Solve for x, if x2 − 3x + 2  x − 2 |
Solution :

Example 18.6

Solution :
WORKED-OUT PROBLEMS - 18

1.
Solution :

2.
Solution :

3.
Solution :
4.
Solution :

5.
Solution :

6.
Solution :
7.
Solution :

8.

Solution :
9.

Solution :

10.

Solution :
11.

Solution :
12.

Solution :
LECTURE-19 GREATEST INTEGER FUNCTION

GREATEST INTEGER OR STEP UP FUNCTION:


The function y = f(x) = [x] is called the greatest integer function where [x] denotes the greatest integer less than or
equal to x. Note that for :
Example 19.1

Solution :

Example 19.2

Solution :

Example 19.3

Solution :
Example 19.4
Find the fractional part of function for following
Solution : (i) {3.7} = 3.7 – 3 = 0.7
(ii) {39.2} = 39.2 – 39 = 0.2
(iii) {–3.7} = –3.7 – (–4) = 0.3
(iv) {–5.8} = –5.8 – (–6) = 0.2
(v) {7} = 7 – 7 = 0
(vi) {100} = 100 – 100 = 0

Example 19.5
Solve for x, if 4[x] = x + {x}
Solution : 4[x] = x + {x}
 4[x] = [x] + {x} + {x}
 3[x] = 2{x}
3
 {x} = [x]
2
If [x] = 0  {x} = 0  x = 0
3
If [x] = 1  {x} = , not possible
2
3
If [x] = –1  {x} = − , not possible
2
So, answer is x = 0.

Example 19.6
Solve the equation |2x – 1| = 3[x] + 2{x} where [.] denotes greatest integer and {.} denotes fractional part function.
Solution : We are given that, |2x – 1| = 3[x] + 2{x}
1
Let, 2x – 1  0 i.e. x  . The given equation yields.
2
1 – 2x = 3[x] + 2{x}
 1 – 2[x] – 2{x} = 3[x] + 2{x}  1 – 5[x] = 4{x}
1 − 5[x] 1 − 5[x]
 {x} =  0 1
4 4
3 1
 0  1 – 5[x] < 4  −  [x] 
5 5
3 1
Now, [x] = 0 as zero is the only integer lying between − and
5 5
1
{x} =
4
1 1 1
 x = which is less than , Hence is one solution.
4 2 4
1
Now, let 2x – 1> 0 i.e. x >
2
 2x – 1 = 3[x] + 2{x}
 2[x] + 2{x} – 1 = 3[x] + 2{x}
 [x] = –1
1
 –1  x < 0 which is not a solution as x >
2
1
 x = is the only solution.
4

WORKED-OUT PROBLEMS - 19

1. Evaluate :–
10
1. [0] 2. [5] 3. [5.28] 4. [.25] 5.
3
6. [–0.5] 7. [–2.99] 8. [–1.7] 9. [] 10. [ 2 ]
11. {3} 12. {3.1} 13. {–3.64} 14. {1} 15. { 2}
 −17 
16. {} 17.  
 5 
Solution :
1. 0 2. 5 3. 5 4. 0 5. 3
6. [–0.5] = –1 7. [–2.99] = –3 8. [–1.7] = –2 9. [] = 3 10. [ 2 ] = 1
11. {3} = 0 12. {3.1} = 3.1 – 3 = .1 13. {–3.64} –3.64 – [–3.64]– 3.64 – (–4) = 0.36 14. {1} = 0
−17
15.  2 = 2 − 1 = 0.4142 …. 16.  – 3 = 0.14159… 17. − {−4} =
3
5 5
2. Sketch a graph of GIF
y = [2x]
Solution :
3. Sketch a graph of GIF
y = [1.5x]
Solution :

4. Sketch a graph of GIF


y = [–0.25x]
Solution :
5. Sketch a graph of GIF
y = 2[x]
Solution :

6. Sketch a graph of GIF


y = [x + 3]
Solution :

7. Sketch a graph of GIF


y = –0.75[2 – x]
Solution :
9. Sketch a graph of GIF
y = [x2]
Solution :

10. Sketch a graph of GIF


y = [x]2
Solution :
11. Sketch a graph of Fractional part function (FPF)
y = {2x}
Solution :

12. Sketch a graph of Fractional part function (FPF)


y = {x}2
Solution :
LECTURE-20 PROBLEM ON GIF AND FPF

1. Solve for x : [x] = 7


Solution : [x] = 7
 x  [7, 8)

2. Solve for x : [2x – 1] = 99


Solution : [2x – 1] = 99
 2x – 1  [99, 100)
 101 
 2x  [100, 101)  x  50,
 1 

3. Solve the equation [2x + 5] = 9


Solution :
9  2x + 5 < 10
9.5  2x < 10 – 5
4  2x < 5
4 5
x
2 2
2  x < 2.5
x  [2, 2.5)

4. Solve the equation [1.25 + [x]] = 12


Solution :
Let [x] = u
[1.25 + u] = 12
12  1.25 + u < 13
12 < 1.25 + u < 13
10.75  u < 11.75
10.75  [x] < 11.75
11  x < 12

5. Solve : [5.4 – [1.3x]] = 17


Solution :
Let u = [1.3x]
[5.4 – u] = 17
17  5.4 – u < 18
17 – 5.4 < –u <18 – 5.4
11.6  – u < 12.6
–11.6  u > – 12.6
–11.6  [1.3x] > –12.6
–12  1.3x < –11
−12 −11
x
1.3 1.3
–9.23.7  x < –8.4615
 −120 −110 
x .
13 
,
 13

6. Solve : [x]  10
Solution : [x]  10
 x  (–, 11)

7. 7 < [5x – 1] < 20


Solution : 7 < [5x – 1] < 20
 5x – 1  [8, 20)
 5x  [9, 21)
 9 21 
 x , 
5 5 

1
8. Solve for x : [3x – 5] =
200
1
Solution : [3x – 5] = is a given equation but integer part of something is always an integer. So, the given equation
200
has no solution.

9. Solve for x : 2[x]2 – 10[x] + 12 = 1


1
Solution : Given equation can be written as [x]2 – 5[x] + 6 = L.H.S. is integer but RHS is fraction, hence the given
2
equation has no solution.

10. Solve the following inequality


[.] denotes GIF
{} denotes FPP
1
{x} 
2
 1
Solution : x  n, n +  n  1
 2

11. Solve the following inequality


[.] denotes GIF
{} denotes FPP
[x + [x]] <0
Solution : x + [x] < 0
x  (–, 0)

12. Solve the following inequality


[.] denotes GIF
{} denotes FPP
[2x2 – x] < 1
Solution : 2x2 – x < 1
2x2 – x – 1< 0
(2x + 1)(x – 1) < 0
 1 
x   − ,1
 2 

1
13. Find the domain of the function given by f (x) =
 − [x]
Solution :  – [x] > 0
 > [x]
The value of π is 3.14. Here, [x] returns integral value. Clearly, it can assume a maximum value of 3. But, GIF returns
integer value “n” for x < n + 1. The inequality, therefore, has solution given by :
x<4
Domain x Î (–, 4)

1
14. Find the domain of the function given by f (x) =
[x − 2]
Solution : Given function is in rational form having GIF as its denominator. The denominator should not evaluate to
zero for real values of x. The domain of GIF is real number set R. But, we know that GIF evaluates to zero in an interval
which is spread over unit value. In order to know this interval, we determine interval of x for which [x – 2] is zero.
 [x – 2] = 0
We can write this function as :
= [x + (–2)] = 0
Using property [x + y] = [x] + [y], provided one of x and y is an integer. This is the case here,
= [x + (–2)] = [x] + [–2] = 0
 [x] – 2 = 0
 [x] = 2
2x<3
x  [2, 3)
Domain R – [2, 3).

15. Solve [3.2 + [2.5x – 739]] = –5


Solution : Let u = [2.5x – 7.9]
[3.2 + u] = – 5
–5  3.2 + u < –4
–5 – 3.2  u < –4 – 3.2
–8.2  u < – 7.2
–8.2  [2.5x – 7.9] < –7.2
–8 < 2.5x – 7.9 < –7
–8 + 7.9  2.5x < –7 + 7.9
–0.1  2.5x  0.9
−0.1 0.9
x
2.5 2.5
x  [–0.04, 0.36).
LECTURE-21 BASIC TRANFORMATIONS ON GRAPHS

Example 21.1

Example 21.2
Example 21.3

Solution :
Example 21.4

Example 21.5
Solution :

Example 21.6

Solution :

Example 21.7
Example 21.8

Solution :

Example 21.9
Solution :

Example 21.10

Solution :

Example 21.11

Solution :
Example 21.12

Solution :
Example 21.13
Example 21.14

Example 21.15
Example 21.16

Example 21.17
Example 21.18

Solution :

Example 21.19
Example 21.20
Example 21.21

Find f(x) = max {1 + x, 1 – x, 2}


Solution :
1 − x; x  −1

From the graph it is clear that f(x) = 2 ; −1  x  1
1 + x; x  1

Example 21.22

Solution :
Example 21.23

Solution :

Example 21.24

Draw the graph of y =| e|x| − 2 | .

WORKED-OUT PROBLEMS - 21

1.
Solution :
2.
Solution :

3.

Solution :

4.
Solution :

5.

Solution :

6.

Solution :
7.

Solution :

8.

Solution :

9.

Solution :
10.

Solution :
11.

Solution :

LECTURE-22 COMPOSITE FUNCTION

Example 22.1

Solution :
Example 22.2

Solution :

1 1
(g o f)x = g(f(x))  g(x2 + 1)  =
x2 + 1 − 1 x2

Example 22.3

Solution :

Example 22.4

Solution :

Example 22.5

Solution :
WORKED-OUT PROBLEMS - 22

1.
Solution :

2.
Solution :

3.

Solution :
4.

Solution :

5.

Solution :

6.

Solution :

7. Given f(x) = x2 + 2x and g(x) = 6 – x2, find


(i) f + g,
(ii) f – g,
(iii) f g.
Determine the domain for each function in interval notation.
Solution :
(i) (f + g) (x) = 2x + 6, domain: (–, )
(ii) (f – g) (x) = 2x2 + 2x – 6, domain: (–, )
(iii) (fg) (x) = –x4 – 2x3 + 6x2 + 12x, domain: (–, )

8. Given f(x) = 3x2 and g(x) = x – 5 , find


(i) f + g,
(ii) f – g,
(iii) fg,
f
(iv) .
g
Determine the domain for each function in interval notation.
Solution :
(i) (f + g) (x) = 3x2 + x − 5 , domain: [5, )
(ii) (f – g) (x) = 3x2 – x − 5 , domain: [5, )
(iii) (f g) (x) = 3x2 x − 5 , domain: [5, )
f  3x2
(iv)   (x) = , domain: (5, )
g x −5

9. For the following exercises, use each set of functions to find


f(g(h(x))). Simplify your answers.
1
f(x) = x2 + 1, g(x) = , and h(x) = x + 3
x
Solution :
f(x) = x2 + 1
n
g(x) =
x
h(x) = x + 3
1
f(g(h(x)))  g(h(x)) =
x +3
2
 1 
f(g(h(x))) =   +1
 x +3

You might also like